You are on page 1of 108

8 Electromagnetic waves

Chapter
Electromagnetic waves- electromagnetic waves are EM-waves easily polarised which shows its

the waves which are produced due to the change transverse nature.

in electric field E and magnetic field B sinusoidally


Important characteristics of EM waves- the
and propagating through the space such that, the
characteristics of EM waves are listed below
two field are perpendicular to the direction of

wave propagation. 1. EM waves are produced by accelerating charge

particle.
Source of electromagnetic waves- An oscillating
2. They do not required medium to propagate.
charge is an example of accelerating charge. An
3. These waves travel in the free space with the
oscillating charge produces an oscillating electric
maximum speed same as speed of light (c=3×
field in the space, which produces an oscillating
108 m/sec). Given by the relation 𝑐 = . It means
1

magnetic field, which in turn produces an √𝜇0 𝜖0

that light waves are electromagnetic nature.


oscillating field and so on. The oscillating electric
4. The direction of variation of electric and magnetic
and magnetic fields generate each other as the
field are perpendicular to the each other and
wave propagates through space. The frequency of
perpendicular to the direction of propagation of
EM wave naturally equals the frequency of
EM waves. Thus electromagnetic waves are
oscillation of charge ʋ = .
1
2𝜋√𝐿𝐶
transverse in nature.
Electromagnetic waves are also produce when fast 5. In the free space, the magnitudes of electric and
moving electrons are suddenly stopped by metal magnetic fields in the electromagnetic waves are
target of high atomic number. These related by 𝑐 =
𝐸0
.
𝐵0
electromagnetic waves are known as X-rays. 6. The energy in electromagnetic waves is divided, on

an average, equally between electric and magnetic


Transverse nature of electromagnetic waves- it
fields. Ue=Um where Ue is the energy associated
can be shown from Maxwell’s equations that
with the electric field and Um is the energy
electric and magnetic field in an electromagnetic
associated with the magnetic field.
waves are perpendicular to each other and to the
7. The energy density (energy per unit volume) in an
direction of propagation of em-waves. Hence the
electric field E in the vacuum is 𝜖0 𝐸 2 and in the
1
electromagnetic waves are traverse in nature. 2
Chapter- 8 Electromagnetic waves
magnetic field is 𝐵2 . The energy associated with ii. These are used in TV waves ranging from 54
1
2𝜇0

the electromagnetic waves is given by U= MHz to 890 MHz.


1
𝜖 𝐸 2
+
1
𝐵2 . iii. These are used in FM (frequency Modulation)
2 0 2𝜇0

8. Electromagnetic waves are uncharged and are not ranging from 88 MHz to 108 MHz.

deflected by electric and magnetic fields. iv. UHF (Ultra high frequency) waves are used in

9. The electromagnetic waves like other waves carries cellular phones.

energy as well momentum. An em-waves also

exert a pressure on the surface called radiation Microwaves- Micro waves are called short radio

pressure. p= .
𝑈 waves which are produced by vacuum tubes. Their
𝑐

Note- light carries the energy from the sun to frequency lies in the range of 3×1011-1010Hz.

earth, thus making possible the life on the earth.


Name- Microwaves
Electromagnetic spectrum- the orderly
Wavelength- 10-3-10-1 m
arrangement of EM waves in increasing or
Frequency- 3×1022-1010Hz
decreasing order of wavelength 𝜆 or frequency ʋ is
Production- Klystron Valve or magnetron value
called electromagnetic spectrum. The range varies Detection- Point contact diode
from 10-12m to 104m from γ-rays to radio Uses- Cooking radar and communication
waves.
Use of microwaves are given below:

Electromagnetic spectrum i. These are used in the RADAR systems for

Radio-waves- Radio-waves are produced due to aircraft navigation.

oscillating charge particles. The frequency varies ii. These are used in microwaves oven for cooking
from 1010-3×104 Htz. purpose.

iii.These are used in study of atomic and


Name- Radio-waves molecular structures.
Wavelength- 10-1-10-4 m iv. These are used to measure the speed of vehicles,
Frequency- 1010-3×104Hz
speed of cricket balls, etc.
Production- Oscillating circuit
Detection- Rapid acceleration and de-acceleration Infrared Waves- These Waves were discovered by
of electron in aerials
Herschell. These waves are also called heat waves.
Uses- communication
These waves are produced from the heat radiating

bodies and molecules. These waves have high


Uses of radio waves are given below:
penetrating power. Its frequency range from
i. These are used in AM (Amplitude modulation)
3×1011-4×1014Hz.
from 530 kHz to 1710 kHz. These are also

used in the ground wave propagation. Name- Infrared Ray(IR)


Wavelength- 7.5×10-7-10-3m

Advent Tutorial Front of Nehru Inter College Banarsidas, Auraiya


KP Singh 7248370933 ks5122213@gmail.com Page 2
Chapter- 8 Electromagnetic waves
Frequency- 4×1014-3×1011Hz Ultraviolet Rays- These rays were discovered by
Production- Excitation of atom. Vibration of atom Ritter in 1801. UV-Rays are produced by special
or molecule
lamps and very hot bodies. The sun is an
Detection- Thermopiles, bolometer, infrared
important source of UV-Rays but fortunately
photographic film
absorbed by ozone layer. Its frequency range is
Uses- Physical therapy, infrared photography
warfare looking through fog from 7.5×1014-3×1016 Hz.

Name- Ultraviolet Rays (UV Rays)


Uses of infrared waves are given below:
Wavelength- 10-8-4×10-7 m

i. These are used in physical therapy. Frequency- 3×1016-7.5×1014Hz


Production- Excitation of atoms
ii. These are used in satellite for army purposes.
Detection- Photocells, Photo graphic film
iii. These are used in whether forecasting.
Uses- Sterilization and study of molecular
iv. These are used in dehydrated fruits.
structure, burglar alarm.
v. These are used in solar water heater, solar

cells and cooker. Uses of ultraviolet rays are given below:

Visible rays- It is the part of spectrum which is i. These are used in burglar alarms.
visible by human eye which range from 4×1014- ii. These are used in checking mineral sample.
7×1014 Hz. These radiations are produced by iii.These are used to study molecular structures.
excited atoms.
iv. To kill germs in minerals.

Name- Visible Light v. To sterilised surgical instruments.


Wavelength- 4×10-7-7.5×10-7 m
X-Rays- these rays were discovered by German
Frequency- 7.5×1014-4×1014Hz
professor Roentgen. Its frequency range is from
Production- Excitation of atom, inner shell
electrons moving from higher energy level to lower 3×1014-7.5×1016 Hz. Uses of X-Rays are given

energy level below:


Detection- The eye, photocell photographic film.
Uses- Seeing photography. Name- X-Rays
Wavelength- 1×10-10-1×10-8 m

The uses of visible rays are given below: Frequency- 3×1018-3×1016Hz

i. To see the things, avoid bumping from them Production- Bombarding of electron to high Z
targets
and escape danger.
Detection- Photographic film, Geiger tubes.
ii. To find stuff to eat.
Ionization chamber
iii. To find other living things with which to
Uses- Study of crystal structure diagnostic tool
consort so as to prolong the species. detection of art forgery

Advent Tutorial Front of Nehru Inter College Banarsidas, Auraiya


KP Singh 7248370933 ks5122213@gmail.com Page 3
Chapter- 8 Electromagnetic waves
i. These are used in surgery to detect the facture, Origin of Displacement current -Consider a

diseased organs, stones in the body, etc. parallel plate capacitor of plate A and B is

ii. These are used in engineering to detect fault, connected to a battery L through a tapping key K.

crack on bridges, testing of welds. after pressing the key K the conduction current

iii.These are used in metro stations to detect flow through the conducting wires. The capacitor

metal or explosive materials. starts storing the charge. As the charge on the

plate grows the conduction current in the wire


iv. These are used in scientific research.
decreases. When the capacitor is fully charged, the

conduction current stops flowing in the wire.


Gamma (γ) Rays- γ-Rays are discovered by During the charging during the charging of
Rutherford. They travel with the speed of light capacitor there is no conduction current between
and having high penetrating power. The the plates of capacitor. During the charging, let at
frequency ranges from 3×1018-3×1022Hz. an instant I be the conduction current in the

wires. This current will produce magnetic field


Name-Gamma rays
Wavelength- 1×10-13-1×10-10 m around the wires which can be detected by using a

frequency range- 3×1022-3×1018Hz compass needle. Here cosider a loop C1 of radius r.


Production- Radio decay of nucleus. C1 P Q
Detection- Photographic film, Geiger tubes. C2
Ionization chamber
Use- To study of structure of atom Cancer
treatment igniciating the nuclear reaction

Uses of γ-Rays are given below:


i. These are used to produce nuclear reactions.

ii. These are used in radio therapy for the


According to Ampere circuital law in the loop C1
treatment of tumour and cancer.
we have ∮ 𝐵
⃗ . 𝑑𝑙 = 𝜇0 𝐼________(a)
iii. These are used in food industry to kill

pathogenic microorganism. Now if we consider a point between the plates Q


iv. These are used to provide valuable information an tiffin type surface C2. we observe a deflection in
about the structure of atomic nucleus. magnetic compass placed at that point but there

is no such conduction current between the plates


Displacement current- Displacement current arises
and through the loop C2 ampere circuital law
due to changing electric field between the
⃗ . 𝑑𝑙 = 0 ______(b)
∮𝐵
capacitor plates. Electric field changes due to

variation in charge on capacitor plates. By the equation a and b we can say clearly that

Ampere circuital law is inconsistence for the loop

Advent Tutorial Front of Nehru Inter College Banarsidas, Auraiya


KP Singh 7248370933 ks5122213@gmail.com Page 4
Chapter- 8 Electromagnetic waves
C1 and C2 Maxwell make it consist he introduce Gauss law in magnetism- this law state that the

the concept of displacement current. Because in net magnetic flux through close surface is equal to

the expression some term are missing that missing zero. It is given by ∫ 𝐵
⃗ . 𝑑𝑠 = 0

term is displacement current the equation coming


This equation is called Maxwell’s second equation.
is known as generalised Ampere circuital law. It is
⃗ . 𝑑𝑙 = 𝜇0 (𝐼 + 𝐼𝑑 )________(c)
∮𝐵 Faraday law of electromagnetic induction- this

law state that the induced emf in a circuit is


According to him capacitor plates gets charged
numerically equal to rate of change of magnetic
due to conduction current. This produces an
flux through it. It is given by
electric field between the plates. Which changes

the electric flux and hence produce the ∫ 𝐸⃗ . 𝑑𝑙 = −


𝑑Ф𝑚
this equation is known as Maxwell’s
𝑑𝑡
displacement current between the plates. third equation.

Let the plates of the capacitor have instantaneous Ampere Circuital law- this law state that the line
charge q then according to gauss law in integral of magnetic field along a close path is
electrostatics the electric flux Ф𝑒 = or 𝑞 = 𝜖0 Ф𝑒
𝑞
𝜖0 equal to 𝜇0 times the total current threading the
differentiating both side with respect to t surface bounded by that close path. It is given by

or id=𝜖0 putting the value in the ) this equation is known as


𝑑𝑞 𝑑Ф𝑒 𝑑Ф𝑒 𝑑Ф𝑒
= 𝜖0 ⃗ . 𝑑𝑙 = 𝜇0 (𝐼 + 𝜖0
∮𝐵
𝑑𝑡 𝑑𝑡 𝑑𝑡 𝑑𝑡

equation c we have that is Maxwell’s fourth equation.

) generalised ampere circuital Objective exercise


𝑑Ф𝑒
⃗ . 𝑑𝑙 = 𝜇0 (𝐼 + 𝜖0
∮𝐵 𝑑𝑡

law
1. A linearly polarised electromagnetic waves

Maxwell’s equations- Maxwell’s equations are the given as 𝐸 = 𝐸0 cos(𝑘𝑧 − 𝜔𝑡) 𝑖̂ is incident normally
on a perfect reflecting infinite wall at z=a.
basic laws of electricity and magnetism. These
assuming that the material of wall is optically
equations are gave complete description of all
inactive, the reflected wave will be given as
electromagnetic interactions. Maxwell on the basis
a. 𝐸 = −𝐸0 cos(𝑘𝑧 − 𝜔𝑡) 𝑖̂
of these equations, predicted the existence of b. 𝐸 = 𝐸0 cos(𝑘𝑧 + 𝜔𝑡) 𝑖̂
electromagnetic waves. These are four equations c. 𝐸 = −𝐸0 cos(𝑘𝑧 + 𝜔𝑡) 𝑖̂
d. 𝐸 = 𝐸0 sin(𝑘𝑧 − 𝜔𝑡) 𝑖̂ Ans (b)
which are explained as given below:

Gauss law in electrostatics- this law state that the 2. Light with energy flux of 20 W/cm2 fall on a
non-reflecting surface at normal incidence. If
total electric flux through any close surface is
the surface has an area of 30 cm2, the total
always equal to times charge enclose by the
1
𝜖0 momentum delivered (for complete absorption)
surface. It is given by ∫ 𝐸⃗ . 𝑑𝑠 =
1
𝜖0
(𝑞) during 30 minutes is
a. 36× 10−5 kgm/sec
This equation is called Maxwell’s first equation
b. 36× 10−4 kgm/sec

Advent Tutorial Front of Nehru Inter College Banarsidas, Auraiya


KP Singh 7248370933 ks5122213@gmail.com Page 5
Chapter- 8 Electromagnetic waves
c. 36× 104 kgm/sec incorrect option from the following;
d. 36× 107 kgm/sec Ans (b) a. The associated magnetic field is given as B
= 𝑘̂ × 𝐸 =
1 1
(𝑘̂ × 𝐸)
𝑐 𝜔
3. Electric field intensity produce by the radiation b. The electromagnetic field can be written in
coming from 100 W bulb at a 3 m distance is terms of associated magnetic field as
E. electric field intensity produced by the E=c(𝐵 × 𝑘̂ )
radiations coming from 50 W bulb at the same c. 𝑘̂ . 𝐸 = 0, 𝑘̂. 𝐵 = 0
distance is d. 𝑘̂ × 𝐸 = 0, 𝑘̂ × 𝐵 = 0 Ans (d)

a. E/2 c. E/√2
b. 2E d. √2 E Ans (d) 8. A plane electromagnetic waves propagating
along x direction can have the following pair of

4. If E and B represent electric and magnetic field E and B

vector of electromagnetic waves respectively, a. Ex, By c. Ey, Bx

the direction of propagation of electromagnetic b. Ey, Bz d. None of these

waves is along Ans (b)

a. E c. B×E
b. B d. E×B Ans (d) 9. A charge particle oscillates about its mean
equilibrium position with a frequency of 109

5. An em-wave radiate from a dipole antenna, Hz. For producing electromagnetic waves which

with E0 as the amplitude of its electric field one is not true?

vector. The electric field E0 which transports a. They will have frequency of 109 Hz

significant energy from the source falls off as b. They will have frequency of 2×109 Hz

a. 1/r3 c. 1/r c. They will have a wavelength of 0.3 m.

b. 1/r2 d. Remains constant d. They fall in the region of radio waves.

Ans (c) Ans (b)

6. An electromagnetic waves travels in vacuum


along z direction 10. The source of electromagnetic waves can be a

E = (E1𝑖̂+E2𝑗̂) cos(𝑘𝑧 − 𝜔𝑡). Choose the correct charge

options from the following: a. Moving with constant velocity.

a. The associated magnetic field is given as b. Moving in circular orbit.

B=
1
(E1𝑖̂-E2𝑗̂) cos(𝑘𝑧 − 𝜔𝑡) c. At rest
𝑐
b. The associated magnetic field is given as d. Falling in a magnetic field. Ans (b)

B= (E1𝑗̂-E2𝑖̂) cos(𝑘𝑧 − 𝜔𝑡)


1
𝑐
c. The given electromagnetic waves is circular 11. Which of the following have minimum wave

polarised length

d. The given electromagnetic wave is un- a. Blue light c. infrared

polarised. Ans (a) b. γ-ray d. Microwaves


Ans (b)

7. An electromagnetic waves travelling along z-


axis is given as E=E0cos(𝑘𝑧 − 𝜔𝑡). Choose the 12. Which of the following have minimum

Advent Tutorial Front of Nehru Inter College Banarsidas, Auraiya


KP Singh 7248370933 ks5122213@gmail.com Page 6
Chapter- 8 Electromagnetic waves
penetrating power? field gives rise to
a. Ultraviolet radiation a. An electric field
b. Microwaves b. An induced emf
c. γ-ray c. A magnetic field
d. Radio waves Ans (c) d. A magnetic dipole Ans (c)

13. Electromagnetic waves travelling in a medium 19. Electromagnetic waves are transverse in nature
having relative permeability 𝜇𝑟 = 1.3 and is evidence by
relative permittivity 𝜖𝑟 = 2.14. the speed of a. Polarisation c. Reflection
electromagnetic waves in medium must be b. Interference d. Diffraction
a. 1.8×108 m/sec c. 1.8×106 m/sec Ans (a)
b. 1.8×104 m/sec d. 1.8×102 m/sec
Ans (a) 20. Which of the following are not electromagnetic
waves?
14. In an electromagnetic wave the phase a. Cosmic rays c. β-Rays
difference between electric field and magnetic b. γ-Rays d. X-Rays
field vectors are Ans (c)
a. Zero c. 𝜋/2 21. 10 cm is a wavelength corresponding to the
b. 𝜋/4 d. 𝜋 Ans (a) spectrum of
a. Infrared c. Microwaves
15. The quantity √𝜖0 𝜇0 represent b. Ultraviolet rays d. X-rays
a. Speed of sound Ans (c)
b. Speed of light in the vacuum
c. Speed of em waves 22. If 𝐸⃗ and 𝐵
⃗ represent electric and magnetic field
d. Inverse of speed of light in vacuum vector of the electromagnetic waves then the
Ans (d) direction of propagation of the em-waves is
that of
16. In electromagnetic waves if ue and um are mean a. 𝐸⃗ .𝐵
⃗ c. 𝐸⃗ × 𝐵⃗
electric and magnetic energy densities b. ⃗ . 𝐸⃗
𝐵 d. ⃗𝐵 × 𝐸⃗
respectively, then Ans (c)

a. ue = um c. u e < um 23. The structure of solid is investigated by

b. ue > um d. u e2 =
1
um2 a. Cosmic rays c. γ-Rays
2
Ans (a) b. X-rays d. infrared
Ans (b)

17. which of the following is called heat radiations


a. x-Rays c. Infrared 24. the condition under which a microwave oven

b. γ-Rays d. Microwaves heats up a food item containing water

Ans (c) molecules most efficiently is


a. Frequency of microwaves must match the

18. From Maxwell’s hypothesis, a charging electric resonant frequency of water molecules.
b. The frequency of the microwaves has no

Advent Tutorial Front of Nehru Inter College Banarsidas, Auraiya


KP Singh 7248370933 ks5122213@gmail.com Page 7
Chapter- 8 Electromagnetic waves
relation with natural frequency of the wavelength of e.m waves is
water molecules. a. 0.80 cm c. 2×10-2 m
c. Microwaves are heat waves, so always b. 1×103 m d. 0.67 cm
produce heating. Ans (d)
d. Infrared waves produce heating in a Fill in the blanks
microwave oven. Ans (a)
1. Electromagnetic waves possess both_________ and
25. Which of the radiation are used in treatment of
________ . Ans. Energy, momentum
muscle ache?
2. Electromagnetic waves produce by bombardment
a. Infrared c. Microwaves
of high energy electrons on a metal target
b. Ultraviolet d. X-Rays
is_________. Ans. X-rays
Ans (a)
3. Electromagnetic waves used in RADAR are ________
26. Correct option, if speeds of gamma rays, X-
Ans. Microwaves
rays and microwaves are Vg Vx and Vm
4. Displacement current is produced in the region of
respectively will be
changing__________ . Ans. displacement current
a. Vg > Vx > Vm c. Vg > Vm > Vx
5. Ampere Maxwell’s law state that line integral of
b. V g < Vx < Vm d. Vg = Vx = Vm
magnetic field along the boundary of a closed path
Ans (d)
is equal to ______________. Ans. 𝝁𝟎 (𝑰𝒄 + 𝑰𝒅 )
27. Waves in decreasing order of wavelength are
6. Displacement current across a capacitor is _________
a. X-rays, Infrared rays, Visible rays, Radio-waves
b. Radio-waves, Visible rays, Infrared rays, X- to conducting current in conducting wires.
rays Ans. equal
c. Radio-waves, , Infrared rays, Visible rays, X- 7. Electromagnetic waves are produced by _________
rays electric charge. Ans. oscillating
d. Radio-waves, ultraviolet, visible rays, X-rays 8. Electromagnetic radiation of frequency 5×1019
Ans (c) Htz belong to __________ region.
28. The electric field associated with an e.m-waves Ans. X-ray, γ-ray
in vacuum is given by 𝐸⃗ = 40 cos(𝑘𝑧 − 6 × 108 𝑡)𝑖̂ , 9. Ascending order of frequency for the radiation
where E, Z and t are in volt/metre and infrared, visible, Ultraviolet are _________.
seconds respectively. The value of wave vector k Ans infrared > visible > Ultraviolet
is 10. Radio waves have the frequency range__________ Htz.
a. 2 m-1 c. 6 m-1
b. 0.5 m-1 d. 3 m-1 Ans (a) Very short answer type questions (1 Marks)

29. Electromagnetic waves with wavelength 𝜆 are


1. To which part of electromagnetic spectrum does a
used by a FM radio station for broadcasting.
wave of frequency 5×1019 Hz belong?
Here 𝜆 belongs to
Ans. the wave of frequency 5×1019 Hz belong to
a. Radio waves c. UHF radio waves
X-ray or γ-rays of em spectrum.
b. VHF radio waves d. microwaves
2. An electromagnetic wave exerts pressure on the
Ans (b)
surface on which it is incident. Justify.
30. The oscillating magnetic field in a plane
3. Welder wear special goggles or face marks with
electromagnetic waves is given as
glass windows to protect their eyes from the
By= (8×10-6)sin[2 × 1011 𝑡 + 300𝜋𝑥] Tesla,

Advent Tutorial Front of Nehru Inter College Banarsidas, Auraiya


KP Singh 7248370933 ks5122213@gmail.com Page 8
Chapter- 8 Electromagnetic waves
electromagnetic radiation. Name the radiations 11. When an ideal capacitor is charged by a DC
and write the range of their frequency. battery, no current flows. However when AC
4. What are the direction of electric field and source is used, the current flows continuously. How
magnetic field vector relative to the each other does one explain this based on the concept of
and relative to the direction of propagation of displacement current?
electromagnetic waves? 12. Explain briefly how electromagnetic waves are
5. Name the electromagnetic waves which produced related to that of the oscillating charge?
i. Maintain the earth’s warmth and
Short answer type questions (3 Marks)
ii. Are used for air craft navigation
6. A plane electromagnetic waves travel in vacuum 13. Write the expression for generalized expression
along z-direction. What can you say about the form of the Ampere’s circuital law. Discuss its
direction of electric and magnetic field vectors? significance and describe briefly how the concept
of displacement current is explained through
Short answer type questions (2 Marks)
charging/discharging of a capacitor in an electric
7. (i) How are electromagnetic waves produced? circuit.
(ii) How do you convince yourself that 14. Name the part of electromagnetic spectrum which
electromagnetic waves carry momentum and is
energy? i. Suitable for the RADAR systems in air craft
8. i. Arrange the following electromagnetic waves in navigations.
the descending order of their wavelengths. ii. Used to treat muscular strain.
a. Microwaves iii. Used as diagnostic tool in medicine
b. infrared rays Write brief, how these waves are produced.
c. ultraviolet radiation 15. Answer the following questions.
d. 𝛾-rays i. Name the electromagnetic waves which are used
ii. Write one use each of any two of them. for the treatment of certain forms of cancer.
9. i. an electromagnetic waves is travelling in a Write their frequency range.
medium with a velocity 𝑣 = 𝑣𝑖̂. Draw a sketch ii. Thin ozone layer on top of stratosphere is crucial
showing propagation of the oscillating electric and for human survival. Why?
magnetic fields. iii. Why is the amount of the momentum transferred
ii. How the magnitudes of the electric field and by electromagnetic waves incident on the surface
magnetic fields related to velocity of the so small?
electromagnetic wave? 16. Answer the following questions.
10. A capacitor of capacitance C is being charged by i. Name the electromagnetic waves which are
connecting it across DC source along with an produced during the radioactive decay or
ammeter. Will the ammeter show a momentary nucleus. Write their frequency range.
deflection during the process of charging? If so, ii. Welders wear special glass goggales while
how would you explain this momentary deflection working. Why? Explain.
and the resulting continuity of the current in the
Why infrared waves are often called as heat
circuit? Write the expression for the current inside
the capacitor? waves? Give their one application.

Advent Tutorial Front of Nehru Inter College Banarsidas, Auraiya


KP Singh 7248370933 ks5122213@gmail.com Page 9
9 Ray Optics
Mirror- Are the smooth and silver surface which
Chapter reflect major amount of light in to the same
Light- Light is the form of energy which provides medium are called mirror.
us the sensation of visibility. It is travel in in the a. Plane mirror
form of electromagnetic waves. b. Spherical mirror
c. cylindrical mirror.
Properties of light-
i. Light always follow a straight path. Spherical mirrors- Are the mirror which is formed
ii. It consists of seven colours in the form of by the spherical surfaces by polishing them
VIBGYOR. silvered. They are of two types
iii. When light fall over the smooth and silver (a) convex mirror (b) concave mirror.
surface it return in to the same medium is
called reflection of light. Centre of curvature- The centre of the sphere by
iv. When pass through two transparent medium which this spherical surface are formed is called
it deviate from its path it called refraction of centre of curvature.
light.
v. When the light pass through a prism it split Radius of curvature- The radius of spherical
in to its seven colours it called dispersion of surface by which the mirror is formed is called
light. radius of curvature.
vi. When the light fall over the metallic surface it
ejaculate the electron from it is called photo- Principle axis- A line from the centre of curvature
electric effect of light. and pass through the mirror is called principle
axis.
Laws of reflection-
1. Incident ray reflected ray and normal to the Pole- Point of intersection of principle axis to the
surface lie in the same plane. mirror is called pole.
2. Angle of incident and angle of reflection are
equal for a reflecting surface. Focus- Is a point on principle axis at which the
ray coming from infinity pass after the reflection
through mirror is called focus.
i r
r r Focal length- The separation between the pole and
focus is called focal length. The focal length is equal
to half of radius of curvature. f= R/2
Chapter- 9 Ray Optics

Mirror formulae- The relation between the object (d).


M
(C). M
distance image distance and focal length is called
mirror formulae = +
𝟏 𝟏 𝟏
𝒇 𝒗 𝒖 C F O
C F P

Sign convention- N
N

1- All the measurement are taken from pole.


2- The direction of light taken positive while
opposite of that are taken negative. (e). M
(f).
3- Direction upward to the principle axis taken
positive and down ward taken negative. P
M
C F
4- Focal length of convex mirror or lens is
positive while that of concave mirror or lens N
P
C F
are negative.
N

Magnification- The image height to the object


height is defined as the magnification. Image formed by the convex mirror- Convex
𝑚 =
ℎ𝑖
= −
𝑣
. mirror formed the virtual erect diminish image
ℎ𝑜 𝑢
between the pole and focus of the object placed

Image formed by the concave mirror between pole and infinity.


Positions of the Object Position of Image Nature and Size of
Image
Uses of concave mirror- Concave mirror are used
1. Object at infinity Image formed at the Real, inverted,
in fig (a) focus or in the focal extremely in head light of vehicles and used by dentists.
plane diminished in size

2. Object beyond the Image is formed Real, inverted, and


centre of curvature between the focus diminished in size Formation of Image by Convex Mirror for Different
but at a finite and the centre of
Positions of the Object:-
distance curvature
in fig (b). Position of Object Position of Image Nature and Size
Position of
3. Object at centre of Image formed at the Real, inverted, and
curvature in fig (c) centre of curvature diminished in size as Image
itself that of object. Object at infinity Image is formed at Virtual, erect and
4. Object between the Image is formed Real, inverted, and
see fig. a the principal focus, extremely
centre of curvature beyond centre of larger than object
behind the mirror diminished
and focus in fig (d) curvature
Object between Image is formed Virtual, erect and
5. Object at the focus Image is formed at Real, inverted, and
in figure (e) infinity highly magnified infinity and the between the the diminished

6. Object between the Image is formed Virtual, erect and pole (i.e., at a finite principal focus and
focus and pole behind the mirror magnified. distance) pole, behind the
in figure (f)
see fig. (b) mirror
(b). M
(a). M
A Ray diagrams for the image formation by a convex
mirror:
C F P
C P
F
N
N

Advent Tutorial Front of Nehru Inter College Banarsidas, Auraiya


KP Singh 7248370933 ks5122213@gmail.com Page 2
Chapter- 9 Ray Optics
M
(a). M
(b). Snell’s rule- The ratio of sine of incident angle to
A the refracted angle equal to a constant which is
A’
O O called relative refractive index of second medium
F C B B’ F C to the first 1µ2
N
=1 µ 2 = = =
sin 𝑖 µ2 𝑐/𝑣2 𝑣1 𝜆1
=
cos 𝑟 µ1 𝑐/𝑣1 𝑣2 𝜆2
N
Uses of convex mirror- These mirror are used side
view mirrors and at the metro stations. Lens formulae- The relation between the object
distance image distance and focal length is called
Refraction of light- When light ray goes from one the lens formulae
𝟏
=
𝟏
̶
𝟏
𝒇 𝒗 𝒖
transparent medium to other transparent
medium it deviate from its path is called Lens maker formulae- the focal length in terms of
refraction of light. Refraction occurs due to the the refractive index and
change in the speed of light. Frequency of light Radius of curvature is called the lens maker
remains same because it is the fundamental formulae
𝟏
= (
µ2
̶ 1) (
1
̶
1
)
𝒇 µ1 𝑅1 𝑅2
property of light which depends only on the source
of light where it produces. In refraction the
Ray diagrams for the image formation by a convex lens
wavelength also changes. Positions of the Object Position of Image Nature and Size of Image
Object at infinity Image formed at the focus Real, inverted, extremely
in fig (a) or in the focal plane diminished in size
𝜇1
i Object beyond the 2F but at Image is formed between Real, inverted, and
𝑀𝑒𝑑𝑖𝑢𝑚 1 𝑣1 a finite distance the focus and 2F diminished in size
in fig (b).
r
𝜇2 Object at 2F Image formed at the 2F Real, inverted, and

𝑀𝑒𝑑𝑖𝑢𝑚 2 in fig (c) itself diminished in size as that


𝑣2 of object.

Laws of refraction- Object between 2F and F Image is formed beyond 2F Real, inverted, and larger
in fig (d) than object
1- Incident ray refracted ray and normal to the
surface lie in the same plane. Object at the focus Image is formed at infinity Real, inverted, and highly
in figure (e) magnified
2- When light ray goes from denser medium to
Object between the Image is formed behind Virtual, erect and
rare medium it move away to the normal. focus and optical centre object magnified.

3- When light ray goes from rare medium to in figure (f)

Ray diagrams for the image formation by convex lens


denser medium it move towards the normal. M
(a). (b) M
.

Absolute refractive index- It is defined as the ratio


F1 2F2
2F1 F1 O 2F2
F1
2F1 F1 O
of speed of light in the vacuum to the speed in the
A MN (d) M
(c).
medium it is unit less and dimension less quantity
N
. A
B F1 2F2 F1 2F2
which depends only on medium. µ=
𝑐 B’
O B’ B O
𝑣 2F1 F1 2F1 F1
Relative refractive index- It is defined as the ratio A’ A
N N A’
of absolute refractive index of one medium to the
(f). ’
(e).
other 1µ2 = = =
µ2 𝑐/𝑣2 𝑣1 𝜆1 M M
=
µ1 𝑐/𝑣1 𝑣2 𝜆2 A
O
B’ O
2F1 F1 F1 2F2 2F1 F1 B F1 2F2

N N

Advent Tutorial Front of Nehru Inter College Banarsidas, Auraiya


KP Singh 7248370933 ks5122213@gmail.com Page 3
Chapter- 9 Ray Optics
Ray diagrams for the image formation by a concave Critical angle- The maximum incident angles
Lens:- beyond which the incident light must get reflect
Position of Object Position of Image Size of Nature of completely through the boundary of the two
Position Image Image
transparent media. The critical angle 𝜃𝑐 = sin−1 ( )
1
Object at infinity Image is formed at the extremely Virtual, µ

see fig. (a) focus on same side of lens diminished erect


as the object.
Object at anywhere Image is formed between diminished Virtual,
between infinity and the focus and optical erect
Medium air
the optical centre see centre and on the same
fig. (b) side of the lens as the
object. 𝜃𝑐 Dancer medium µ
(b).
(a). M
M

O
O
Dispersion of light- The splitting of light into its
2F1 F1
2F1 F1
seven component when White light fall on a prism
N is called the dispersion of light and the pattern
N
is called dispersion pattern.
Magnification of lens- The image height to the The reason of dispersion is that the refractive
object height is defined as The magnification of the index of a medium is different
lens. 𝑚 = .
ℎ𝑖 𝑣
For different wave length of light µ = A+ and
𝐵
=
ℎ𝑜 𝑢 𝜆2
Effect on nature and focal length on the lens on angle of deviation depend on the Refractive index
putting in to the medium of refractive index µ𝑚 of the medium. The minimum angle of deviation of
and the refractive index of the medium is µ𝑙 then light is given by 𝛿𝑚 = (µ ̶1)𝐴. Where A is prism
1. If µ𝑚 > µ𝑙 then nature of the lens will invert angle.
convex will behave like concave and concave
behave like convex. Angle of deviation- The angle between incidents
2. If µ𝑚 < µ𝑙 then nature of the lens will not light to the emergent light through prism is called
change convex will behave like convex and angle of deviation. The angle of deviation depends
concave behave like concave. on the incident angle for a particular angle the
3. If µ𝑚 = µ𝑙 then lens will disappear in such angle of deviation is minimum.
medium or behave like the glass slab.
Prism- Is transparent medium covered with two
Total internal reflection- The reflection of light at rectangular surface at some angle is called prism
the interface of two media when it goes from an and angle between the surface is called prism
optical denser medium to optically rare medium is angle.
called total internal reflection. For total internal
reflection to take place two conditions must be Optical instrument- The combination of lens
satisfied (a) the incident light must travel from whose purpose to form an image which can be
denser to rare medium (b) The angle of incident detect clearly is called optical instrument. These
must be greater than the critical angle. are microscope and telescope.

Advent Tutorial Front of Nehru Inter College Banarsidas, Auraiya


KP Singh 7248370933 ks5122213@gmail.com Page 4
Chapter- 9 Ray Optics
Microscope- Is an optical instrument which forms 1. A converging lens is used to form an image on
a magnified image of small object like bacteria or a screen. When the upper half of the lens is
dust particles. It works on the principle of mutual covered by an opaque screen.
image formation it consist of two convex lens (a) half the image will disappear

objective (small focal length and small aperture) (b) incomplete image will be formed.
(c) intensity of image will decrease but complete
and eye piece (small focal length and large
image is formed.
aperture). The image form by objective behaves
(d) intensity of image will increase but image is not
like the object for the eye piece. The magnification
distinct.
produced by the simple microscope
Ans. (c)
i. m= object placed at infinity
𝐷
𝑓 2. In optical fibres, the refractive index of the
ii. m = 1+ when image formed at D = 25 cm.
𝐷
𝑓 core is
for compound microscope (a) greater than that of the cladding.
i. m= ̶ ( ) final image at infinity
𝐿 𝐷
𝑓𝑜 𝑓𝑒 (b) equal to that of the cladding.
ii. m=̶ (1 + ) final image at D = 25cm.
𝐿 𝐷
𝑓𝑜 𝑓𝑒
(c) smaller than that of the cladding.
(d) independent of that of cladding.
Telescope- It is the optical instrument which Ans. (a)
forms a clear image of planet satellite. It consist of 3. An object is placed at a distance of 0.5 m in
two convex lenses objective (large focal length and front of a plane mirror. The distance between
large aperture) eye piece (small aperture and object and image will be
small focal length). Its magnification power (a) 0.25 m (b) 0.5 m
i. m= final image formed at infinity (c) 1.0 m (d) 2.0 m Ans. (c)
𝑓𝑜
̶
𝑓𝑒
ii. m=
𝑓𝑜
(1 + ) final image formed at D = 25
𝑓𝑜
4. Air bubble in water behaves as
𝑓𝑒 𝐷

cm. (a) sometimes concave, sometimes convex lens


(b) concave lens

Resolving power of microscope- It is the ability of (c) convex lens

micro scope to show as two closely lying objects. It (d) always refracting surface Ans. (b)

is given by the expression RP =


2µ sin 𝜃
where µ is 5. We combine two lenses, one is convex and
𝜆
the refractive index of the medium of lens θ is other is concave having focal lengths f1 and f2

semi cone angle and 𝜆 is wave length of the light and their combined focal length is F.

used. Combination of the lenses will behave like


concave lens, if

Resolving power of telescope- It is the ability of (a) f1 > f2 (b) f1 = f2

telescope to show as distinct two closely lying (c) f1 < f2 (d) f1 ≤ f2 Ans. (a)

objects. It is given by the expression RP =


𝐷 6. The length of an astronomical telescope for
1.22𝜆
where D is the aperture of the telescope. normal vision (relaxed eye) will be
(a) 𝑓0 − 𝑓𝑒 (b)
𝑓0
𝑓𝑒

Very Short Answer/Objective Type Questions [1 (c) 𝑓0 × 𝑓𝑒 (d) 𝑓0 + 𝑓𝑒 Ans. (d)

Mark) 7. The focal length of a biconvex lens of radii of


each surface 50 cm and refractive index 1.5,
is

Advent Tutorial Front of Nehru Inter College Banarsidas, Auraiya


KP Singh 7248370933 ks5122213@gmail.com Page 5
Chapter- 9 Ray Optics
(a) 40.4 cm (b) 75 cm 14. In the formation of a rainbow, the light from
(c) 50 cm (d) 80 cm Ans. (c) the sun on water droplets undergoes
8. A metal coin is at bottom of a beaker filled (a) dispersion only. (b) only TIR.
with a liquid of refractive index = 4/3 to (c) dispersion and TIR. (d) scattering.
height of 6 cm. To an observer looking from Ans. (b)
above the surface of liquid, coin will appear at 15. In an experiment to find focal length of a
a depth concave mirror, a graph is drawn between
(a) 1.5 cm (b) 6.75 cm the magnitude of u and v. The graph looks
(c) 4.5 cm (d) 7.5 cm Ans. (c) like
9. Four lenses of focal lengths ± 15 cm and
b.
± 150 cm are available for making a a.
telescope. To produce the largest v v
magnification, the focal length of the eyepiece
should be
(a) +15 cm (b) +150 cm u u
(c)- 150 cm (d) - 15 cm Ans. (a) c. d.
10. If a convex lens of focal length 80 cm and a v
v
concave lens of focal length 50 cm are
combined together, what will be their
resulting power? u u
(a) +6.5 D (b) -6.5 D Ans. (c)
(c) +7.5 D (d) -0.75 D Ans. (d) 16. A convex lens of refractive index 3/2 has a
11. A convex lens and a concave lens, each having power of 2.5 D in air. If it is placed in a liquid
the same focal length of 25 cm, are put in of refractive index 2 then the new power of
contact to form a combination of lenses. The the lens is
power of the combination (in dioptres) is (a) -1.25 D (b) - 1.5 D
(a) zero (b) 25 (c) 50 (d) infinity (c) 1.25 D (d) 1.5 D Ans. (a)
Ans. (a) 17. A ray of light incident at an angle θ on a
12. The refractive index of the material of an refracting face of a prism emerges from the
equilateral prism is √3. What is the angle of other face normally. If the angle of the prism
minimum deviation? is 5° and the prism is made of a material of
(a) 45° (b) 60° (c) 37° (d) 30° refractive index 1.5, the angle of incidence is
Ans. (b) [NCERT Exemplar]
13. An object is immersed in a fluid. In order that (a) 7.5° (b) 15° (c) 5° (d) 2.5°
the object becomes invisible, it should Ans. (a)
(a) behave as a perfect reflector. 18. A short pulse of white light is incident from
(b) absorb all light falling on it. air to a glass slab at normal incidence. After
(c) have refractive index one. travelling through the slab, the first colour to
(d) have refractive index exactly matching emerge is [NCERT Exemplar]
with that of the surrounding fluid. Ans. (d) (a) blue (b) green (c) violet (d) red
Ans. (d)

Advent Tutorial Front of Nehru Inter College Banarsidas, Auraiya


KP Singh 7248370933 ks5122213@gmail.com Page 6
Chapter- 9 Ray Optics
19. An object approaches a convergent lens from (b) act as a concave lens for the objects that
the left of the lens with a uniform speed 5 lie on its curved side.
m/s and stops at the focus. The image (c) act as a convex lens irrespective of the side
[NCERT Exemplar] on which the object lies.
(a) moves away from the lens with an (d) act as a concave lens irrespective of side
uniform speed 5 m/s. on which the object lies.
(b) moves away from the lens with an Ans. (c)
uniform acceleration. 22. The direction of ray of light incident on a
(c) moves away from the lens with a non- concave mirror is shown by PQ while
uniform acceleration. directions in which the ray would travel after
(d) moves towards the lens with a non- reflection is shown by four rays marked 1, 2,
uniform acceleration. 3 and 4. Which of the four rays correctly
Ans. (c) shows the direction of reflected ray? [NCERT
20. You are given four sources of light each one Exemplar] 1
providing a light of a single colour- red, blue, (a) 1 2 Q 4
green and yellow. Suppose the angle of (b) 2
C F
refraction for a beam of yellow light (c) 3
corresponding to a particular angle of (d) 4 3
P
incidence at the interface of two media is Ans. (b)
90°. Which of the following statements is 23. The optical density of turpentine is higher
correct if the source of yellow light is replaced than that of water while its mass density is
with that of other lights without changing the lower. Figure shows a layer of turpentine
angle of incidence? [NCERT Exemplar] floating over water in a container. For which
(a) The beam of red light would undergo total one of the four rays incident on turpentine in
internal reflection. figure the path shown is correct? [NCERT
(b) The beam of red light would bend towards Exemplar] 1 2 3
normal while it gets refracted through the (a) 1
second medium. (b) 2 Air

(c) The beam of blue light would undergo (c) 3


Turpentine
total internal reflection. (d) 4
(d) The beam of green light would bend away Ans. (b) Water
from the normal as it gets refracted through
the second medium. 24. A car is moving with at a constant speed of
Ans. (c) 60 km/h on a straight road. Looking at the
21. The radius of curvature of the curved surface rear view mirror, the driver finds that the
of a plano-convex lens is 20 cm. If the car following him is at a distance of 100 m
refractive index of the material of the lens be and is approaching with a speed of 5 km h. In
1.5, it will [NCERT Exemplar] order to keep track of the car in the rear, the
(a) act as a convex lens only for the objects driver begins to glance alternatively at the
that lie on its curved side. rear and side mirror of his car after every 2 s
till the other car overtakes. If the two cars

Advent Tutorial Front of Nehru Inter College Banarsidas, Auraiya


KP Singh 7248370933 ks5122213@gmail.com Page 7
Chapter- 9 Ray Optics
were maintaining their speeds, which of the Ans. (a)
following statement (s) is/are correct? 27. An astronomical refractive telescope has an
[NCERT Exemplar] objective of focal length 20 m and an
(a)The speed of the car in the rear is 65 km eyepiece of focal length 2 cm. Which one of
h. the following is not correct?
(b) In the side mirror the car in the rear (a) The length of the telescope tube is 20.02 m.

would appear to approach with a speed of 5 (b) The magnification is 1000.


(c) The image formed is inverted.
km/h to the driver of the leading car.
(d) An objective of a larger aperture will increase
(c) In the rear view mirror the speed of the
the brightness and reduce chromatic aberration of
approaching car would appear to decrease as
the image.
the distance between the cars decreases.
Ans. (d)
(d) In the side mirror, the speed of the
28. Virtual image formed by convex mirror has
approaching car would appear to increase as
Magnification _________________. Ans. negative
the distance between the cars decreases.
29. Optical denseness of a medium is measured in
Ans. (d)
terms of _________________ . Ans. refractive index
25. Consider an extended object immersed in
30. Minimum angle of incidence in the denser
water contained in a plane trough. When seen
medium for which angle of refraction becomes
from close to the edge of the trough the
90° is called _______________ . Ans. critical angle
objeet looks distorted. Which of the following
31. Relation between critical angle and refractive
is not correct.
index is ________________ . Ans. 𝝁 =
𝟏
𝐬𝐢𝐧 𝜽𝒄
(a) the apparent depth of the points close to
32. Optical fibre works on the principle of ________
the edge are nearer the surface of the water
. Ans. Total internal reflection
compared to the points away from the edge.
33. The splitting of white light into its constituent
(b) the angle subtended by the image of the
colours when it passes through a glass prism is
object at the eye is smaller than the actual
called ______________ . Ans. dispersion
angle subtended by the object in air.
34. Blue colour of sky is due to phenomenon of
(c) some of the points of the object far away
of _______________ sunlight. Ans. scattering
from the edge may not be visible because of
total internal reflection.
Very short answer type questions (1 Marks)
(d) water in a trough acts as a lens and
1. A ray of monochromatic light passes from
magnifies the object.
medium (1) to medium (2). If the angle of
Ans (d)
incidence in medium (1) is and the
26. A magnifying glass is used, as the objeet to be
corresponding angle of refraction in medium
viewed can be brought closer to the eye than
(2) is θ/2, which of the two media is optically
the normal near point. This results in
denser? Give reason. [Foreign 2013]
(a) a larger angle to be subtended by the
Ans. Given: i = θ, r = θ/2
object at the eye and hence viewed in greater 𝑆𝑖𝑛𝑖 𝑛2
= 𝑠𝑖𝑛𝑟 < 𝑠𝑖𝑛𝑖 ⇒ 𝑛2 > 𝑛1
detail. 𝑆𝑖𝑛𝑟 𝑛1
Hence, 2nd medium is optically denser.
(b) the formation of a real inverted image.
2. For the same value of angle of incidence, the
(c) increase in the field of view.
angles of refraction in three media A, B and
(d) infinite magnification at the near point.

Advent Tutorial Front of Nehru Inter College Banarsidas, Auraiya


KP Singh 7248370933 ks5122213@gmail.com Page 8
Chapter- 9 Ray Optics
Care 15°, 25° and 35° respectively. In which index cannot be less than unity as it is given
medium would the velocity of light be by the relation 𝑛 = .
𝑐
𝑣
minimum? [AI 2012] 8. For which material the value of refractive
Ans. 𝑛 = = Thus, the medium for which index is (i) minimum and (ii) maximum?
𝑠𝑖𝑛𝑖 𝑐
𝑆𝑖𝑛𝑟 𝑣
angle of refraction is of 15°, the speed of Ans. (i) Refractive index is minimum for
light is minimum. vacuum (µ = 1).
3. A concave lens of refractive index 1.5 is (ii) Refractive index is maximum for diamond.
immersed in a medium of refractive index 9. Can virtual image be photographed?
1.65. What is the nature of the lens? [Delhi Ans. Yes, because the lens in the camera
2015] produces a real image of the virtual image
Ans. The nature of the lens is converging being formed.
4. A lens behaves as a converging lens in air and 10. How does the angle of minimum deviation of
a diverging lens in water (µ = 4/3). What will a glass prism vary, if the incident violet light
be the condition on the value of refractive is replaced with red light? [Chennai 2019, AI
index (µ) of the material of the lens? [Delhi 2017]
2011C] Ans. It decreases.
Ans. The refractive index µ of the lens is less 11. An object is first seen in red light and then in
than the refractive index of water, ie violet light through a simple microscope. In
4
3
> 𝜇𝐿 > 1 which case is the magnifying power larger?
5. An air bubble is formed inside water. Does it Ans. As we know m = 1 +
𝐷
𝑓𝑉 < 𝑓𝑅
𝑓
act as a converging lens or a diverging lens? So, the magnifying power is larger when the
Ans. An air bubble behaves as a diverging lens object is seen in violet light.
inside the water. 12. Name the phenomenon due to which one
6. The image of an object formed by a lens on cannot see through the fog.
the screen is not in sharp focus. Suggest a Ans. Due to scattering of light, one cannot see
method to get the clear focussing of the through the fog.
image on the screen without disturbing the 13. How does the angle of minimum deviation of
position of the object, the lens or the screen. a glass prism of refractive index 1.5 change, if
Ans. The image of an object formed by a lens it is immersed in a liquid of refractive index
can be brought to a sharp focus on a unfixed 1.3?
screen by changing the focal length of the lens Ans. It will decrease.
by any of the following methods: 14. A convex lens is placed in contact with a
(i) By placing another lens of suitable focal plane mirror. A point object at a distance of
length in contact with the previous lens. 20 cm on the axis of this combination has its
(ii) By immersing the given lens in a liquid of image coinciding with itself. What is the focal
appropriate refractive index. length of the lens? [Delhi 2014]
7. Can absolute value of refractive index of a Ans. The focal length of convex lens is 20 cm.
medium be less than unity? 15. If an object is moved from infinity to convex
Ans. As the speed of light is maximum in mirror then in which direction will the image
vacuum, therefore absolute value of refractive shift?

Advent Tutorial Front of Nehru Inter College Banarsidas, Auraiya


KP Singh 7248370933 ks5122213@gmail.com Page 9
Chapter- 9 Ray Optics
Ans. The image will shift from focus to the Ans. Given: 𝑖 = 2𝐴, 𝑟 = 90 − (90° − 𝐴) = 𝐴
convex mirror. 𝑛=
𝑠𝑖𝑛𝑖 𝑠𝑖𝑛2𝐴 2𝑠𝑖𝑛𝐴𝑐𝑜𝑠𝐴
= =
𝑠𝑖𝑛𝑟 𝑠𝑖𝑛𝐴 𝑠𝑖𝑛𝐴
16. A green light is incident from water to the 𝑛 = 2𝑐𝑜𝑠𝐴
air-water interface at the critical angle (θ).
Which part of the spectrum will come out in
the air medium? 90°

Ans. The spectrum of visible light whose


2A A

frequency is less than that of green light will


come out of the water into the air.
17. How do the increasing (i) wavelength and (ii) 22. Does the magnifying power of a microscope
intensity of light affect the speed of light in depend on the colour of the light used?
glass? Justify your answer. [Foreign 2017]
Ans. (i) ∵ 𝑣 ∝ 𝜆; ∴ speed of light increases on Ans. Magnifying power of a microscope,
𝐿 𝐷
increasing the wavelength in glass. 𝑚 = − (1 + )
𝑓0 𝑓𝑒
(ii) There is no effect on speed of light on Since the focal length of a convex lens
changing the intensity. depends on the refractive index, and
18. Name the principle on which an optical fibre refractive indices for different colours are
works. different, so according to the lens maker's
Ans. Total internal reflection. formula
1 1 1
= (𝑛 − 1) ( − )
𝑓 𝑅1 𝑅2
19. Why is there no dispersion in the light The magnifying power of a microscope
refracted through a rectangular glass slab? depends on the colour of the light used.
Ans. A glass slab can be considered as two
glass prisms pieced together and the position Short Answer Type Questions [2 Marks]
of one prism is inverted w.r.t. another. 1. (a) Write the necessary conditions for the
Therefore, the various colours of white light phenomenon of total internal reflection to
dispersed by the first prism are again occur
combined to form white light. (b) Write the relation between the refractive
20. The line AB in the ray diagram represents a index and critical angle for a given pair of
lens. State whether the lens is convex or optical media.
concave. [Delhi 2019, 2013]
Ans. It is a diverging (concave lens) and the Ans. (a) the necessary condition for total
refracted ray are bending away from the internal reflection occur is
principal axıs. i. light ray must go from denser to rare
21. A ray of light incident on one of the faces of a medium.
glass prism of angle A has angle of incidence ii. incident angle must greater than the
2A. refracted ray in the prism strikes the critical angle.
opposite face which is silvered, the reflected (b) aµb =
1
; where 𝜃𝐶 is the critical angle.
𝑠𝑖𝑛𝜃𝐶
ray from it retracing its path. Trace the ray 2. What is the relation between critical angle
gram and find the relation between the and refractive index of a material?
refractive index of the material of the prism (ii) Does critical angle depend on the colour of
and the angle of the prism. light? Explain.

Advent Tutorial Front of Nehru Inter College Banarsidas, Auraiya


KP Singh 7248370933 ks5122213@gmail.com Page 10
Chapter- 9 Ray Optics
Ans. (i) 𝜇 = some distant lake of water. This illusion is
1
𝑠𝑖𝑛𝑖𝐶
(ii) For a given medium, µ depends on the called a mirage). Explain.
colour Ans. The air layers closer to the ground are
As 𝜇𝑉 > 𝜇𝑅 so, the critical angle 𝑖𝐶 =
1
sin−1 ( ) is hotter than higher layers. Oblique rays coming
𝜇
greater for red colour from distant sky, therefore, travel from

3. If half of the convex lens is painted black, denser to rarer parts of the atmosphere and

then draw ray diagram to show the image get more and more oblique. When the angle of

formation. How will this image formed by the incidence exceeds, critical angle (for denser

lens be different from painted lens? air rarer air interface), rays gets totally

Ans. Although complete image will be formed. reflected and may enter the observer's eye.

But due to decrease in the area of the The observer, therefore, sees a reflected image

refracting aperture, brightness of the image of the distant part of the sky.

will be reduced. 9. (i) State the principle on which the working

4. Define the power of a lens. draw a plat of an optical fibre is based.

showing the variation of a power of a lens, (ii) What are the necessary conditions for this

with the wavelength of the incident light. phenomenon to occur?

5. Using the lens formula, show that an object Ans. (i) Total internal reflection.

placed between the optical centre and the (ii) Light must travel from a denser into a

focus of a convex lens produces a virtual and rarer medium and the angle of incidence

an enlarged image. should be more than the critical angle, where


1
6. A convex lens of focal length 𝑓1 is kept in 𝑖𝐶 = sin−1 ( )
𝜇

contact with a concave lens of 𝑓2 . Find the 10. Figure shows a ray of light passing through a
90°
focal length of the combination. [AI 2013] prism. If the refracted ray QR is parallel
A
to
7. Define the magnifying power of a compound the base BC show that
microscope when the final image is formed at (i) r1 = r2 = A/2 Q R
i e
infinity. Why must both the objective and the (ii) angle of minimum r1 r2
P S
eyepiece of a compound microscope has short 𝐷𝑚 = 2𝑖 − 𝐴.
B C
focal lengths? Explain. [Delhi 2017]
Ans. Magnifying power: It is the ratio of the
angle subtended by the image formed at 11. (a) The bluish colour predominates in clear
infinity to the angle subtended on the eye by sky.
the object placed at least distance of distinct (b) Violet colour is seen at the bottom of
vision. spectrum when white light is dispersed by a

𝑚=− .
𝐷 𝐿 prism.
𝑓𝑒 𝑓𝑜
12. Show that the power of a lens is 𝑃 =
1

Therefore, to increase angular magnification, 𝑓


13. (i) Why does the Sun appear reddish at sunset
𝑓𝑜 and 𝑓𝑒 should be small.
or sunrise? [AI 2016]
8. On a hot summer day in a desert, one sees
(ii) For which colour the refractive index of
the reflected image of distant parts of the
prism material is maximum and minimum?
sky. (This is sometimes mistaken by the
observer to be the reflection of the sky in

Advent Tutorial Front of Nehru Inter College Banarsidas, Auraiya


KP Singh 7248370933 ks5122213@gmail.com Page 11
Chapter- 9 Ray Optics
14. Explain the basic differences between the of real image formation by a convex lens of
construction and working of a telescope and a power + 5D. One of these observations is
microscope. [AI 2015] incorrect. Identify this observation and give
15. A thin double convex lens of focal length fis reason for your choice:
broken into two equal halves at the axis. The
two halves are combined as shown in figure. Serial No. 1 2 3 4 5 6
Object distance (cm) 25 30 35 45 50 55
What is the focal length of combination in (ii)
and (iii)? Image distance (cm) 97 61 37 35 32 30

19. A ray PQ incident normally on the refracting


face BA is refracted in the prism BAC made
of material of refractive index 1.5. Complete
the path of ray through the prism. From
which face will the ray emerge? Justify your A
16. Using the data given below, state which two answer. [AI 2016] P
of the given lenses will be preferred to
Q
construct a (i) telescope, and (ii) microscope.
Also indicate which is to be used as objective
60°
and as eyepiece in each case. [DoE] [AI 2017] B C

Long Answer Type [I] Questions [3 Marks]


Lenses Power Aperture 1. A point source S is placed mid-way between
(P) (A) two concave mirrors having equal focal length
L1 6D 1 cm f as shown in figure. Find the value of d for
L2 3D 8 cm which only one image is formed.
L3 10D 1 cm
S
17. A right angled prism made from a material
d
of refractive index µ is kept in air. A ray PQ
is incident normally on the side AB of the
prism as shown. A 2. With the help of a suitable ray diagram,
derive a relation between the object distance
P Q
(u), image distance (v) and radius of
curvature (R) for a convex spherical surface,

B
θ when a ray of light travels form rarer to
C
Find (in terms of µ) the maximum value of µ denser medium. [Delhi 2011C]
up to which this incident ray necessarily 3. (a) How is the focal length of a spherical
undergoes total internal reflection at the face mirror affected when it is immersed in
AC of the prism. glycerine?
18. The following data was recorded for values of (b) A convex lens has 15 cm focal length in
object distance and the corresponding values air. What is its focal length in water?
of image distance in the experiment on study (Refractive index of air-water = 1.33,
refractive index of air-glass

Advent Tutorial Front of Nehru Inter College Banarsidas, Auraiya


KP Singh 7248370933 ks5122213@gmail.com Page 12
Chapter- 9 Ray Optics
= 1.5) obtain expression for total magnification
4. A convex lens made up of glass of refractive when the image is formed at infinity.
index 1.5 is dipped, in turn, in (i) a medium 10. A convex lens of material of refractive index
of refractive index 1.65, (ii) a medium of n1, is kept in a medium of refractive index n2.
refractive index 1.33. The parallel rays of light are incident on the
(a) Will it behave as a converging or a lens Complete the path of rays of light
diverging lens in the two cases? refracted from the lens when
(b)How will its focal length change in the two (i) n2 = n1
media? [AI 2011] (ii) n2 > n1, and
5. Define power of a lens. Write its units. Deduce (iii) n2 < n1
the relation for two thin lenses kept
1 1 1
= +
𝑓 𝑓1 𝑓2
in contact coaxially. [Dehradun 2019, Foreign Long Answer Type [II] Questions [5 Marks]
2012 Al 2017] 1. (a) Draw a ray diagram to show image
6. Show that a convex lens produces an N times formation when the concave mirror produces
magnified image when the object distances, a real, inverted and magnified image of the
from the lens, have magnitudes (𝑓 ± ). Here f
𝑓
object.
𝑁
is the magnitude of the focal length of the (b) Obtain the mirror formula and write the
lens. Hence find the two values of object expression for the linear magnification.
distance, for which a convex lens, of power (c) Explain two advantages of a reflecting
2.5 D, will produce an image that is four telescope over a refracting telescope. [CBSE
times as large as the object? 2018]
7. (a) Draw a labelled ray diagram of a 2. (a) A point object O is kept in a medium of
compound microscope. [Al 2017] refractive index n1, in front of a convex
(b) Derive an expression for its magnifying spherical surface of radius of curvature R
power which separates the second medium of
(c) Why is objective of a microscope of short refractive index n2, from the first one, as
aperture and short focal length? Give reason shown in the figure.
8. Draw a schematic labelled ray diagram of a n1 n2
reflecting type telescope. [Panchkula 2019, AI C
O
2017]
u R
(ii) Write two important advantages justifying
why reflecting type telescopes are preferred
over refracting telescopes. [AI 2017, Foreign Draw the ray diagram showing the image
2017] formation and deduce the relationship
(iii) The objective of a telescope is of larger between the object distance and the image
focal length and of larger aperture (compared distance in terms of n1, n2 and R. [Foreign
to the eyepiece). Why? Give reasons. 2017]
[Chennai 2019, Foreign 2017, Delhi 2017,
Foreign 2013] (b) When the image formed above acts as a
9. Draw a ray diagram showing the image virtual
formation by a compound microscope. Hence

Advent Tutorial Front of Nehru Inter College Banarsidas, Auraiya


KP Singh 7248370933 ks5122213@gmail.com Page 13
Chapter- 9 Ray Optics
object for a concave spherical surface material of the prism is √3, determine the
separating the medium n2, from n1,(n2> n1), values of angle of incidence and angle of
draw this ray diagram and write the similar deviation. [AI 2015]
[similar to (a)] relation. Hence obtain the 6. (a) A ray of monochromatic light is incident
expression for the lens maker's formula. [Delhi on one of the faces of an equilateral
2015] triangular prism of refracting angle A. Trace
3. A thin convex lens having two surfaces of the path of ray passing through the prism.
radii of curvature R1 and R2 is made of a Hence derive an expression for the refractive
material of refractive index µ2. It is kept in a index of the material of the prism in terms of
medium of refractive index µ1. Derive, with the angle of minimum deviation and its
the help of a ray diagram, the lens maker's refracting angle. [Foreign 2017]
formula when a point object placed on the (b) Three light rays red (R), green (G) and
principal axis in front of the radius of blue (B) are incident on the right angled
curvature R1, produces an image I on the prism abc at face ab. The refractive indices of
other side of the lens. [Foreign 2013] the material of the prism for red, green and
4. How would you estimate rough focal length of blue wavelengths are respectively 1.39, 1:44
a converging lens? Draw a ay diagram to and 1.47. Trace the paths of these rays
show image formation by a diverging lens. reasoning out the differences in their
Using this diagram, derive the relation behaviour. [Foreign 2011]
between object distance u, image distance v A
and focal length of the lens. Sketch the graph B
between 1/u and 1/v for this lens. G

5. (a) Two thin convex lenses L1 and L2 of focal R


lengths f1 and f2 respectively, are placed co- B
45°
C
axially in contact. An object is placed at a
point beyond the focus of lens L1. Draw a ray 7. (i) Plot a graph to show variation of the angle
diagram to show the image formation by the of deviation as a function of angle of incidence
combination and hence derive the expression for light passing through a prism. Derive an
for the focal length of the combined system. expression for refractive index of the prism in
(b) A ray PQ incident on the face AB of a terms of angle of minimum deviation and
prism ABC, as shown in the figure, emerges angle of prism. [Dehradun 2019]
from the face AC such that AQ = AR. (ii) What is dispersion of light?
A What is its cause?
(iii) A ray of light incident
Q R normally on one face of a
right isosceles prism is totally reflected as

P
shown in figure. What must be the minimum
B C
value of refractive index of glass? Give
Draw the ray diagram showing the passage of relevant calculations. (Delhi 2016)
the ray through the prism. If the angle of the
prism is 60° and refractive index of the

Advent Tutorial Front of Nehru Inter College Banarsidas, Auraiya


KP Singh 7248370933 ks5122213@gmail.com Page 14
Chapter- 9 Ray Optics
8. (i) A ray of light incident on face AB of an half-filled when viewed from the top of the
equilateral glass prism, shows minimum container. (aµw = 4/3)?
deviation of 30°. Calculate the speed of light 14. How does the refractive index of a
through the prism. A transparent medium depend on the
wavelength of incident light used? Velocity of
light in glass is 2x108 m/s and in air is 3x108
m/s. If the ray of light passes from glass to
air, calculate the value of critical angle.

B C [Foreign 2015]
(ii) Find the angle of incidence at face AB so 15. Calculate the speed of light in a medium
that the emergent ray grazes along the face whose critical angle is 30°.
AC. [Delhi 2017] 16. Velocity of light in a liquid is 1.8x108 m/s
9. (a) Draw a ray diagram showing image Find by how much the bottom of vessel
formation in a compound microscope. Define containing this liquid appears to be raised if
the term limit of resolution' and name the the depth of the liquid is 25 cm.
factors on which it depends. How is it related 17. A convex lens of focal length 20 cm and
to resolving power of microscope? made of glass (µ = 1.5) is immersed in water
(b) Suggest two ways by which the resolving of -1.33. Calculate change in focal length of
power of a microscope can be increased. the lens. [Foreign 2017]
(c) “A telescope resolves whereas a microscope 18. An object is placed 15 cm in front of a
magnifies.” Justify this statement. [Foreign convex lens of focal length 10 cm. Find the
2015] nature and position of the image formed.
Where should a concave mirror of radius of
Numerical Problems curvature 20 cm be placed so that the final
10. A ray of light passes through an equilateral image is formed at the position of the object
prism (refractive index 1.5) such that angle of itself? [AI 2015]
incidence is equal to angle of emergence and 19. An equi-convex lens of focal length f is cut
the latter is equal to 3/4th of angle of prism. into two identical plane convex lenses. How
Calculate the angle deviation. will the power of each part be related to the
11. Light falls from glass to air. Find the angle of focal length of the original lens?
incidence for which the angle of deviation is A double convex lens of + 5D is made of glass
90°, if refractive index of glass is √2. of refractive index 1.55 with both faces of
12. A light ray passes from air into a liquid as equal radii of curvature. Find the value of its
shown in figure. Find refractive index of radius of curvature. [Foreign 2015]
liquid. [aµb = √3/2] 20. A ray of light falls on a transparent sphere
60°
with centre C as shown in the figure. The ray
emerges from the sphere parallel to the line
AB. Find the angle of refraction at A if
15°
refractive index of the material of the sphere
13. How much water should be filled in a is √3. [Foreign 2014]
container 21 cm in height, so that it appears

Advent Tutorial Front of Nehru Inter College Banarsidas, Auraiya


KP Singh 7248370933 ks5122213@gmail.com Page 15
Chapter- 9 Ray Optics
Air
Air
ray diagram and find the location of the
A point at which the beam would now converge.
B
60° C [Delhi 2011C]
28. Trace the path of a ray of light passing
through a glass prism (ABC) as shown in the
21. The radii of curvature of the faces of a double figure. If the refractive index of glass is √3,
convex lens are 10 cm and 15 cm. If focal find out the value of the angle of emergence
length of the lens is 12 cm, find the from the prism. [Foreign 2012]
refractive index of the material of the lens.
22. Find the radius of curvature of the convex 1. A concave lens of refractive index 1.5 is immersed

surface of a plano-convex lens, whose focal in a medium of refractive index 1.65. what is the

length is 0.3 m and the refractive index of nature of the lens?

the material of the lens is 1.5. 2. Why does the bluish colour predominant in a clear
23. Calculate the distance of an object of height h sky?
from a concave mirror of focal length 10 cm, 3. A biconvex lens made of a transparent material of
so as to obtain a real image of magnification refractive index 1.25 is immersed in water of
2. refractive index 1.33. Will the lens behave as a
Or conversing or diverging lens?
Calculate the distance of an object of height h 4. A convex lens is placed in contact with a plane
from a concave mirror of radius of curvature mirror. A point object at a distance of 20cm on the
20 cm, so as to obtain a real image of axis of this combination has its image coinciding
magnification 2. Find the location of image with itself. What is the focal length of the lens.
also. [Delhi 2016] 5. Write the relationship between angle of incident i,
24. A Screen is placed 80 cm from an object. The angle of prism A and angle of minimum deviation
image of the object on the screen is formed 𝛿𝑚 for a triangular prism.
by a convex lens at two different locations,
6. How does the focal length of a lens change? Give
separated by 10 cm. Calculate the focal
the reason for your answer.
length of the lens used.
7. Under what condition does a biconvex lens of glass
25. Two thin lenses of power +6 D and -2 D are
having a certain refractive index act as a plane
in contact. What is the focal length of the
glass sheet, when immersed in a liquid?
combination?
26. A convex lens of focal length 25 cm is placed
Short answer type questions (2 Marks)
co-axially in contact with a concave lens of
1. You are given two conversing lenses of focal length
focal length 20 cm. Determine the power of
1.25 cm and 5cm to design a compound
the combination. Will the system be
microscope. If it desired to have a magnification of
converging or diverging in nature? [Delhi
30, then find out the separation between the
2013]
objective and eyepiece.
27. A beam of light converges at a point P. A
2. A small telescope has an objective lens of focal
concave lens of focal length 16 cm is placed
length 150 cm and eyepiece of focal length 5cm.
in the path of this beam 12 cm is placed in
what is the magnification power of the telescope for
the path of this beam 12 cm from P. Draw a

Advent Tutorial Front of Nehru Inter College Banarsidas, Auraiya


KP Singh 7248370933 ks5122213@gmail.com Page 16
Chapter- 9 Ray Optics
viewing distant objects in normal adjustments. If from a distant object are received at the eyepiece.

this telescope is used to view a 100 m tall tower 3 Write its two important advantages over a

km away, then what is the height of the tower by refracting telescope.

the objective lens. 10. Trace the path of a ray of light passing through a

3. Use the mirror equation to that an object placed glass prism (ABC) as shown in figure. If the

between f and 2f of a concave mirror produces a refractive index of the glass is √3. Find out the value

real image beyond 2f. of the angle of emergence from the prism.

4. Two monochromatic light are incident on the face


A
AB of an isosceles right angled prism ABC. The

refractive index of glass prism of the two rays 1

and 2 are 1.35 and 1.45, respectively. Trace the 60°


B C
path of three rays after entering through the

prism. 11. An object AB is kept in front of a concave mirror


A
as shown in figure.
1
i. Complete the ray diagram showing the image
2 formation of the object.

B C ii. How will the position and intensity of the image be

affect, if the lower half of the mirror’s reflecting

5. A convex lens of focal length 25cm placed co- surface is painted black?

axially in contact with the concave lens of focal

length 20cm. determines the power of the

combination. Will the system be conversing or


C F
diverging in nature?

6. A convex lens of focal length f1 is kept in contact

with a concave lens of focal length f2. Find the focal Short answer type questions (3 Marks)

length of the combination. 1. A giant reflecting telescope has an objective lens of

7. When monochromatic light travels from a rare to a focal length 15m. if an eyepiece of focal length 1cm

dancer medium, explain the following, giving the is used , what is the3 angular magnification of the

reasons. telescope?

i. Is the frequency of reflected and refracted light ii. If the telescope is used to view the moon, what is

same as the frequency of incident light? the diameter of the image of the moon formed by

ii. Does the decrease in speed imply a reduction in the the objective lens? The diameter of the moon is

energy carried by the light wave? 3.48×106m and the radius of lunar orbit is

8. (i) Write the necessary conditions for the 3.8×108m.

phenomenon of total internal reflection to occur. 2. A convex lens of focal length 30cm is placed co-

(ii) Write the relation between the refractive index axially with a convex mirror of radius of curvature

and critical angle for a given pair of optical 20cm. the two are kept at 15cm from each other.

media. A point object lies 60cm in front of the convex lens.

9. Draw a schematic arrangement of reflecting type Draw a ray diagram to show the formation of the

telescope (Cassegrain) showing how rays coming

Advent Tutorial Front of Nehru Inter College Banarsidas, Auraiya


KP Singh 7248370933 ks5122213@gmail.com Page 17
Chapter- 9 Ray Optics
image by the combination. Determine the nature 8. Draw a ray diagram showing the image formation

and position of the image formed. by a compound microscope. hence ,obtained

3. A convex lens of focal length 20cm is placed co- expression for total magnification, when the image

axially with a concave mirror of focal length 10cm is formed at infinity.

at a distance of 50 cm apart from each other. A 9. Define power of a lens. Write its SI unit. Deduce the

beam of light coming parallel to the principle axis is relation for two thin lenses kept in contact
1 1 1
= +
𝑓 𝑓1 𝑓2

incident on the convex lens. Find the position of the coaxially.


final image formed by this combination. Draw the 10. You are given three lenses L1, L2 and L3 each of
ray diagram showing the formation of the image. focal length 10cm. an object is kept at 15cm in
4. A convex lens of focal length 20cm placed co- front of L1 as shown in the figure. The final real
axially with a convex mirror of radius of curvature image is formed at the focus I of L3 find the
20cm. the two are kept 15cm apart. A point separation between L1, L2 and L3.
objects placed 40cm in front of convex lens. Find

the position of the image formed by this


L1 L2 L3
combination. Draw the ray diagram showing the I
image formation.
15cm 10cm
5. i. Draw the ray diagram showing formation of final

image by a compound microscope at least distance

of distinct vision. Long answer type questions (5 Marks)


ii. The magnification produced by a compound 11. i. A ray PQ of light is incident on the face AB of
microscope is 20. The final image produced by the glass prism ABC (as shown in the figure) and
eyepiece is 5. The microscope is focused on a certain emerges out of the face AC. Trace the path of the
object. The distance between the objective and the ray. Show that ∠𝑖 + ∠𝑒 = ∠𝐴 + ∠𝛿 where, 𝛿 and e
eyepiece is observed to be 14cm. if the least denote the angle of deviation and angle of
distance of distinct vision is 20cm, calculate the emergence respectively. Plot a graph showing the
focal length of the objective and the eyepiece. variation of the angle of deviation as a function of
6. i. A mobile phone lies along the principle axis of a angle of incidence. State the condition under which
concave mirror. Show with the help of a suitable ∠𝛿 is minimum.
diagram, the formation of its image. Explain why ii. Find out the relation between the refractive index
magnification is not uniform. (µ) of the glass prism and ∠𝐴 for the case, when
ii. Suppose the lower half of the concave mirror’s the angle of prism (A) is equal to the angle of
reflecting surface is covered with an opaque minimum deviation (𝛿𝑚 ). Hence, obtain the value of
material. What effect this will have on the image of the refractive index for angle of prism A=60°
the object explain.

7. Draw the labelled ray diagram of a reflecting type


A
telescope. Define its magnification power and write i
Q
the expression for it. Write two important P
limitations of a refracting type telescope over a
B C
reflecting type telescope.

Advent Tutorial Front of Nehru Inter College Banarsidas, Auraiya


KP Singh 7248370933 ks5122213@gmail.com Page 18
Chapter- 9 Ray Optics
12. A point object O is kept in a medium of refractive 17. Light from a point source in air falls on a
index µ1 in front of a convex spherical surface of spherical glass surface (𝜇 = 1.5 and radius of
radius of curvature R which separates the second
curvature = 20 cm). The distance of the light
medium of refractive index µ2 from the first one, as
source from the glass surface 100 cm. At
shown in the figure. Draw the ray diagram showing
what position the image is formed [NCERT
the image formation and deduce the relationship
Example]
between the object distance and the image distance

in terms of µ1 , µ2 𝑎𝑛𝑑 𝑅. 18. An object is placed at (i) 10 cm, (ii) 5 cm in

front of a concave mirror of radius of

O µ1 µ2 curvature 15 cm. Find the position, nature,


C and magnification of the image in each case.
u R
19. (i) If f = 0.5 m for a glass lens, what is the
power of the lens?
ii. When the image formed above acts as a virtual
(ii) The radii of curvature of the faces of a
object for a concave spherical surface separating the
double convex lens are 10 cm and 15 cm. Its
medium µ2 from µ1 (µ2 > µ1 ) draw this ray diagram focal length is 12 cm. What is the refractive
and write the similar to relation i. hence obtain the index of glass?
expression for the lens maker formula. (iii) A convex lens has 20 cm focal length in air.
13. Define the magnifying power of a telescope. Write What is focal length in water?
its expression. A small telescope has an objective (Refractive index of air-water = 1.33,
lens of focal length 150cm and eyepiece of focal refractive index for air-glass = 1.5). [NCERT
length 5cm. if this telescope is used to view a 100m Example]
high tower 3km away. Find the height of the final 20. A convex lens of focal length 20 cm is placed
image when it is formed 25cm away from the coaxially with a concave mirror of focal length
eyepiece. 10 cm at a distance of 50 cm apart from each
14. How is the working of a telescope different from other. A beam of light coming parallel to the
that of a microscope? The focal length of the principal axis is incident on the convex lens.
objective and eyepiece of microscope are 1.25cm Find the position of the final image formed by
and 5cm, respectively. Find the position of the this combination. Draw the ray diagram
object relative to the objective in order to obtain an showing the formation of the image. [Similar
angular magnification of 30 in normal adjustment. Pariksha 2019, Al 2014]

15. A biconvex lens has a focal length 2/3 times 21. A convex lens of focal length 20 cm is placed
co-axially with a convex mirror of radius of
the radius of curvature of either surface.
curvature 20 cm. The two are kept at 15 cm
Calculate the refractive index of lens material.
from each other. A point object lies 60 cm in
16. A convex lens of refractive index 1.5 has a
front of the convex lens. Draw a ray diagram
focal length of 18 cm in air. Calculate the to show the formation of the image by the
change in its focal length when it is immersed combination. Determine the nature and
in water of refractive index, 4/3. position of the image formed. [Similar
Panchkula 2019, Al 2014]

Advent Tutorial Front of Nehru Inter College Banarsidas, Auraiya


KP Singh 7248370933 ks5122213@gmail.com Page 19
Chapter- 9 Ray Optics
22. The total magnification produced by a emergence is 4 times the angle of prism,
compound microscope is 20. The magnification evaluate the refractive index of the glass prism
produced by eyepiece is 5. The microscope is 28. In the following diagram, find the local length
focussed on a certain Object. The distance of lens L2. Trace the complete diagram for
between the objective and eyepiece is observed image formation. L2

to be 14 cm. If least distance of distance vision


𝐴
L1
is 20 cm, calculate the focal length of the
I
object and the eyepiece. [Delhi 2014]
23. The figure shows a ray of light falling normally 15 cm
on face AB of an equilateral glass prism having 20 cm 80 cm
refractive index 3/2 placed in water of
refractive index 4/3. Will this ray suffer total 29. A convex lens, of focal length 20 cm, has a
internal reflection on striking the face AC? point object placed on its principle axis at
Justify your answer. 𝐴 distance of 40 cm from it. A plane mirror is
placed 30 cm behind the convex lens. Locate
the position of image formed by this
combination. [CBSE Supplementary Textual
𝜇𝑔
Material]
30. A convex lens is placed in contact with a plane
𝐵 𝐶 mirror. An axial point object, at a distance of
24. A figure divided into squares each of size 1 20 cm from this combination, has its image
mm2 is being viewed at a distance of 8 cm coinciding with itself. What is the focal length
through a converging lens of focal length 12 of the convex lens? [CBSE Supplementary
cm. Textual Material]
(i) What is the magnification produced by the 31. A convex lens, and a convex mirror, (of radius
lens? of curvature 20 cm) are placed co-axially with
(ii) How much is the area of each square in the the convex mirror placed at a distance of 30
virtual image? cm from the lens. For a point object at a
25. A ray of light passing through an equilateral distance of 20 cm from the lens, the final
triangular glass from air undergoes minimum image; due to this combination, coincides with
deviation when angle of incidence is 3/4th of the object itself. What is the focal length of the
the angle of prism. Calculate the speed of light convex lens?
in the Prism. [AI 2017] 32. A thin prism (µg = 1.5) when placed in air deviates a

26. Determine the value of the angle of incidence ray by a minimum angle of 5°. What will be the
angle of minimum deviation if it is kept immersed in
or a ray of light travelling from a medium of
oil of refractive index µ0 = 1.25?
refractive index µ1= √2 into the medium of
33. One face of a prism with a refracting angle of 30° is
refractive index µ2 = 1, so that it just grazes
coated with silver. A ray incident on another face at
along the surface separation. [Foreign 2017] an angle of 45° is refracted and reflected from the
27. A ray of light passes through an equilateral silver coated face and retraces its path. Find the
glass prism, such that the angle of incidence is refractive index of the material of the prism.
equal to the angle of emergence, If the angle of

Advent Tutorial Front of Nehru Inter College Banarsidas, Auraiya


KP Singh 7248370933 ks5122213@gmail.com Page 20
10 Wave Optics

Chapter minimum intensity of light remains fixed on the

Wave optics- The study of light considering it as the screen. Interference effect in light is not easy to

wave is called wave optics. observe because of the short wavelength involved. In

order to observe the interference, the following

Wave front- A wave front is defined as the locus of conditions must satisfied.

all the points at which the phase of vibration of 1. The two sources should emit the waves

physical quantity associated with the wave is the continuously.

same. 2. The two waves should be of same wavelength.

3. The two waves should be preferable have the

Huygens principle- It is based upon the following same amplitude.

two assumptions- 4. The waves emitted by the two sources must

1. all the points on a given wave front are have the same phase or constant phase

taken as point sources for the production of difference.

spherical secondary waves, called wavelets 5. The two sources must lie very close to each

which propagate outward with speed other.

characteristic of wave in that medium. 6. The two sources must be very narrow.

2. After some time has elapsed, the new

position of the wave front is the surface Diffraction- The divergence of light from its initial

tangent to the envelope of the wavelets in line of travel is called diffraction of light.

the forward direction.

Polarisation of light- The restriction of all the

Interference of light- Interference is the vibration of light waves leaving only a particular

phenomenon of non-uniform distribution of energy vibration is called polarisation and light so found is

in the medium due to superimposition of two light called polarised light. And the crystal (Nicole prism)

waves. is called polariser.

Sustained interference- Interference is said to be Malus law- It state that, when a beam of

sustained if the position of the maximum and completely polarised light is incident on the

analyser, the result in the intensity of light (I) varies


Chapter- 10 Wave optics
directly as the square of the cosine of angle between 5. If two sources have a randomly varying

the plane of transmission of analyser and polariser phase difference Ф(t), the resultant intensity
will be given by
mathematically we have I= I0 cos2θ I0 is the
(a) 𝐼𝑜 √2 (b) (c) 2𝐼0 (d)
𝐼0 𝐼0
intensity of light incident on analyser. 2 √2
Ans. (c)
6. According to Huygens' principle, light is a
Brewster’s law- When reflected and refracted ray
form of
are perpendicular to the, then tangent of incident (a) particle (b) rays
angle is equal to the refractive index of the medium (c) wave (d) radiation

and incident angle is called the polarising angle or Ans. (c)


7. Two coherent monochromatic light beams of
Brewster angle mathematically µ = tan 𝜃𝑝
intensities I and 4I superimpose. The
maximum and minimum possible intensities
Very Short Answer/objective Type Questions [1
in the resulting beam are:
Mark]
(a) 5I and I (b) 5I and 3I
1. Resolving power of telescope can be
(c) 3I and I (d) 9I and I
increased by increasing
Ans. (d)
(a) the wavelength
8. What is path difference for destructive
(b) the diameter of objective
interference?
(c) the diameter of eyepiece
(a) 𝑛𝜆 (b) 𝑛(𝜆 + 1)
(d) the focal length of eyepiece
(c) (2𝑛 + 1) (d) (𝑛 + 1)
𝜆 𝜆
Ans. (b) 2 2
Ans. (c)
2. Polarisation of light proves
9. When exposed to sunlight, thin films of oil
(a) corpuscular nature of light.
on water of ten exhibit brilliant colours due
(b) quantum nature of light.
to the phenomenon of
(c) transverse wave nature of light.
(a) interference (b) diffraction
(d) longitudinal wave nature of light.
(c) dispersion (d) polarisation
Ans. (c)
Ans. (a)
3. The wave-front due to a source situated at
10. What happens, if the monochromatic light
infinity is.
used in Young's double slit experiment is
(a) spherical (b) cylindrical
replaced by white light?
(c) planar (d) circular
(a) No fringes are observed.
Ans. (c)
(b) All bright fringes become while.
4. A laser beam is coherent because it contains
(c) All bright fringes have colour between
(a) waves of several wavelengths.
violet and red.
(b) incoherent waves of a single wavelength.
(d) Only the central fringe is white and all
(c) coherent waves of several wavelengths
other fringes are coloured.
(d) coherent waves of a single wavelength
Ans. (d)
Ans. (d)
11. When compact disk is illuminated by a
source of white light, coloured lines are
observed. This is due to

Advent Tutorial Front of Nehru Inter College Banarsidas, Auraiya


KP Singh 7248370933 ks5122213@gmail.com Page 2
Chapter- 10 Wave optics
(a) dispersion (b) diffraction 17. In Young's double slit experiment, if the
(c)Interference (d) refraction mono-chromatic source of yellow light is
Ans. (b) replaced by red light, the fringe width
12. An un-polarised beam of intensity I is (a) increases (b) decreases
incident on a pair of nicols making angle of (c) remains unchanged
60° with each other. The intensity of right (d) the fringes disappear Ans. (a)
emerging from the pair is 18. In Young's double-slit experiment, the
(a) 𝐼0 (b) (c) (d) intensity at the central maximum is 𝐼0 , if
𝐼0 𝐼0 𝐼0
2 4 8
Ans. (d) one of the slit is covered, then the intensity
13. When un-polarised light beam is incident at the central maximum become
from air onto glass (n = 1.5) at the (a) (b) (c) (d) 𝐼0
𝐼𝑜 𝐼𝑜 𝐼𝑜
2 √2 4
polarising angle. Ans. (c)
(a) Reflected beam is polarised completely 19. The angle of incidence at which reflected
(b) Reflected and refracted beams are light is totally polarised for reflection from
partially polarised air to glass (refractive index n) is
(c) Refracted beam is plane polarised (a) sin−1 (𝑛) (b) sin−1 ( )
1
𝑛
(d) Whole beam of light is refracted (c) tan−1 ( )
1
(d) tan−1 (𝑛)
𝑛
Ans. (b) Ans. (d)
14. Resolving power of microscope depends 20. In Young's double-slit experiment, the
upon intensity is I at a point, where the path
(a) wavelength of light used (directly difference is
𝜆
(𝜆-wavelength of light used).
6
proportional) If 𝐼0 , denotes the maximum intensity then
(b) wavelength of light used (inversely 𝐼
is equal to
𝐼0
proportional) (a)
√3
(b)
1
(c)
3
(d)
1
2 2 4 √2
(c) frequency of light used
Ans. (c)
(d) focal length of objective
21. Consider a light beam incident from air to a
Ans. (b)
glass slab at Brewster's angle as shown in
15. The phenomenon of interference is based on
Figure A polaroid is placed in the path of
(a) conservation of momentum.
the emergent ray at point P and rotated
(b) conservation of energy.
about an axis passing through the centre
(c) conservation of momentum and energy.
and perpendicular to the plane of the
(d) quantum nature of light.
polaroid.
Ans. (b)
16. A double slit interference experiment is
carried out in air and the entire
arrangement is dipped in water. The fringe
width
(a) increases (b) decreases
(c) remains unchanged.
(d) fringe pattern disappears. Ans. (b)

Advent Tutorial Front of Nehru Inter College Banarsidas, Auraiya


KP Singh 7248370933 ks5122213@gmail.com Page 3
Chapter- 10 Wave optics
(a) For a particular orientation there shall arrangement with slits S3, S4 and a second
be darkness as observed through the screen behind them. [NCERT Exemplar]
polaroid. Screen
(b) The intensity of light as seen through the S1 P1

polaroid shall be independent of the Screen 2

rotation. S
(c) The intensity of light as seen through the
P
S3
polaroid shall go through a minimum but
S2 P2 S4
not zero for two orientations of the
polaroid.
(d) The intensity of light as seen through the (a) There would be no interference pattern
polaroid shall go through a minimum for on the second screen but it would be lighted.
four orientations of the polaroid. (b) The second screen would be totally dark.
[NCERT Exemplar] Ans. (c) (c) There would be a single bright point on
22. Consider sunlight incident on a slit of width the second screen.
104 Å. The image seen through the slit shall (d) There would be a regular two slit
(a) be a fine sharp slit white in colour at the pattern on the second screen. Ans (d)
centre. 25. Consider the diffraction pattern for a small
(b) a bright slit white at the centre diffusing pinhole. As the size of the hole is increased
to zero intensities at the edges. (a) the size decreases.
(c) a bright slit white at the centre diffusing (b) the intensity decreases.
to regions of different colours. (c) the size increases.
(d) only be a diffused slit white in colour. (d) the intensity decreases.
[NCERT Exemplar] Ans. (a) [NCERT Exemplar] Ans (a)
23. In a Young's double slit experiment, the
source white light. One of the holes is 26. For light diverging from a point source
covered by a red filter and another by a (a) the wave-front is spherical.
blue filter. In this case (b) the intensity increases in proportion to
(a) there shall be alternate interference the distance squared.
patterns of red and blue. (c) the wave-front is parabolic
(b) there shall be an interference pattern for (d) the wave-front is parabolic the intensity
red distinct from that for blue at the wave-front does not depend on the
(c) there shall be no interference fringes distance. Ans (a)
(d) there shall be an interference pattern for 27. Continuous locus of particles of medium
red mixing with one for blue. Ans (c) vibrating in the same phase at any instant
[NCERT Exemplar] is known as _______________ .
24. Figure Shows a standard two slit Ans. wave-front
arrangement with slits S1, S2. P1, P2 are 28. Superposition of two waves in the same
the two minima points on either side of P phase to produce maximum intensity is
(Figure). At P2 on the screen, there is a hole known as ____________ interference.
and behind P2, is a second 2- slit Ans. constructive

Advent Tutorial Front of Nehru Inter College Banarsidas, Auraiya


KP Singh 7248370933 ks5122213@gmail.com Page 4
Chapter- 10 Wave optics
29. Phenomenon of bending of light around Ans. Plane wave-front
S Plane wave
corners of a small obstacle and spreading Spherical wave-front front
F
into region of geometrical shadow is called Spherical wave
front
______________ of light. 38. Draw the wave-front, if a point source is
Ans. diffraction placed at point 2F of a convex lens.
30. Resolving power of an optical instrument is Ans.
_______________ to limit resolution.. 2F1 2F2
Ans. reciprocal
31. Plane of polarisation is _____________ to plane
of vibration. 39. When monochromatic light travels from one
Ans. perpendicular medium to another its wavelength changes
32. For a destructive interference, phase but frequency remains the same. Explain.
difference between wave is ______________ . [NCERT Example, Delhi 2011]
Ans. (2n-1)π Ans. When a monochromatic light is incident
33. Path difference between two waves, having on the surface separating two media, the
wavelength 𝜆 and undergoing constructive atoms of the media interact with the light
interference is _____________ . and may be viewed as oscillators, which take
Ans. 𝒏𝝀 up the frequency of light causing forced
34. What is a phase difference between two oscillations. Thus, the frequency remains
points on the same wave-front? same. But in medium, the speed changes,
Ans. Zero. and accordingly the wavelength changes.
35. Sketch the shape of wave-front emerging 40. When light travels from a rarer to a denser
from a point source of light and also mark medium, the speed decreases. Does the
the rays. reduction in speed imply a reduction in the
Ans. 1, 2, 3,…….. Rays energy carried by the light wave?
Spheres → Wave-fronts Ans. No, as energy ∝ (amplitude)2 and is
S → Source independent of speed of wave.

1
41. What is meant by interference of light?
Ans. The phenomenon of non-uniform
S distribution of energy in a medium due to
2
the superposition of two light waves from
two coherent sources is called interference of
3
light.
36. What type of wave-front will emerge from 42. The interference fringe pattern according to
a (i) point source, and (ii) distant light the theory is hyperbola. What is the
source? condition for seeing nearly straight fringes?
Ans. (i) Point source Spherical wave-front. Ans. If distance between the plane of the
(ii) Distant light source Plane wave-front. two coherent sources and screen is very
37. Draw the wave-front coming out of a large compared to the fringe width; the
convex lens when a point source of light is fringes will be nearly straight lines.
placed at its focus.

Advent Tutorial Front of Nehru Inter College Banarsidas, Auraiya


KP Singh 7248370933 ks5122213@gmail.com Page 5
Chapter- 10 Wave optics
43. Why are coherent sources required to create distance of separation between the slits and
interference of light? screen is doubled? [Al 2012]
Ans. Without coherent sources, the intensity Ans. 𝛽 = , when the distance between the
𝜆𝐷
𝑑
of the interference pattern will not be slits and the screen is doubled. The fringe
sustained. width will also get doubled.
44. State the reason, why two independent 50. When a monochromatic source is replaced
sources of light cannot be considered as by a source of white light in Young's double-
coherent sources. slit experiment, then which colour will
Ans. They cannot keep a constant phase appear closest to the central white fringe.
difference. Ans. The fringe closest on either side of the
45. In the wave picture of light, intensity of central white fringe is red.
light is determined by the square of the Explaination: For a point P for which
amplitude of the wave. What determines the S1P-S2P =
𝜆𝑏
2
intensity of light in the photon picture of where 𝜆b represents wavelength for blud
light. colour, the blue component will be absent
Ans. In photon picture, the intensity is and red colour fringe will appear. [𝜆b < 𝜆R]
determined as [NCERT Example] 51. What is the effect on the interference
Intensity = fringes in a Young's double slit experiment,
𝑁𝑢𝑚𝑏𝑒𝑟 𝑜𝑓 𝑝ℎ𝑜𝑡𝑜𝑛𝑠
𝐴𝑟𝑒𝑎×𝑇𝑖𝑚𝑒
46. What is the effect on the interference if the width of the source slit is increased?
fringes in a Young's double slit experiment if [NCERT Example]
the screen is moved away from the plane of Ans. As the source slit width increases, the
the slits? fringe pattern gets less and less sharp. When
Ans. The angular separation of the fringes the source slit is so wide that the condition
remains cons (= ). The actual linear s/S ≤ 𝜆/d is not satisfied, the interference
𝜆
𝑑
separation of the fringes increases in pattern disappears.
proportion to the distance of the screen 52. One of the two slits in Young double slit
from the plane of the two slits. experiment is so painted that it transmit
47. What is the ratio of the fringe width for half the intensity of the other. What is the
bright and dark fringes in Young's double effect on interference fringes? [HOT]
slit experiment? [HOTS] Ans. The contrast between the bright and
Ans. 1: 1 dark fringes decreases.
48. How many interference fringes will be seen 53. Why does the intensity of the secondary
if the size of each slit is ( )th the separation maximum become less as compare to the
1
5
between the two slits? central maximum?
Ans. a = Ans. As the order increases only ( where
𝑑 1 th
5 𝑛
2𝜆𝐷 𝑛𝜆𝐷
= n is an odd number) of the slit will
𝑎 𝑑
2×5 𝑛 contribute in producing brightness at a
= ⇒ 𝑛 = 10
𝑑 𝑑 point in the diffraction. So, the higher order
49. How does the fringe width, in Young's
maxima are not so bright as the central.
double slit experiment, change when the
54. How does the angular separation of
interference fringes change, in Young's

Advent Tutorial Front of Nehru Inter College Banarsidas, Auraiya


KP Singh 7248370933 ks5122213@gmail.com Page 6
Chapter- 10 Wave optics
experiment, if the distance between the slits Short Answer Type Questions [2 marks]
is increased? 1. Find the ratio of intensities of two points P
Ans. Decreases, as 𝜃 = and Q on a screen in Young's double slit
𝜆
𝑑
55. What is the condition that only interference experiment when from sources S1 and S2
is observed in double slit experiment, have phase difference of (i) 0°, and (ii)
diffraction is not? respectively.
𝜋
2
Ans. When << 1 then the diffraction Ans. (i) When rays are out of phase by 0°,
𝑎
𝑑
pattern will become very flat and we we get constructive interference and hence
observe only the interference pattern. we will obtain maximum intensity.
56. How are interference and diffraction
Ф
𝐼1 = 4𝐼0 cos2 = 4𝐼0 cos2 0° ⇒ 𝐼1 = 4𝐼0
2
consistent with the principle of conservation (ii) Ф =
𝜋 𝜋 4𝐼
, 𝐼2 = 4𝐼𝑜 cos2 = 0 ⇒ 𝐼1 = 2𝐼0
2 4 2
of energy? 𝐼1 : 𝐼2 = 4: 2 = 2: 1
2. Draw the intensity pattern for single slit
Ans. In interference and diffraction, the
diffraction and double slit interference.
light energy is redistributed. If it reduces in
Hence, state two differences between
one region, producing a dark fringe, it
interference and diffraction patterns.
increases in another region, producing a
[Delhi 2019, AI 2017]
bright fringe. There is no gain or loss of
Ans. Intensity pattern for:
energy, and the two phenomena are
𝐼
(a) Single slit diffraction
consistent with the principle of conservation
of energy
57. Why does the bluish colour predominate in a
clear sky?
2𝜆 3𝜆
Ans. Our eyes are more sensitive to blue
3𝜆 𝜆 0 𝜆
− − − 2𝜆
𝑑 𝑑 𝑑 𝑑 𝑑
𝑑

colour.
𝐴𝑛𝑔𝑙𝑒 𝜃

58. An un-polarised light of intensity I is passed


(b) Double slit interference
through a polaroid. What is the intensity of
4I
the light transmitted by the polaroid?
Ans as it will get polarised.
𝐼
2
59. If the angle between the axis of polariser
and the analyser is 45°, write the ratio of
the intensities of original light and the −
5𝜆𝐷 3𝜆𝐷 𝜆𝐷 𝜆𝐷 3𝜆𝐷 5𝜆𝐷
2𝑑 − − 2𝑑
transmitted light after passing through the
2𝑑 2𝑑 2𝑑 2𝑑

Differences between interference and


analyser.
diffraction patterns:
Ans As 𝐼 = cos2 𝜃, where 𝐼0 is the original
𝐼0

(i) Interference is the superposition of light


2
intensity and 𝜃 the angle between the axis
waves from two different wave-fronts
of the polariser and the analyser.
cos2 𝜃 originating from the same source, while the
(∵ θ = 45°)
𝐼 1
= =
diffraction is the interaction of light waves
𝐼0 2 4
𝐼0 : 𝐼 = 4: 1
from different parts of the same wave-front
(ii) In an interference pattern, fringes may
or may not be of the same width, while in

Advent Tutorial Front of Nehru Inter College Banarsidas, Auraiya


KP Singh 7248370933 ks5122213@gmail.com Page 7
Chapter- 10 Wave optics
diffraction pattern, they are never of the 𝐼′𝑎𝑣 = 𝐼1 + 𝐼2 _______________ (ii)
same width. From (i) and (ii), we get =a
(iii) In an interference pattern, bright Therefore, energy is neither be created nor
fringes are of uniform intensity, while in be destroyed, but it is transferred from the
diffractions pattern, they are of varying sight of minima to the sight of maxima.
intensity.
(any two) 6. What is the effect on the interference
fringes if the monochromatic source is
3. For a single slit of width a, the first replaced by a source of white light?
minimum of the interference pattern of a Ans. When the monochromatic light is
monochromatic light of wavelength occurs replaced by a white light, (i) the central
at an angle of . At the same angle of , we bright remains white, and (ii) all the other
𝜆 𝜆
𝑎 𝑎
get a maximum for two narrow slits a colours will form individual maxima with
separated by a distance a. Explain. The least wavelength violet forming its
[Delhi 2014] bright close the central bright.
Ans. In case of a single slit of width a, the
wavelets from two halves of slit produce a 7. How will the interference pattern in Young's
minimum as the corresponding wavelets double slit experiment get affected, when
have a path difference of In the second case, (distance between the slits S1 and S2, is
the overlapping of the wave-fronts from the reduced, and (i) the entire set-up is
two slits produce first maximum because immersed in water? Justify your answer in
they meet with a path difference of . each case. [Delhi 2011C]
Ans. Fringe width β of the dark or bright
4. Find the intensity at a point on a screen in fringes is given by 𝛽 = 𝜆
𝐷
𝑑
Young's double slit experiment where the where d is the distance between the slits.
interfering waves of equal intensity have a (i) When the distance between the slits S1
path difference of and S2 (i.e. d) is reduced, β will increase. The
(i) 𝜆/4, and (ii) . [Foreign 2017] interference pattern will become broader.
𝜆
3
5. Is the phenomenon of interference of light in (ii) The wavelength in water will 𝜆’ = 𝜆/𝑑.
accordance with the law of conservation of 𝜆’ < 𝜆 𝛽’ = i.e. fringe width decreases.
𝛽
µ
energy? Justify.
Ans. Yes. If I1, and I2, be the intensities of 8. (a) In what way is diffraction from each slit
two interfering waves, then before related to the interference pattern in a
interference, average intensity is double slit experiment?
Iav = 𝐼1 + 𝐼2 ___________________(i) (b) Two wavelengths of sodium light 590
After, the waves interfere, the maxima and nm and 596 nm are used, in turn, to study
minima will be formed and the diffraction taking place at a single slit of
𝐼𝑀𝑎𝑥 + 𝐼𝑀𝑖𝑛
𝐼′𝑎𝑣 = aperture 2x10-4 m. The distance between
2
2
Here 𝐼𝑀𝑎𝑥 = (√𝐼1 + √𝐼2 ) , 𝐼𝑀𝑖𝑛 = (√𝐼1 − √𝐼2 )
2 the slit and the screen is 1.5 m. Calculate
2
(√𝐼1 + √𝐼2 ) + (√𝐼1 − √𝐼2 )
2 the separation between the positions of the
𝐼′𝑎𝑣 =
2

Advent Tutorial Front of Nehru Inter College Banarsidas, Auraiya


KP Singh 7248370933 ks5122213@gmail.com Page 8
Chapter- 10 Wave optics
first maxima of the diffraction pattern separation between two distant objects
obtained in the two cases. [Delhi 2013] which the telescope can make out.
Resolving Power =
1 𝐷
=
𝜃𝑚𝑖𝑛 1.22𝜆
9. Name the phenomenon which is responsible (i) On increasing the aperture of the
for bending of light round sharp corners of objective lens, the resolving power increases
an obstacle. Under what conditions does this because resolving power ∝ D.
phenomenon take place? Give one (ii) On increasing the focal length of an
application of this phenomenon in everyday objective lens, the resolving power does not
life. [CBSE Sample Paper 2015] get affected.
Ans. Diffraction. Condition for diffraction:
The size of an obstacle or aperture should be 12. Define resolving power of a compound
comparable to the wavelength of the light microscope. How does the resolving power of
falling a compound microscope change when
Diffraction is used in spectrometer, X-ray (i) refractive index of the medium between
crystallography, holography, etc. the object and objective lens increases?
“In the double-slit experiment, the pattern (ii) wavelength of the radiation used is
on the screen is due to superposition of increased?
single-slit diffraction from each slit.” Justify Ans. Resolving power of a microscope is the
this statement. reciprocal of the minimum separation of
two points seen as distinct.
10. Sketch of a graph showing the variation of Resolving power =
1
=
2𝜇 sin 𝛽
𝑑𝑚𝑖𝑛 1.22𝜆
fringe width versus the distance of the (i) As resolving power ∝ 𝜇, on increasing the
screen from the plane of the slits (keeping refractive index of the medium between the
other parameters same) in Young's double object and objective lens, resolving power
slit experiment. What information increases.
Can one obtain from the slope of this graph? (ii) As resolving power ∝ , On increasing the
1
𝜆
Ans. 𝛽 =
𝜆𝐷
𝑑
⇒ 𝛽 ∝𝐷 β wavelength of the radiation resolving power
𝛽 𝜆
𝐷
=
𝑑 Decreases.
slope of graph =
𝜆
𝑑
Knowing the value of d and 13. At what angle of incidence should a light
the slope of the graph, the beam strike a glass slab of refractive index
D
Wavelength of light used can be calculated. √3, such that the reflected and the
refracted rays are perpendicular to each
11. Define the term resolving power of a other?
telescope. How does it get affected on Ans. 𝜇 = tan𝑖𝑝 , under the given condition.
(i) Increasing the aperture of the objective Therefore, 𝑖𝑝 = tan−1 (√3) = radian.
𝜋
3
lens?
(ii) Increasing the focal length of the 14. What is the intensity of transmitted light
objective lens? when a polaroid sheet is rotated between
Ans. The resolving power of a telescope is two crossed polaroids?
the reciprocal of the smallest angular (𝜃𝑚𝑖𝑛 )

Advent Tutorial Front of Nehru Inter College Banarsidas, Auraiya


KP Singh 7248370933 ks5122213@gmail.com Page 9
Chapter- 10 Wave optics
Ans. Let 𝐼0 , be the intensity of polarised plane surface making the angle of incidence
light after passing through the first polariser i as shown in the figure. It enters into a
P1. Then the intensity of light after passing medium of refractive 𝜇2 (𝜇2 > 𝜇1 ). Use
through second polariser P2, will 𝐼 = Huygens' construction of secondary wavelets
𝐼0 cos2 𝜃, i.e. intensity will vary according to to trace the propagation of the refracted
𝐼 ∝ cos2 𝜃. wave-front. Hence verify Snell's law of
refraction. [Delhi 2019, Foreign 2015]
15. (a) Using the phenomenon of polarisation, 2. (a) Using Huygen's construction of secondary
show how transverse nature of light can be wavelets explain how a diffraction pattern is
demonstrated. obtained on a screen due to a narrow slit on
(b) Two polaroids P1 and P2, are placed which a monochromatic beam of light is
with their pass axes perpendicular to each incident normally.
other. Un-polarised light of intensity 𝐼0 is (b) Show that the angular width of the first
incident on P1. A third polaroid P3, is kept diffraction fringe is half that of the central
in between P1 and P2 such that its pass axis fringe.
makes an angle of 30° with that of P1. (c) Explain why the maxima at
Determine the intensity of light transmitted become weaker and weaker
1 𝜆
𝜃 = (𝑛 + )
2 𝑎
through P1, P2 and P3 (AI 2014) with increasing n. [Delhi 2015]
3. What does a polaroid consist of? Show, using
16. (a) Distinguish between un-polarised and a simple polaroid, that light waves are
linearly polarised light. transverse in nature. Intensity of light
(b) A partially plane polarised beam of light coming out of a polaroid does not change
is passed through a polaroid. Show irrespective of the orientation of the pass-
graphically the variation of the transmitted axis of the polaroid. Explain why. [AI 2015]
light intensity with angle of rotation of the 4. (a) Show, with the help of a diagram, how
polaroid. [Delhi 2019] un-polarised sunlight gets polarised due to
Dehradun 2019, Panchkula 2019] scattering [Dehradun 2019]
17. Un-polarised light is passed through P1. (b) Two polaroid’s P1 and P2, are placed
When this polarised beam passes through with their pass axes perpendicular to each
another polaroid P2 and if the pass-axis of other. Un-polarised light of intensity 𝐼0 , is
P2, makes angle 𝜃 with the pass-axis of P1, incident on P1. A third polaroid P3, is kept
then write the expression for the polarised in between P1 and P2 such that its pass axis
beam passing through P2. Draw a plot makes an angle of 45° with that of P1.
showing the variation of intensity when 𝜃 Determine the intensity of light transmitted
Varies from 0 to 2𝜋. through P1, P2 and P3 [AI 2014]
(Similar Panchkula 2019, Al 2017 5. (a) Light passes through two polaroids P1,
18. How does diffraction limit the resolving and P2, with pass-axis of P2, making an
power of an optical instrument? angle 𝜃 with the pass-axis of P1. For what
Long Answer Type [I] Questions [3 Marks] value of 𝜃 is the intensity of emergent light
1. A plane wave-front propagating in a zero?
medium refractive index 𝜇, is incident on a

Advent Tutorial Front of Nehru Inter College Banarsidas, Auraiya


KP Singh 7248370933 ks5122213@gmail.com Page 10
Chapter- 10 Wave optics
(b) A third polaroid is placed between P1 the expression for the resultant intensity at
and P2 with its pass-axis making an angle 𝛽 the point.
with the pass-axis of P1. Find a value of 𝛽 (b) In Young's double slit experiment, using
for which the intensity of light emerging monochromatic light of wavelength, the
from P2, is , where 𝐼0 , is the intensity of intensity of light at a point on the screen
𝐼0
8
light on the poaroid P1. where path difference is 𝜆, is K units. Find
6. In a double slit interference experiment, the out the intensity of light at a point where
two coherent beams have slightly different path difference is . [Delhi 2014]
𝜆
3
intensities 𝐼 and 𝐼 + 𝛿𝐼 (𝛿𝐼 << 𝐼). Show that 9. (a) In Young's double slit experiment,
the resultant intensity at the maxima is describe briefly how bright and dark fringes
nearly 4𝐼 while that at the maxima is nearly are obtained 0n the screen kept in front of
a double slit. Hence obtain the expression for
|𝛿𝐼|2
.
4𝐼
the fringe width. [Similar Dehradun 2019]
Long Answer Type [II] Questions [5 Marks] (b) The ratio of the intensities at minima to
7. (a) State Huygen's principle. Using this the maxima in the Young' double slit
principle draw a diagram to show how a experiment is 9 : 25. find the ratio of the
plane wave-front incident at the interface widths of the two slits. [AI 2014]
of the two media get refracted when it 10. (a) In Young's double slit experiment, deduce
propagates from a rarer to a denser the conditions for obtaining constructive
medium. Hence verify Snell's law of and destructive interference fringes. Hence
refraction. deduce the expression for the fringe width.
(b) When monochromatic light travels from [Delhi 2019]
a rarer to a denser medium, explain the (b) Show that the fringe pattern on the
following, giving reasons: screen is actually a superposition of single
(i) Is the frequency of reflected and slit diffraction from each slit.
refracted light same as the frequency of (c) What should be the width of each slit to
incident light? obtain 10 maxima of the double slit pattern
(ii) Does the decrease in speed imply a within the central maximum of the single
reduction in the energy carried by light slit pattern, for green light of wavelength
wave? [Delhi 2013] 500 nm, if the separation between two slits
8. (a) (i) Two independent monochromatic is 1 mm? [AI 2015]
sources of light cannot produce a sustained 11. (a) What are coherent sources of light? Two
interference pattern'. Give reason. slits in young's double slit experiment are
[Chennai 2019] illuminated by two different sodium lamps
(ii) Light waves each of amplitude a and emitting light of the same wavelength. Why
frequency ʋ, emanating from two coherent is no interference pattern observed?
light sources superpose at a point. If the (b) Obtain the conditions for getting dark
displacements due to these waves is given by and bright fringes in young's experiment.
𝑦1 = 𝑎 cos 𝜔𝑡 and 𝑦1 = 𝑎 cos(𝜔𝑡 + Ф) what is the Hence write the expression for the fringe
phase difference between the two, obtain width.

Advent Tutorial Front of Nehru Inter College Banarsidas, Auraiya


KP Singh 7248370933 ks5122213@gmail.com Page 11
Chapter- 10 Wave optics
(c) If s is the size of the source and d its when passed through a polaroid gets
distance from the plane of the two slits. polarised?
What should be the criterion for the (b) A beam of un-polarised light is incident
interference fringes to be seen? on a glass-air interface. Show, using a
12. A monochromatic light of wavelength 𝜆 is suitable ray diagram, that light reflected
incident normally on a narrow slit off width from the interface is totally polarised, when
'a' to produce a diffraction pattern on the 𝜇 = tan 𝑖, where 𝜇 is the refractive index of
screen placed at a distance D from the slit. glass with respect to air and iB, is the
With the help of a relevant diagram deduce Brewster's angle. [Delhi 2018, 14]
the conditions for obtaining maxima and 16. State the essential condition for diffraction
minima on the screen. Use these conditions of light to take place.
to show that angular width of central Use Huygen's principle to explain diffraction
maximum is twice the angular width of of light due to a narrow single slit and the
secondary maximum. [Foreign 2017] formation of a pattern of fringes obtained
13. (a) Write three characteristic features to on the screen. Sketch the pattern of fringes
distinguish between the interference fringes formed due to diffraction at a single
in Young double slit experiment and the slit showing variation of intensity with angle
diffraction pattern obtained due to a 𝜃. [Similar Chennai 2019]
narrow single sit 17. (a) Define wave-front. Use Huygen's
(b) A parallel beam of light of Wavelength principle verify the laws of refraction.
500 nm falls on a narrow slit and the [Delhi 2019]
resulting diffraction pattern is observed on a (b) How is linearly polarised light obtained
screen 1 m away, It is observed that the by the process of scattering of light? Find
first minimum is a distance of 2.5 mm the Brewster angle for air-glass interface,
away from the centre. Find the width of the when the refractive index of glass = 1.5
slit. [Foreign 2014] [AI 2017]
14. (a) A monochromatic source of light of
wavelength 𝜆 illuminates a narrow slit of Numerical Problems
width d to produce a diffraction pattern on 1. The ratio of intensity at maxima and
the screen. Obtain the conditions when minima is 25: 16. What will be the ratio of
secondary wavelets originating from the slit the width of the two slits in Young's double
interfere to produce maxima and minima slit experiment? [HOTS]
on the screen. 2. In Young's double slt experiment, using light
(b) How would the diffraction pattern be of wavelength 400 nm, Interference fringes
affected when of width X are obtained. The wavelength of
(i) the width of the slit is decreased? light is Increased to 600 nm and the
(ii) the monochromatic source of light is separation between the slits is halved.
replaced by white light? [Foreign 2013] lf one wants the observed fringe width on
15. (a) How does one demonstrate, using a the screen to be the Name In the two cases,
suitable diagram, that un-polarised light find the ratio of the distance between the

Advent Tutorial Front of Nehru Inter College Banarsidas, Auraiya


KP Singh 7248370933 ks5122213@gmail.com Page 12
Chapter- 10 Wave optics
screen and the plane of the Interfering narrow slit and the resulting diffraction
sources in the two arrangements. pattern is obtained on a screen 1 m away.
3. In a double slit experiment with It is observed that the first minimum is at a
monochromatic light, fringes are obtained distance of 2.5 mm from the centre of the
on a screen placed at some distance from screen. Find
the slit. If the screen is moved by 5x10-2 m (a) the width of the slit. [AI 2013]
towards the slits, the change in fringe width (b) the distance of the second maximum
is 3x10-5 m. If the distance between slits is from the centre of the screen.
10-3 m, calculate the Wavelength of light (c) the width of the central maximum.
used. 9. In a Young's experiment the width of the
4. Laser light of Wavelength 630 nm incident fringes obtained with light of wavelength
on pair of slits produces an interference 6000 Å is 2.0 mm, What will be the fringe
pattern in which the bright fringes are width if the entire apparatus is immersed in
separated by 7.2 mm. Calculate the a liquid of refractive index 1.33? HOTS
wavelength of another sources of laser light 10. In Young's double slit experiment, two slits
which produces Interference fringes are separated by 3 mm distance and
separated by 8.I mm using same pair of slits illuminated by light of wavelength 480 nm.
5. In an interference experiment, the The screen is at 2 m from the plane of the
amplitudes of the two waves are of 2 units slits. Calculate the separation between the
each. Calculate the resultant amplitude at a 8th bright fringe and the 3rd dark fringe
point P in he Interference pattern where observed with respect to the central bright
the Waves meet fringe. [Similar Delhi 2019, Chennai 2019]
(a) With a phase difference of 60°. 11. A monochromatic light of wavelength 500
(b) With a path difference of 𝜆/2. Consider nm is incident normally on a single slits of
the Interfering Waves to be In phase width 0.2 mm to produce a diffraction
initially. pattern. Find the angular width of the
6. In a Young's double slit interference central maximum obtained on the screen.
experiment, the first minima on the screen Estimate the number of fringes obtained in
is found just in front of one of the slits when Young’s double slits experiment with fringe
the slits are illuminated with a width 0.5 mm, which can be
monochromatic source of wavelength 6000 accommodated within the region of total
Å. If the distance of the screen from the slits angular spread of the central maximum due
is 60 cm, calculate the separation between to single slits.
the slits. 12. In Young’s double slits experiment, the two
7. In a double slit experiment using light of slits 0.15 mm apart are illuminated by
wavelength 600 nm, the angular width of monochromatic light of wavelength 450
the fringe formed on a distant screen is nm. The screen is 1.0 m away from the
0.1°. Find the spacing between the two slits. slits.
[Delhi 2015] (a) find the distance of the second (i) Bright
8. A parallel beam of monochromatic light of fringe, (ii) dark fringe from the central
wavelength 500 nm falls normally on a maximum.

Advent Tutorial Front of Nehru Inter College Banarsidas, Auraiya


KP Singh 7248370933 ks5122213@gmail.com Page 13
Chapter- 10 Wave optics
(b) How will the fringe pattern change if the 20. Assuming the diameter of the eye pupil to
screen is moved away from the slits? be 2.0 mm, calculate the smallest angular
13. In a single slits diffraction experiment, the separation at which two point objects can
first minima for red light (660 nm) coincide be distinctly seen when viewed in light of
with first maxima of some other wavelength wavelength 6000 Å. [HOTS]
𝜆’. find the value of 𝜆’. 21. Calculate the limit of resolution of a 100
14. Two slits are made one millimetre apart cm telescope with visible light of wavelength
and the screen is placed one metre away. = 5500 Å.
What should the double slits pattern within 22. Consider a two slit interference
the central maximum of the single slit arrangements (in figure) such that the
pattern? distance of the screen from the slits is half
15. Two wavelengths of sodium length 590 nm the distance between the slits. Obtain the
and 596 nm are used, in turn, to study the value of D in terms of A such that the first
diffraction taking place due to single slit of minima on the screen falls at a distance D
aperture 1x10-4 m. The distance between from the centre O.
the slit and the screen is 1.3 m calculate the T OP = x
S1
separation between the positions of the first 1P
CO = D
maxima of the diffraction pattern obtained 𝑆 O
C
in the two Cases. [Delhi 2013] S1C = CS2 = D
S2 T
16. In a single slit diffraction experiment, a slit
2

of width d is illuminated by red light of


wavelength 650 nm. For what value of d 23. The human eye has an approximate angular
will resolution of Ф = 5.8x10-4 rad and a typical
(i) the first minimum fall at an angle of photo printers a minimum of 300 dpi (dots
diffraction of 30°, and per inch, 1 inch = 2.54 cm). At what
(ii) the first maximum fall at an angle of minimal distance z should a printed page be
diffraction of 30°? held so that one does not see the individual
17. A parallel beam of light of wavelength 600 dots.
nm is incident normally on a slit of width a. 24. For the same objective, find the ratio of the
If the distance between the slits and the least separation between two point to be
screen is 0.8 m and the distance of 2nd distinguished by a microscope for light of
order maximum from the centre of the 5500 Å and electrons accelerated through
screen is 15 mm, calculate the width of the 100 V used as the illuminating substance.
slit. 25. Consider a point at the focal point of a
18. What is the value of polarising angle of a convergent lens. Another convergent lens of
medium of refractive index √3? short focal length is placed on the other
19. Assume that light of wavelength 6000 Å is side. What is the nature of the Wave-fronts
coming from a star. What is the limit of emerging from the final image?
resolution of a telescope whose objective has 26. What is the shape of the wave-front on
a diameter of 100 inch? earth for sunlight?

Advent Tutorial Front of Nehru Inter College Banarsidas, Auraiya


KP Singh 7248370933 ks5122213@gmail.com Page 14
Chapter- 10 Wave optics
27. A polariod (I) is placed in front of a observed that the first minimum is at a
monochromatic source. Another polariod (II) distance of 2.5mm from the centre of the
is placed in front of this polaroid (I) and
screen. Calculate the width of the slit.
rotated till no light passes. A third polaroid
Short answer type questions (3 Marks)
(III) is now placed in between (I) and (II).
7. Use Huygens’ principle to show how a plane
In this case, will light emerge from (II).
Explain. wave front propagates from denser to rare

Very short answer type questions (1 Marks) medium. Hence verify Snell’s law of refraction.

1. Which of the following wave can be polarised 8. State clearly how an unpolarised light get

i. Heat waves linearly polarised, when passes through a

ii. Sound waves? Give the reason to support Polaroid.

your answer. i. Unpolarised light of intensity I0 is incident

2. How does the fringe width in Young’s double on a polaroid P1 which is kept near

slit experiment change, when the distance of polaroid P2 whose pass axis is parallel to

separation between the slit and screen is that of P1. How will the intensities of light,

doubled? I1 and I2 transmitted by the polaroids P1

3. When light travels from rarer to a denser and P2 respectively, change on rotating P1

medium, the speed decreases. Does this without disturbing P2?

decrease in speed imply a decrease in the ii. Write the relation between the intensities

energy carried by the light wave? Justify your I1 and I2.

answer. 9. (i). using the phenomenon of polarisation, show

Short answer type questions (2 Marks) how transverse nature of light can be

4. Find an expression for intensity of transmitted demonstrated.

light, when a Polaroid sheet rotated between ii. Two polaroids P1 and P2 are placed with

two crossed Polaroid. In which position of the their pass axes perpendicular to each other.

Polaroid sheet will the transmitted intensity is Un-polarised light of intensity I0 is incident

maximum? on P1. A third polaroid P3 is kept in

5. For a single slit of width a, the first minimum between P1 and P2 such that its pass axis

of the interference pattern of a monochromatic makes an angle of 30° with that of P1.

light of wavelength 𝜆 occurs at an angle of . At


𝜆
Determine the intensity of light transmitted
𝑎

the same angler of


𝜆
, we get a maximum for on through P1, P2 and P3.
𝑎

two narrow slits serated by a distance a. 10. Define a wave-front. Use Huygens geometrical

explain. construction to show the propagation of plane

6. A parallel beam of light of 500 nm falls on a wave-front from rarer medium

narrow slit and the resulting diffraction i. To a denser medium

pattern is observed on a screen 1m away. It is ii. Undergo refraction, hence derive snell’s law

of refraction

Advent Tutorial Front of Nehru Inter College Banarsidas, Auraiya


KP Singh 7248370933 ks5122213@gmail.com Page 15
Chapter- 10 Wave optics
11. i. what is linearly polarised light? Describe pattern let, displacement of the wave produce

briefly using a diagram how sunlight is by S1 and S2 is given by S1 = acos 𝜔𝑡 and S2 = =

polarised. bcos(𝜔𝑡 + Ф) find out the expression for the

ii. Un-polarised light is incident on a polaroid. amplitude of the resultant displacement at a

How would the intensity of transmitted point and show that the intensity at that point

light change when the polaroid rotated. will be I= 4a2cos2 hence establish the condition
Ф
2

12. i. why are coherent sources necessary to for constructive and destructive interference.

produce a sustained interference pattern? (b). what is the effect of on the interference

ii. In Young’s double slit experiment using fringes in Young’s double slit experiment when

monochromatic light of wavelength 𝜆, the (i) the width of the source slit is increased

intensity of light at a point on the screen. (ii) the monochromatic source is replaced by a

Where path difference is 𝜆 is K unit. Find source of white light?

out the intensity of light at a point where 16. i. describe briefly how a diffraction pattern is

the path difference is 𝜆/3? obtained on a screen due to a narrow slit

Long answer type questions (5 Marks) illuminated by a monochromatic source of light.

13. i. In a double slit experiment using light of Hence, obtain the condition for the angular

wavelength 600nm, the angular width of the width of secondary maxima and secondary

fringe formed on a distant screen is 0.1°. Find minima.

the spacing between the two slits. ii. Two wavelength s of sodium light of 590nm

ii. Light of wavelength 5000Å propagating in and 596nm are used in turn to study the

air gets partly reflected from the surface of diffraction taking place at a single slit of

water. How will the wavelength and aperture 2×10-6m. the distance between the

frequencies of the reflected light and refracted slit and the screen 1.5m. Calculate the

light be affected? seperat6ion between the positions of first

14. i. using Huygens principle construction of maxima of the diffraction pattern obtained

secondary wavelets explain how a diffraction in the two cases.

pattern is obtained on a screen due to a 17. i. In Young’s double slit experiment, describe

narrow slit on which a monochromatic beam of briefly how bright and dark fringes are

light is incident normally. obtained on the screen kept in front of a

ii. Show that the angular width of the first double slit. Hence, obtain the expression for the

diffraction fringe is half that of the central fringe width.

fringe. ii. The ratio of the intensities at minima to the

iii. Explain why the maxima at 𝜃=(𝑛 + ) maxima in the Young’s double slit experiment
1 𝜆
2 𝑎

become weaker and weaker with increasing n. is 9:25 find the ratio of the widths of the two

15. (a). consider two coherent sources S1 and S2 slits.

producing monochromatic wave to produce

Advent Tutorial Front of Nehru Inter College Banarsidas, Auraiya


KP Singh 7248370933 ks5122213@gmail.com Page 16
Chapter- 10 Wave optics
18. (a) two independent monochromatic sources of are inclined at angle of 60° and 90° with

light cannot produce a sustain interference respect to the pass axis of P1.

pattern. Give reason. A monochromatic source S of unpolarised light

(b) Light waves each of amplitude a and of intensity I is kept in front of the polaroid

frequency ω, emanating from two coherent sheet P1 as shown in figure determine the

light sources superpose at a point. If the intensities of light as observed by the observers

displacements due to these waves is given by O1, O2 and O3 as shown in the figure

y1 = acos 𝜔𝑡 and y2 = = bcos(𝜔𝑡 + Ф), where Ф

is the phase difference between the two, S O1 O2 O3


obtain the expression for the resultant D
intensity at the point.
P1 P2 P3
19. i. use Huygens’ geometrical construction to 22. i. In Young’s double slit experiment, derive the

show how a plane wave front at t = 0 condition for


a. Constructive interference
propagate and produces a wave front at a later
b. Destructive interference at the point on
time.
the screen
ii. Verify, using Huygens principle, Snell’s law of
ii. A beam of light consisting of two
refraction of a plane propagating from a wavelengths, 800 nm and 600 nm is used to
denser to a rarer medium. obtain the interference fringes on a screen
iii. When monochromatic light is incident on a placed 1.4m away in a Young’s double slit

surface separated two media, the reflected experiment. If the two slits are separated by
0.28nm, calculate the least distance from the
and refracted light both have the same
central bright maximum, where the bright
frequency. Explain why?
fringes of the two wavelengths coincide.
20. i. State Huygens’ Principle. Using this principle
23. i. How does an un-polarised light incident on a
draw a diagram to show how a plane wave- polaroid gets polarised? Describe briefly, with
front incident at the interface of the two the help a necessary diagram, diagram the
media gets refracted, when it propagates from polarisation of light by reflection from a

a rare to a denser medium. Hence, verify the transparent medium.


ii. Two polaroids A and B are kept in crossed
snell’s law of refraction.
position. How would a third polaroid, C be
ii. Is the frequency of reflected and refracted
placed between them , so that the intensity of
light same as the frequency of incident light?
polarised light transmitted by polaroid, B
21. (i). Describe briefly how an unpolarised light reduced to 1/8th of the intensity of un-
get linearly polarised when it pass through a polarised light incident on A?

polaroid.

(ii). Three identical polaroid sheet P1, P2 and P3

are oriented, so that the pass axis of P2 and P3

Advent Tutorial Front of Nehru Inter College Banarsidas, Auraiya


KP Singh 7248370933 ks5122213@gmail.com Page 17
11 Dual Nature of Matter
and Radiation

Chapter
Electron volt- It is the kinetic energy gained by
Electron- It is the elementary particle revolving an electron when it is accelerated through a
around the nucleus having a negative charge of potential difference of 1 volt. 1eV= 1.6×10-19
1.6×10-19C and mass 9.1×10-31kg. joule, 1MeV=1.6×10-13 joule the work function
of metal is generally measured in electron volt.
Work function- The minimum amount of energy
required by the electron to just escape from the Photons- According to plank’s quantum theory
metal surface is known as the work function of of radiation, an electromagnetic wave travels in
the metal. It is denoted by the W0. the form of discrete packets of energy called
quanta. One quanta of light radiation is called a
Electron emission- The phenomenon of
photon. The main features of photons are as
ejaculation of electron from a metal surface is
follows:
called electron emission. It is of following types:

i. A photon travel with the speed of light


i. Thermionic emission- Here the electrons are
3×108 m/sec.
emitted by from the metal surface with the
ii. The frequency does not change as it travels
help of thermal energy.
from one medium to another medium.
ii. Field or cold cathode emission- Electrons are
iii. The speed of photon changes as it travels
emitted from the metal surface by subjecting
through different media due to the change
it to a very high electric field.
in its wavelength.
iii. Photoelectric emission- Electrons are emitted
iv. The rest mass of photon is zero i.e., a
from metal surface with the help of suitable
photon cannot exist at rest.
electromagnetic radiations.
v. Energy of photon, E=hʋ= .
ℎ𝑐

iv. Secondary emission- Electrons are ejected 𝜆


vi. Momentum of photon, p=mc= = .
ℎʋ ℎ

from the metal surface by sticking fast 𝑐 𝜆


vii. From Einstein’s mass-energy relationship,
moving electrons over it.
the equivalent mass m of a photon is given
by E= mc2 = hʋ or m = .
ℎʋ
Kinetic energy gained by the electron- When an 𝑐2

electron is accelerated from rest through a


Photoelectric effect- The phenomenon of emission
potential difference of V volts, the gain in its
of electrons from a metal surface, when
kinetic energy is eV= m𝑣 2 .
1
2
electromagnetic radiations of sufficiently high
frequency are incident on it, is called photoelectric
Chapter- 11 Dual Nature of Mater and radiation
effect. The photon (light)-generated electrons are ii. For a metal there will be a certain minimum
called photoelectrons. The alkali metal like Li, Na, frequency below which no photoelectrons are
K, Ce show photoelectric effect with visible light. emitted, however high is the intensity of
Metal like Zn, Cd, Mg respond to ultraviolet light. incident radiation. This frequency is called
threshold frequency.
Photoelectric emission involves the conversion of iii. For the radiation higher than threshold
light energy into electrical energy. It follows the frequency, the maximum kinetic energy of the
law of conservation of energy. It is an photoelectrons is directly proportional to the
instantaneous process. frequency of incident radiation
iv. The photoelectric emission is an instantaneous
Photoelectric current- The current constituted by
process.
photoelectrons is called photoelectric current. It
depends on- Failure of wave theory to explain photoelectric
effect- The classical wave theory of radiation could
(i) The intensity of light
not explain the main features of photoelectric
(ii) The nature of the cathode materials.
effect. Its picture of continuous absorption of
(iii) The potential difference applied between the
energy from radiation could not explain.
two electrodes.

i. The independence of Kmax on intensity


Cut off or stopping potential- It is the minimum
ii. The existence of threshold frequency ʋ0
value of negative potential that must be applied to
iii. The instantaneous nature of the phenomenon.
the anode of photo-cell to make the photoelectric
current zero. It is denoted by V0. Its value depends Einstein’s theory of photoelectric emission-
upon (i) the frequency of the incident radiation of Einstein explained photoelectric effect with help of
light (ii) the nature of cathode material. For a planks quantum theory. When a radiation of
given frequency of incident light, it is independent frequency ʋ is incident on the metal surface, it is
of its intensity. The stopping potential is directly absorbed in the form of discrete energy packet of
proportional to the kinetic energy of the emitted energy called photons or quanta. A part of energy
electron. Kmax= m𝑣𝑚𝑎𝑥 =eV0.
12
2 of hʋ is used in removing the electron from the
metal surface and remaining energy used in
Threshold frequency- The minimum value of the
providing kinetic energy to the photoelectron.
frequency of incident radiation below which the
Einstein’s photoelectric equation is Kmax=
1

photoelectric emission stops altogether is called 2


m𝑣𝑚𝑎𝑥
2
= eV0 = hʋ-W0 = ℎ(ʋ-ʋ0). Where W0 is the
threshold frequency. It is denoted by ʋ0 , and is
work function of metal and ʋ0 is the threshold
characteristic of the metal.
frequency.

Law of photoelectric effect-

i. A given metal and a radiation of fixed


frequency, the rate of photoelectric emission V
of photoelectrons is proportional to intensity V
µA
of light incident.

Advent Tutorial Front of Nehru Inter College Banarsidas, Auraiya


KP Singh 7248370933 ks5122213@gmail.com Page 2
Chapter- 11 Dual Nature of Mater and radiation
Photocell- It is an arrangement which converts
I3/ i3
light energy into electrical energy. It works on the
I2/ i2
principle of photoelectric effect. It is used in I1/ i1
cinematography for the reproduction of sound.
Photocells are used to operate various control
systems and light measuring devices.
Electrode potential
Dual nature of radiation- Light has dual nature. It Kinetic energy
manifests itself as a wave in diffraction,
interference, polarisation, etc., while it shows
particle nature in photoelectric effect, Compton
ʋ0
scattering etc.
Frequency (ʋ)
Dual nature of matter- As there is complete −𝑊0
equivalency between matter (mass) and radiation
(energy) and the principle of symmetry is always
obeyed, de-Broglie suggest that moving particles
Intensity
like protons, neutrons, electrons, etc. should be Saturation current are same while
associated with the waves known as de-Broglie stopping potential increases with
rese in frequency
waves and their wavelength is called de-Broglie Photo
Photoelectric current
wavelength. The de-Broglie wavelength associated current
i3> i2> i1
with a particle of mass m and velocity v is 𝜆 =

𝑝
= . Where h is plank constant. p = mv = √2𝑚𝐾

𝑚𝑣 Very Short Answer/Objective Type Questions [1
and kinetic energy acquire by the charge particle q
Mark]
accelerated through a potential of V is given by K
1. Light of frequency 1.9 times the threshold
= eV then p = √2𝑚𝑒𝑉, 𝜆= for electron it is

√2𝑚𝑒𝑉 frequency is incident on a photosensitive
.
12.27Å
𝜆=
√𝑉 material. If the frequency is halved and
intensity is doubled, the photocurrent becomes
Davisson and Germer experiment- This
(a) quadrupled (b) doubled
experiment has verified and confirmed the wave
(c) halved (d) zero
nature of electron beam.
Ans, (d)

Electron microscope- It is a device that exploits 2. Threshold wavelength for a metal having work

the wave nature of electrons to provide high function W0 is 𝜆. What is the threshold

resolving power. It is used to investigate the wavelength for the metal having work function

structural details of bacteria, virus, etc it has 2W0?

proved to be a powerful tool of investigation (a) 4𝜆 (b) 2 𝜆 (c) 𝜆/2 (d) 𝜆/4

research in science, technology, metallurgy, Ans. (c)

industry, medicine, etc. 3. Radiations of frequency ʋ are incident on a


photosensitive metal. The maximum K.E. of the
photoelectrons is E. When the frequency of the

Advent Tutorial Front of Nehru Inter College Banarsidas, Auraiya


KP Singh 7248370933 ks5122213@gmail.com Page 3
Chapter- 11 Dual Nature of Mater and radiation
incident radiation is doubled, what is the (c) 2.7 A (d) None of these
maximum kinetic energy of the photoelectrons? Ans. (c)
(a) 2E (b) 4E (c) E + hʋ (d) E - hʋ 8. The figure shows the variation of photocurrent
Ans. (c) with anode potential for a photosensitive
4. Maximum kinetic energy (E) of a photoelectron surface for three different radiations. Let Ia, Ib,
Varies with frequency (ʋ) of the incident and Ic, be the intensities and ʋa, ʋb and ʋc be the
radiation as frequencies for the curves a, b and c
(b). respectively. Then the correct relation is
(a).
Ek (a) ʋa = ʋb and, Ia ≠ Ib I
Ek
(b) ʋa = ʋc and, Ia = Ic
b
(c) ʋa = ʋb and, Ia = Ib c
a
ʋ (d) ʋb = ʋc and, Ib = Ic O
ʋ
(c). Ans. (a)
(d). Anode potential V
9. The slope of the stopping potential versus
Ek Ek
frequency graph for photoelectric effect is equal
to
ʋ (a) h
ʋ V0
(b) he
Ans. (d) (c) h/e
5. The stopping potential V0, for photoelectric (d) e O ʋ
ʋ0
∅0
emission from a metal surface is plotted along Ans. (c)
𝑒
y-axis and frequency ʋ of incident light along 10. A proton and an α-particle are accelerated by
x-axis. A straight line is obtained as shown. the same potential difference. The ratio of their
Planck's constant is given by de-broglie wavelengths (𝜆p/ 𝜆α) is
(a) slope of the line (a) 1 (b) 2 (c) √8 (d)
1
√8
(b) product of the slope of the Ans. (c)
V0
line and charge on electron 11. Work function of three metals A, B and C are
(c) intercept along y-axis divided 4.5 eV, 4.3 eV and 3.5 eV respectively. If a
by charge on the electron O ʋ light of wavelength 4000 Å is incident on the
ʋ0
(d) product of the intercept along metals then
x-axis and mass of the electron (a) Photoelectrons are emitted from A.
Ans. (b) (b) Photoelectrons are emitted from B.
6. The energy of photon of Wavelength 450 nm is (c) Photoelectrons are emitted from C.
(a) 2.5x 10-17 J (b) 1.25 x 10-17 J (d) Photoelectrons are emitted from all the
(c) 4.4 x 10-19 J (d) 2.5x 10-17 J metals.
Ans. (c) Ans. (c)
7. The kinetic energy of an electron is 5 eV, 12. The photoelectric effect can be explained by
Calculate the de broglie wavelength associated (a) Corpuscular theory of light
with it. (h = 6.6x10-34 J-s, m = 9.1x10-31 kg) (b) Wave nature of light
(a) 5.47 A (b) 10.9 A (c) Bohr's theory

Advent Tutorial Front of Nehru Inter College Banarsidas, Auraiya


KP Singh 7248370933 ks5122213@gmail.com Page 4
Chapter- 11 Dual Nature of Mater and radiation
(d) Quantum theory of light Ans. (a)
Ans. (d) 17. The rest mass of a photon of wavelength 𝜆 is
13. Which of the following figure represents the (a) zero (b) (c) (d)
ℎ ℎ ℎ𝑐
𝑐𝜆 𝜆 𝜆
variation of particle momentum and associated Ans. (a)
de-Broglie wavelength? 18. Photoelectric effect is based on the law of
conservation of
(a).
(b). (a) energy (b) mass
p
p (c) linear momentum (d) angular momentum
Ane (a)
19. Einstein's photoelectric equation is:
𝜆
(c). (a) ℎʋ = ℎʋ0 + (b) ℎʋ0 = ℎʋ +
1 1
𝜆 𝑚𝑣 2 𝑚𝑣 2
(d). 2 2
(c) ℎʋ = ℎʋ0 − (d) 2ℎʋ = ℎʋ0 + 𝑚𝑣 2
1
p 𝑚𝑣 2
p 2
Ans. (a)
20. In photoelectric effect, the number of

𝜆 photoelectrons emitted is proportional to


𝜆
Ans. (d) (a) intensity of incident beam.
14. According to Einstein's photoelectric equation (b) frequency of incident beam.
the plot of the kinetic energy of the emitted (c) velocity of incident beam.
photo-electrons from a metal vs the frequency (d) work function of photo cathode.
of the incident radiation gives a straight line Ans. (a)
whose slope 21. For a given kinetic energy which of the
(a) depends on the nature of the metal used. following has smallest de Broglie wavelength?
(b) depends on the intensity of the radiation. (a) Electron (b) Proton
(c) depends both on the intensity of the (c) Deutron (d) α-particle
radiation and the metal used. Ans. (d)
(d) is the same for all metals and independent 22. Which of the following shows particle nature of
of the intensity of the radiation. light?
Ans. (d) (a) Photoelectric effect (b) Refraction
15. If K.E. of free electron is doubled, its de Broglie (c) Interference (d) Polarisation
wavelength will change by factor Ans. (a)
(a) (b) √2 (c) (d) 2 23. A particle is dropped from a height H. The de-
1 1
√2 2
Ans. (a) Broglie wavelength of the particle as a function
16. Work function of metal is of height is proportional to [NCERT Exemplar]
(a) the minimum energy required to free an (a) H (b) H1/2 (c) H0 (d) H-1/2
electron from surface against coulomb forces. Ans. (d)
(b) the minimum energy required to free an 24. A proton, a neutron, an electron and an α-
nucleon. particle have same energy. Then their de
(c) the minimum energy to ionise an atom. Broglie wavelengths compare as [NCERT
(d) the minimum energy required to eject an Exemplar]
electron orbit. (a) 𝜆𝑝 = 𝜆𝑛 > 𝜆𝑒 > 𝜆𝛼 (b) 𝜆𝛼 < 𝜆𝑝 = 𝜆𝑛 < 𝜆𝑒

Advent Tutorial Front of Nehru Inter College Banarsidas, Auraiya


KP Singh 7248370933 ks5122213@gmail.com Page 5
Chapter- 11 Dual Nature of Mater and radiation
(c) 𝜆𝑒 < 𝜆𝑝 = 𝜆𝑛 > 𝜆𝛼 (d) 𝜆𝑒 = 𝜆𝑝 = 𝜆𝑛 = 𝜆𝛼 (b) The particle could not be moving in an
Ans. (b) elliptic orbit with origin as its focus.
25. Relativistic corrections become necessary when (c) When the de Broglie wavelength is 𝜆1, the
the expression for the kinetic energy 1/2 mv2, particle is nearer the origin than when its value
becomes comparable with mc2, where m is the is 𝜆2.
mass of the particle. At what de Broglie (d) When the de Broglie wavelength 𝜆2 the
wavelength, will relativistic corrections become particle is nearer the origin than when its value
important for an electron? 𝜆1 .
(a) 𝜆 = 10 nm (b) 𝜆 = 10-1 nm Ans. (d)
(c) 𝜆 = 10-4 nm (d) 𝜆 = 10-3 nm
Ans. (c) 29. According to Planck's quantum theory of
26. Two particles A1 and A2 of masses m1, m2, (m1 radiation, an electromagnetic wave travel in
> m2) have the same de Broglie wavelength. the form packets of energy called _____________ .
Then, Ans. photon
(a) their momenta are the same. 30. _____________ experiment has verified and
(b) their energies are the same. confirmed the wave nature of electrons.
(c) energy of A1, is greater than the energy of Ans. Davisson and Germer
A2, 31. Photoelectric emission occurs only when the
(d) energy of A1, is equal to the energy of A2 frequency of incident radiation is ___________
Ans. (a) than threshold frequency.
27. Photons absorbed in a matter are converted to Ans. greater
heat. A source emitting n photon/sec of 32. Photoelectric emission is an ____________ process.
frequency ʋ is used to convert 1 kg or ice at Ans. instantaneous
0°C to water at 0°C. Then, the time T taken 33. De Broglie wavelength associated with an
for the conversion electron beam accelerated through a potential
(a) increases with increasing n, with ʋ fixed difference V is _________________ proportional to
(b) decreases with n fixed, v increasing. √𝑉.
(c) does not remain constant with n and ʋ Ans. inversely
changing such that nʋ = constant 34. ____________ is the minimum amount of energy
(d) increases when the product nʋ increases. required to cause photoelectric emission.
Ans. (b) Ans. Wok function
28. A particle moves in a closed orbit around the 35. Wave associated with the material particle is
origin, due to a force which is directed towards known ______________ as
the origin. The de Broglie wavelength of the Ans. matter wave
particle varies cyclically between two values 𝜆1,
𝜆2 with 𝜆1 > 𝜆2. Which following statements are 36. Name the phenomenon which shows the
true? quantum nature of electromagnetic radiation.
(a) The particle could not be moving in a [AI 2017]
circular orbit with origin as centre. Ans. Photoelectric effect.

Advent Tutorial Front of Nehru Inter College Banarsidas, Auraiya


KP Singh 7248370933 ks5122213@gmail.com Page 6
Chapter- 11 Dual Nature of Mater and radiation
37. Define the term 'threshold frequency' in different intensities I1 and I2, where I2 > I1
relation to photoelectric effect. [Delhi 2019, [CBSE 2018, Foreign 2014]
Foreign 2011] 42. The given graph shows the variation of
38. Show the variation of photocurrent with photoelectric current (I) versus applied voltage
collector plate potential for different (V) for two different photosensitive materials
frequencies but same intensity of incident and for two different intensities of the incident
radiation. [AI 2017, Foreign 2011] radiation. Identify the pairs of curves that
Ans. For same intensity but different correspond to different materials but same
frequencies ʋ1 > ʋ2 > ʋ3 of incident radiation. intensity of incident radiation. [Delhi 2013]

Photoelectric Saturation
current Photoelectric
current
current
Saturation
current 1
ʋ1
ʋ2 4
ʋ3 2

-V01 -V02 -V03 3


Collector Plate V
Retarding Potential Vo1 Vo2 V
Potential
Ans. The materials with same intensity of
39. In photoelectric effect, why should the incident light are 1 and 3; 2 and 4.
photoelectric current increases as the intensity 43. The graph shows the variation of stopping
of monochromatic radiation incident on a potential with frequency of incident radiation
photosensitive surface is increased? Explain. for two photosensitive metals A and B. Which
[Foreign 2014] one of the two has higher value of work
Ans. An increase in intensity means increase in function? Justify your answer. [Chennai 2019,
number of photons and thus, increase in Dehradun 2019]
photoelectric current.
40. Show the variation of photocurrent with
collector plate potential for different intensities
V0

but same frequency of incident radiation. [Delhi O ʋ0 ʋ’0


2019, AI 2017, Foreign 2011] Frequency of
incident
radiation
Photoelectric I3 ∅0

current 𝑒
I2
I1 ∅′0

𝑒
Saturation
current
Ans. As threshold frequency of metal A is
greater, its work function ∅ = ℎʋ0 , will also be
-V0 Collector Plate V greater than that of B.
Retarding Potential
Potential 44. For a given photosensitive material and with a
41. Plot a graph showing the variation of source of constant frequency of incident
photoelectric current with collector plate radiation, how does the photocurrent vary with
potential at a given frequency but for two

Advent Tutorial Front of Nehru Inter College Banarsidas, Auraiya


KP Singh 7248370933 ks5122213@gmail.com Page 7
Chapter- 11 Dual Nature of Mater and radiation
the intensity of incident light? [Delhi 2014, AI 49. State de Broglie hypothesis. [Delhi 2012]
2011C] Ans. "Particles of matter moving with a given
Ans. The photoelectric current increases linearly velocity (momentum) can display wave-like
with the intensity of light as shown in the properties".
figure. 50. Write the expression for the de Broglie
Photocurrent wavelength associated with a charged particle
having charge q and mass m, when it is
accelerated by a potential V. [AI 2013]
Intensity
Ans. De Broglie wavelength associated with a
45. Ultraviolet radiations of different frequencies ʋ1 charged particle when accelerated by a
and ʋ2 are incident on two photosensitive potential V is given by 𝜆 = where m =
ℎ ℎ
=
𝑝 √2𝑚𝑞𝑉
materials having work functions W1 and W2 mass of charge particle and q = charge on the
(W1 > W2) respectively. The kinetic energy of particle
the emitted electrons is same in both the cases. 51. A proton and an electron have same kinetic
Which one of the two radiations will be of energy. Which one has smaller de Broglie
higher frequency? wavelength and why? [AI 2012]
Ans. ʋ1 > ʋ2. As W1 > W2 and K.E. is same. Ans. 𝐸𝑘 =
1
𝑚𝑣 2 =
𝑝2
2 2𝑚
From the relation, hʋ = hʋ0 + ½ mv2 𝑝 = √2𝑚𝐸𝑘
and hʋ0, = W = work function 𝜆= =
ℎ ℎ
𝑝 √2𝑚𝐸𝑘
46. Two beams, one of red light and the other of
1
blue light, of the same intensity are incident on 𝜆∝
√𝑚
a metallic surface to emit photoelectrons. 𝑚𝑒 < 𝑚𝑝 ∴ 𝜆 𝑝 > 𝜆𝑒

Which one of the two beams emits electrons of 52. Show on a graph the variation of the de Broglie

greater kinetic energy? [HOTS] wavelength (𝜆) associated with an electron with

Ans. The blue light emits the electrons of the square root of accelerating potential (V).

greater K.E., as ʋb > ʋR [Delhi 2019, Similar Panchkula 2019, Foreign

47. Can non-metals show photoelectric effect? 2012]

[HOTS] Ans.
𝜆
Ans. Yes, when they are exposed to
electromagnetic radiations of higher frequency.
A photosensitive surface emits photoelectrons √𝑉
when red light falls on it. Will the surface emit 53. Write the relationship of de-Broglie

photoelectrons when blue light is incident on it? wavelength) associated with a particle of mass

Give reason. [Foreign 2017] m in term rms of It kinetic energy E. [Delhi

Ans. Since condition for photoemission is 2011C]


ℎ𝑐
≥ ∅0 (work function) and 𝜆B < 𝜆R hg hence, Ans. The required relation is given by
𝜆

surface will emit photoelectrons. 𝜆=
√2𝑚𝐸
48. What happens to the wavelength of a photon
after it collides with an electron?
Ans. The wavelength of a photon increases.

Advent Tutorial Front of Nehru Inter College Banarsidas, Auraiya


KP Singh 7248370933 ks5122213@gmail.com Page 8
Chapter- 11 Dual Nature of Mater and radiation
54. Name an experiment which show wave nature incident radiation that is kept constant in this
of electrons. Which phenomenon was observed experiment.
in this experiment using an electron beam? 5. (b) If the potential difference used to accelerate
Ans. Experiment: Davisson and Germer electrons is doubled, by what factor does the
Phenomenon: Diffraction de-Broglie wavelength associated with the
55. Two lines, A and B, in the plot given below electrons change?
show the variation of de Broglie wavelength, 6. An electron and a photon have the same de-
versus 1⁄ , where V is the accelerating Broglie wavelength. Which one possesses more
√𝑉
potential difference, for two particles carrying kinetic energy?

the same charge. Which one of two represents a 7. Figure shows variation of stopping potential

particle of smaller mass? (V0) with the frequency (ʋ) for two
𝐴 photosensitive materials M1 and M2
𝜆
𝐵 (i) Why is the slope same for both lines?
(ii) For which material will the emitted
electrons have greater kinetic energy for the

1/√𝑉 incident radiations of the same frequency?


Justify your answer.

56. The de Broglie wavelength of a particle of 8. Find out work function of the metal, if the

kinetic energy K is . What would be the kinetic energies of the photoelectrons are E1

wavelength of the particle, if its kinetic energy and E2. With wavelengths of incident light 𝜆1

were K/4? [HOTS] and 𝜆2.


9. Derive an expression for the de Broglie

Short Answer Type Questions [2 Marks] wavelength associated with an electron

1. A proton and an alpha particle are accelerated accelerated through a potential V. Draw a

through the same potential. Which one of the schematic diagram of a localised- wave

two has (i) greater value of de Broglie describing the wave nature of the moving

wavelength associated with it and (ii) less electron.

kinetic energy. Give reasons to justify your 10. An α-particle when accelerated through a

answer. [Delhi 2017, 14] potential difference of V volt has a wavelength

2. Why do the photoelectrons have variable KE? 𝜆 associated with it. In order to have the same

3. Write Einstein's photoelectric equation. State wavelength, by what potential difference a

clearly the three salient features observed in proton must be accelerated? [HOTS]

photoelectric effect, which can be explained on 11. The de Broglie wavelength of a body moving

the basis of the above equation. [Similar with speed v is 𝜆. On its way it loses some of its

Chennai 2019] mass and gains twice the speed. Kinetic energy

4. (a) Draw a graph showing variation of also increases to twice of its initial value. What

photoelectric current (I) with anode potential will be the new value of de Broglie wavelength?

(V) for different intensities of incident [HOTS]

radiation. Name the characteristic of the 12. Plot a graph showing variation of de-Broglie
wavelength 𝜆 versus 1⁄ where V is
√𝑉

Advent Tutorial Front of Nehru Inter College Banarsidas, Auraiya


KP Singh 7248370933 ks5122213@gmail.com Page 9
Chapter- 11 Dual Nature of Mater and radiation
accelerating potential for two particles A and B 4. The following graph shows the variation of
carrying same charge but of masses m1, m2 stopping potential V0, with the frequency ʋ of
(m1 > m2). Which one of the two represents a the incident radiation for two photosensitive
particle of smaller mass and why? metals X and Y:
[Similar Delhi 2019, Delhi 2016] X
V0
13. The frequency of incident light on a metal Y
surface is doubled How will this affect the value
of K.E of emitted photoelectrons? [Chennai
2019] 0.5 1.0 ʋ( ×1015 s-1)
Long Answer Type [II] Questions [3 Marks]
1. Explain giving reasons for the following: (i) which of the metal has larger threshold
(a) Photoelectric current in a photocell increase wavelength? Give reasons.
with the increase in the intensity of the (ii) Explain, giving reason, which metal gives
incident radiation. out electrons, having larger kinetic energy, for
(b) The stopping potential (V) varies linearly the same wavelength if the incident radiation.
with the frequency (V) of the incident radiation (iii) if the distance between the light source and
for a given photosensitive surface with the slope metal X is halved, how will the kinetic energy
remaining the same for different surfaces of electron emitted from it change? Give
(c) Maximum kinetic energy of the reasons.
photoelectrons is independent of the intensity 14. (a) The graph between frequency (ʋ0) of the
of incident radiation. [AI 2017] incident light and maximum kinetic energy (Ek)
2. Light of intensity I and frequency ʋ incident on of emitted photoelectrons is given. Find the
a photosensitive surface and causes values of (i) threshold frequency, and (ii) work
photoelectric emission. What will be the effect function from the graph. [Similar Panchkula
on anode current when (i) the intensity of light 2019]
Stopping potential

is gradually increased, (ii) the frequency of


V0
incident radiation is increased, and (iii) the
anode potential is increased? In each case, all O ʋ0

other factors remain the same. Explain, giving Frequency of incident


radiation
justification in each case. [AI 2015]
∅0
3. Define the term (i) ‘cut-off voltage’ and (ii) −
𝑒
‘threshold frequency, in relation to the
phenomenon of photoelectric effect. [Delhi (b) Draw a graph showing variation of
2019] photocurrent with anode potential for a
Using Einstein’s photoelectric equation show particular intensity of incident radiation. Mark
how the cut-off voltage and threshold saturation current and stopping potential.
frequency for a given photosensitive material 15. Why photoelectric effect cannot be explained on
can be determined with the help of suitable the basis of wave nature of light? Give reasons.
plot/graph. [Delhi 2019, Panchkula 2019]

Advent Tutorial Front of Nehru Inter College Banarsidas, Auraiya


KP Singh 7248370933 ks5122213@gmail.com Page 10
Chapter- 11 Dual Nature of Mater and radiation
(b) Write the basic features of photon picture of happens if the laser source is brought nearer
electromagnetic radiation on which Einstein's and placed 50 cm away? [Delhi 2017]
photoelectric equation is based. 22. Write two characteristic features observed in
16. Define the term 'work function' of a metal. The photoelectric effect which support the photon
threshold frequency of a metal is ʋ0. When the picture of electromagnetic radiation. Draw a
light of frequency 2ʋ is incident on the metal graph between the frequency of incident
plate, the maximum velocity of electrons radiation (ʋ) and the maximum kinetic energy
emitted is v1. When the frequency of the of the electrons emitted from the surface of a
incident radiation is increased to 5Vo, the photosensitive material. State clearly how this
maximum velocity of electrons emitted is v2. graph can be used to determine (i) Planck's
Find the ratio of v1 to v2. constant and (ii) work function of the material.
17. A proton and deuteron are accelerated through [Foreign 2012]
the same accelerating potential. Which one of 23. An electromagnetic wave of wavelength 𝜆 is
the two has incident on a photosensitive surface of negligible
(a) Greater value of de Broglie wavelength work function. If the photo-electrons emitted
associated with it, and from this surface have the de Broglie
(b) Less momentum? [Similar Dehradun 2019] wavelength 𝜆1, prove that 𝜆 = (
2𝑚𝑐
) 𝜆12

18. A proton and an α-particle have the same de- 24. Red light, however bright it is, cannot produce
Broglie wavelength. Determine the ratio of (i) the emission of electrons from a clean zinc
their accelerating potentials (ii) their speeds. surface. But even weak ultraviolet radiation can
[Dehradun 2019, Delhi 2015] do so. Why? X-rays of wavelength 𝜆 fall on a
19. A particle of mass M at rest decays into two photosensitive Surface, emitting electrons.
particles of masses m1 and m2, having velocities Assuming that the work function of the surface
v1 and v2, respectively. Find the ratio of de- can be neglected, prove that the de-Broglie
Broglie wavelengths of the two particles. wavelength of electrons emitted will be √
ℎ𝜆
.
2𝑚𝑐
20. Write Einstein's photoelectric equation and
25. Why are de Broglie waves associated with a
point out any two characteristic properties of
moving football not visible? The wavelength 𝜆,
photons on which this equation is based.
of a photon and the de Broglie wavelength of
Briefly explain the three observed features
an electron have the same value. Show that the
which can be explained by this equation. [Delhi
energy of the photon is times the kinetic
2𝜆𝑚𝑐

2019, AI 2013]

energy of the electron, where m, c, and h have
21. (i) How does one explain the emission of
their usual meanings.
electrons from a photosensitive surface with the
26. An electron is accelerated from rest through a
help of Einstein's photoelectric equation?
potential V. Obtain the expression for the de-
(ii) The work function of the following metals is
Broglie wavelength associated with it. [Foreign
given: Na = 2.75 eV, K = 2.3 eV, Mo = 4.17 eV
2014]
and Ni = 5.15 eV. Which of these metals will
not cause photoelectric emission for radiation
Numerical Problems
of wavelength 3300 Å from a laser source
27. Monochromatic light of frequency 6.0x1014 Hz
placed 1 m away from these metals? What
is produced by a laser. The power emitted is

Advent Tutorial Front of Nehru Inter College Banarsidas, Auraiya


KP Singh 7248370933 ks5122213@gmail.com Page 11
Chapter- 11 Dual Nature of Mater and radiation
2.0x10-3 W. Estimate the number of photons Metal Work function (eV)
emitted per second on an average by the Na 1.92
source. [NCERT Exemplar] K 2.15
28. Consider a metal exposed to light of wavelength Ca 3.20
600 nm. The maximum energy of the electron Mo 4.17
is doubled when light of wavelength 400 nm is
used. Find the work function in eV. 34. The kinetic energy of the electron orbiting in
29. When radiations of wavelengths 3000 Å and the first excited state of hydrogen atom is 3.4
4000 Å fall on the surface of metals A and B, eV. Determine the de Broglie wavelength
and the photoelectrons emitted have maximum associated with it. [Foreign 2015]
kinetic energies of 2 eV and 1 eV respectively. 35. An electron is accelerated through a potential
Calculate the maximum wavelength of the difference of 100 volts. What is the de-Broglie
incident radiation for which there will be wavelength associated with it? To which part of
photoelectron emission from the same surface. the electromagnetic spectrum does this value of
30. By how much would the stopping potential for wavelength correspond? [NCERT Exemplar]
a given photosensitive surface go up if the 36. In the study of a photoelectric effect the graph
frequency of the incident radiations were to be between the stopping potential V and
increased from 4x1015 Hz to 8x1015 Hz? frequency ʋ of the incident radiation on two
[Foreign 2017] different metals P and Q is shown below:
31. Light of wavelength 2500 Å falls on a metal
surface of work function 3.5 eV. What is the P
V0 4
Q
kinetic energy (in eV) of (i) the fastest and (ii) (Volt)
the slowest electrons emitted from the surface? 2
If the same light falls on another surface of
work function 5.5. eV, what will be the energy 3.0 6.0 ʋ (×1014 s-1)
-2
of emitted electrons [Foreign 2011]
32. The ground state energy of hydrogen atom is - (i) Which one of the two metals has higher
13.6 eV. The photon emitted during the threshold frequency?
transition of electron from n = 2 to n = 1 (ii) Determine the work function of the metal
state, is incidence photosensitive material of which has greater value.
unknown work function. The photoelectrons are (ii) in Find the maximum kinetic energy of
emitted from the mater with a maximum electron emitted by light of frequency 8x1014
kinetic energy of 8 eV. Evaluate the threshold Hz for this metal. [Delhi 2017]
wavelength of the material used. 37. An electron and a proton, each have de-Broglie
33. If light of wavelength 412.5 nm is incident on wavelength of 1.00 nm.
each of the metals given below, which ones will (i) Find the ratio of their momenta.
show photoelectric emission and why? [CBSE (ii) Compare the kinetic energy of the proton
2018] with that of the electron. [Foreign 2013]
38. The work function of caesium is 2.14 eV. Find
(a) the threshold frequency for caesium, and (b)

Advent Tutorial Front of Nehru Inter College Banarsidas, Auraiya


KP Singh 7248370933 ks5122213@gmail.com Page 12
Chapter- 11 Dual Nature of Mater and radiation
the wavelength of the incident light if the 3. The frequency ʋ of incident radiation is
photocurrent is brought to zero by a stopping greater than threshold frequency ʋ0 in a
potential of 0.60 V. photocell. How will the stopping potential
39. The wavelength of light in the visible region is vary, if frequency ʋ is increased, keeping other
about 390 nm for violet colour, about 550 nm factors constant.
(average wavelength) for yellow green colour 4. Define the term stopping potential in relation
and About 760 nm for red colour. to photoelectric effect.
(a) What are the energies of photons in (eV) at 5. Show graphically the variation of
the (i) violet and, (ii) average wavelength, photoelectric current with frequency of the
yellow Green colour, and (iii) red end of the incident photons.
visible spectrum? (Take h = 6.63x10-34 J-s and 6. With what purpose Davisson-Germer
1 eV = 1.6x10-19 J) experiment for electrons was performed?
(b) From which of the photosensitive materials 7. Define intensity of radiation on the basis of
with work functions listed in Table 11.1 and photon picture of light. Write SI unit.
using the result of (i), (ii) and (iii) of (a) can 8. The graph shows the variation of stopping
you built a photoelectric device that operates potential with frequency of incident
with visible light? [NCERT Exemplar] radiation for two photosensitive metal A and
40. A particle is moving three times as fast as an B. which one of the two has higher
electron. The ratio of the de-Broglie value of work function? Justify your answer.
wavelength of the particle to that of the 9. The given graph shows the variation of
electron is 1.813x10-4 .Calculate the particle's photoelectric current I versus applied Voltage
mass and identify the particle. [AI 2011C] V for two difference photosensitive materials
41. Given the ground state energy E0 = -13.6 eV and for two different photosensitive materials
and Bohr radius a0 = 0.53Å. Find out how the and for different intensities of the incident
de-Broglie wavelength associated with the radiations. Identify the pairs of curves that
electron orbiting in the ground state would correspond to different materials but same
change when it jumps into the first excited intensity of incident radiation.
state. [AI 2015]
P Q
Stopping potential

Very short answer type questions (1 mark)


1. The de-Broglie wavelength associated with an
electron accelerated through a potential frequency
difference V is 𝜆. what will be its wavelength
when the accelerating potential is increase to
5V? 10. Show on a plot the nature of variation of
2. Two metals M1 and M2 have work function photoelectric current with the intensity of
2eV and 4eV, respectively. Which of the two radiation incident on a photosensitive surface.
has a higher threshold wavelength for the 11. Write the expression for the de-Broglie
photoelectric emission? wavelength associated with a charged particle

Advent Tutorial Front of Nehru Inter College Banarsidas, Auraiya


KP Singh 7248370933 ks5122213@gmail.com Page 13
Chapter- 11 Dual Nature of Mater and radiation
having charge q and mass m when it is of electromagnetic spectrum does this
accelerated by a potential V. wavelength correspond?
12. State de-Broglie hypothesis.
13. A proton and an electron have same kinetic 18. An electron is accelerated through a potential
energy. Which one has greater de-Broglie difference of 80V. What is the de-Broglie
wavelength and why? wavelength associated with it? To which part
of electromagnetic spectrum does this value of
wavelength correspond?
Short answer type questions (2 marks)

19. Calculate the ratio the accelerating potential


14. The graph between the stopping potential V
required to accelerate
and frequency f of the incident radiation on
i. A proton and
two metal plates P and Q are given as.
ii. An 𝛼- particle to have the same de-Broglie
i. Which metal P and Q has the greater value of
wavelength associated with them.
the work function?
ii. What does the slop of the lines depict?
20. For what kinetic energy of a neutron, will the
associated de-Broglie wavelength be
Stopping potential V0

Metal A Metal B
2.64×10-10m?
1
I
2 21. A proton and an 𝛼- particle have the same
𝑊0
Frequency of
de-Broglie wavelength. Determine the ratio of
3 4 incident radiation ʋ
𝑒 (i) their accelerating potential
V 𝑊0 ′ (ii) their speeds.
𝑒
15. The de-Broglie wavelength of a body moving
22. A proton and a deuteron are accelerated
with speed ʋ is 𝜆. on its way,
through the same accelerating potential.
It losses some of its mass and gain twice the
Which one of the two has
speed. Kinetic energy also
i. Greater value of de-Broglie wavelength
Increases to twice of its initial value. What will
associated with it and
be the new value of
ii. Less momentum? Give reasons to Justify your
de-Broglie wavelength?
answer.

16. What are the energies of photons at the (i)


Short answer type questions (3 marks)
violet and (ii) red ends of the visible
1. Write the Einstein’s photoelectric
Spectrum? The wavelength of light is about
equation relating the maximum kinetic
390nm for violet and about 760nm for red.
energy of the emitted electron to the
17. An electron is accelerated through a potential frequency of the radiation incident on a
difference of 250V. What is the de-Broglie photosensitive surface. State clearly, the
wavelength associated with it? To which part basic elementary process involved in
photoelectric effect.

Advent Tutorial Front of Nehru Inter College Banarsidas, Auraiya


KP Singh 7248370933 ks5122213@gmail.com Page 14
Chapter- 11 Dual Nature of Mater and radiation
6. An electron microscope used electrons
2. Light of wavelength 2500Å falls on a accelerated by a voltage of 50kV.
metal surface of work function 3.5eV. Determine the de-Broglie wavelength
What is the kinetic energy (in eV) of (i) associated with the electrons. Taking
the fastest and (ii) the slowest electrons other factors, such as numerical aperture
emitted from the surface? If the same etc, to be same, how does the resolving
light falls on another surface of work power of an electron microscope which
function 5.5eV, what will be the energy uses yellow light?
of emitted electron?
7. i. why photoelectric effect cannot be
3. Define the terms threshold frequency and explained on the basis of wave nature of
stopping potential in the study of light? Give reasons.
photoelectric emission. Explain briefly the (ii). Write the basis features of photon
reasons, why wave theory of light is not picture of electromagnetic radiation on
able to explain the observed features in which Einstein’s photoelectric equation is
photoelectric effect? based.

4. (i). describe briefly three experimentally 8. Write Einstein’s photoelectric equation


observed features in the phenomenon of and point out any two characteristic
photoelectric effect. properties of photons on which this
(ii). Discuss briefly how wave theory of equation is based.
light cannot explain these features.
9. Briefly explain the three observed
5. Write the Einstein’s photoelectric features which can be explained by this
equation and mention which important equation.
features in photoelectric effect can be
explained with the help of this equation. 10. (i). State three important properties of
The maximum kinetic energy of the photons which describe the particle
photoelectrons gets doubled when the picture of electromagnetic radiation.
wavelength of light incident on the (ii). Use Einstein’s photoelectric equation
surface changes from 𝜆1 to 𝜆2 . derive the to define the terms.
expression for the threshold wavelength a. Stopping potential
𝜆0 and work function for the metal b. Threshold frequency
surface.

Advent Tutorial Front of Nehru Inter College Banarsidas, Auraiya


KP Singh 7248370933 ks5122213@gmail.com Page 15
i.

12 ii.

iii.

iv.

v.
Atom and Nuclie

vi.

Chapter vii.

viii.

Atom as constitute of every element- Every ix. An atom consists of a massive central core in

element has characteristics atoms. Atoms of which the entire positive charge and almost the

different elements contain electrons, which are x. whole mass of the atom are concentrated. This

completely identical. Atom as whole is electrically core is called the nucleus.

neutral and therefore contains equal amount of xi. The nucleus occupied very small space as

positive and negative charges. compared to the size of the atom.

xii. The atom is surrounded by a suitable number of

Thomson’s model of an atom- An atom consists of electrons so that their total negative charge is

a sphere of positively charged matter in which the equal to the total positive charge on the nucleus

negative charge electrons are uniformly embedded and the atom as a whole is electrically neutral.

like plumbs in a pudding. This model could not xiii. The electron revolves around the nucleus orbits

explain scattering of 𝛼-particles through thin foils just as planets revolve around the sun. The

and hence discarded. centripetal force required for their revolution is

provided by the electrostatic attraction between

Rutherford’s model of an atom- Geiger and the electron and the nucleus.

Marsden performed experiments on scattering of

𝛼-particles from metal foils. A collimated beam of Demerits of Rutherford’s scattering model- This

5.5 MeV of 𝛼 particles was allowed to fall on a model could not explain the stability of the atom

2.1×10-7m thin gold foil. The scattered 𝛼- because according to classical electromagnet

particles produced scintillations on a ZnS screen, theory the electron revolving around the nucleus

which were counted at different angles 𝜃 from the must continuously radiate energy in the form of

direction of the beam. It was found that most of electromagnetic radiations and hence it should fall

the 𝛼-particles passed un-deviated through thin in to the nucleus.

foils but some of them were scattered through

very large angles. From the result of these Distance of closest approach- When an 𝛼 particle

experiments, Rutherford proposed the following of mass m and velocity v moves directly towards a

model of an atom. nucleus of atomic number Z, its distance of closest


Chapter- 12 Atom And Nuclei
plank’s constant. L= mvr = , n=1,2,3…. Where
2𝑘𝑍𝑒 2 4𝑘𝑍𝑒 2
approach is given by r0= where
𝑛ℎ
=
𝐸 𝑚𝑣 2 2𝜋

E= m𝑣 and k = =9×109. n is principle quantum number.


1 2 1
2 4𝜋Ɛ0

iii. Stationary orbits- While revolving in the

Impact parameter- It is defined as the permissible orbits, an electron does not radiate

perpendicular distance of velocity vector of 𝛼 energy. These non-radiating orbits are called

particle from the centre of nucleus from the atom. stationary orbits.

The shape of the trajectory of the 𝛼 particle iv. Frequency condition- An atom can emit or

depends upon the impact parameter b and the absorb radiation in the form of discrete energy

nature of the potential field. Rutherford deduced photons only when an electron jump from a

the following relationship between the impact higher to lower orbit or lower to a higher orbit.

parameter b and the scattering angle. b= If the E1 and E2 are energy associate with these

permitted orbits then the frequency ʋ of the


𝜃 𝜃
𝑍𝑒 2 cot 𝑍𝑒 2 cot
= .
1 2 1 2
. . 1
4𝜋Ɛ0 𝐸 4𝜋Ɛ0 m𝑣 2
2
emitted or absorbed radiation is given by, hʋ=

E2- E1.
Quantisation or discretisation- The quantisation

and discretisation of a physical quantity means


Bohr’s theory of hydrogen atom- An electron
that it cannot vary continuously to have any
having the charge –e revolves with speed v in a
arbitrary value but can change only
circular orbit of radius r round the nucleus having
discontinuously to take certain specific values. The 𝑚𝑣 2 𝑘𝑍𝑒 2
the charge +e for a circular orbit = and
energy of electrons in an atom is quantised.
𝑟 𝑟2

from the quantisation of angular momentum

L=mvr= , by solving these two equations we get


𝑛ℎ

Bohr’s atom model- This model also called


2𝜋
𝑛2 ℎ 2 𝑛2
the radius of nth orbit, rn = = 0.53 Å;
planetary model of an atom and it is based on 4𝜋2 𝑚𝑘𝑍𝑒 2 𝑍
2𝜋𝑘𝑒 2
and speed in nth orbit, vn = =
1 𝑐

following postulates. 𝑛ℎ 137 𝑛

i. Nuclear concept- An atom consist of an


Total energy of electron in nth orbit- Revolving
massive central core called nucleus around
electron have two energies kinetic and potential
which planetary electrons revolve. The
hence the total energy is equal to the sum of these
centripetal force required for their revolution is
𝑘𝑍𝑒 2 𝑘𝑍𝑒 2 𝑘𝑍𝑒 2
two energies En = K.E.+P.E. = − =−
provided by the electrostatic attraction 2𝑟 𝑟 2𝑟

minus sign indicate that electron bind with the


between the electrons and the nucleus.
nucleus. For removing it energy is required.
ii. Quantisation condition- For all possible circular
2𝜋2 𝑚𝑘 2 𝑍2 𝑒 4
putting the value of r we have En= − =
orbits allowed by the classical theory, the 𝑛2 ℎ 2
𝑍2 𝑅ℎ𝑐 𝑛2
− = −13.6 eV where R is Rydberg constant
electrons are permitted to circulate only in 𝑛2 𝑍

and equal to 1.0973×107m-1.


such orbit in which the angular momentum of

an electron is an integer multiple of h/2𝜋, h is

Advent Tutorial Front of Nehru Inter College Banarsidas, Auraiya


KP Singh 7248370933 ks5122213@gmail.com Page 2
Chapter- 12 Atom And Nuclei
Spectral lines of hydrogen atom- Whenever an of higher atoms. It could not explain the fine

electron makes a transition from a higher energy structure of the spectral lines in the spectrum of
hydrogen atom.
level n1 to lower energy level n2, the difference of

energy appears in the form of a photon of


Excitation energy- It is defined as the energy
2𝜋2 𝑚𝑘 2 𝑍2 𝑒 4
frequency ʋ given by ʋ= ] or wave
1 1
[ −
ℎ3 𝑛12 𝑛22 required to by an electron of an atom to jump
number ʋ = = R𝑍 2
1 1 1
𝜆
[
𝑛12

𝑛22
] from its ground state to any one of its excited

Different spectral series for hydrogen atoms are as states.

follow:
Ionisation energy- It is defined as the energy
i. Lyman series- n1= 1 this
required to remove an electron from an atom, or
series lies in the ultraviolet
n=∞ 0.00eV the energy required to take an electron from
region.
1 1
𝜆
=R[1 −
𝑛 2] n=6 -0.38eV ground state to the outermost orbit (n=∞).
and n=2,3,4,5…..
n=5 -0.54eV
ii. Balmer series- n1=2 this Excitation potential- It is that accelerating
n=4 -0.85eV
series lies in the visible potential which gives to a bombarding electron

n=3 -1.5eV sufficient energy to excite the target atom by


region.
1 1 1
=R[ − ]
22 𝑛2
raising one of its electrons from an inner to an
𝜆

and n=3,4,5,6…..
outer orbit.
iii. Paschen series- n1=3 this
n=2 -3.4eV
series lies in the infrared Ionisation potential- It is that accelerating
region. potential which gives to a bombarding electron
1 1 1
=R[ − ]
𝜆 32 𝑛2

and n=4,5,6,7….. sufficient energy to ionise the target atom by


knocking one of its electrons completely out of the
iv. Bracket series- n1=4 this
atom.
series lies in the infrared n=1 -13.6eV
region. =R[
1 1 1
− ]
𝜆 42 𝑛2 Nucleons- Protons and neutrons which present in
and n=5,6,7,8….. the nuclei of the atoms are called collectively
v. P-fund series- n1=5 this known as nucleons.
series lies in the infrared

region. =R[
1 1

1
] Atomic number- The no of protons present in the
𝜆 52 𝑛2
nucleus is called the atomic number. It denoted by
and n=6,7,8,9…..
Z.

Energy level diagram- It is a diagram in which the


Mass number- The total number of neutron and
energies of the different stationary states of an
proton present in the nucleus is called mass
atom are represented by parallel horizontal lines,
number of the elements. It is denoted by A.
drown according to some suitable energy scale.
Number of protons= Number of electrons in the
atoms= Atomic number of element (Z). Number of
Failure of Bohr’s model- This model is applicable
only to the hydrogen-like atoms and fails in case

Advent Tutorial Front of Nehru Inter College Banarsidas, Auraiya


KP Singh 7248370933 ks5122213@gmail.com Page 3
Chapter- 12 Atom And Nuclei
nucleons in an atom=Mass number (A), number of
1
=r0 𝐴3 where r0=1.2×10-15m or 1.2 fermi one
neutrons in the atom=A−Z. Fermi= 10-15 m.

Nuclear mass- The total mass of neutrons and Nuclear density- The density of nucleus is
protons present in the nucleus is called the nuclear independent of the size of the nucleus and is given
mass. by ρ=
3𝑚
where m is the average mass of nucleons.
4𝜋𝑟03

The nuclear density is of order of 1017kgm-3.


Nuclide- A nuclide is a specific nucleus of an atom
characterised by its atomic number Z and mass Discovery of neutrons- Neutrons were discovered
number A. it is represented as 𝐴𝑍𝑋 where X is by the Chadwick in 1932. When beryllium nuclei
chemical symbol of element and Z, A are atomic are bombarded by 𝛼 particles, highly penetrating
no and mass no of the element. radiations are emitted, which consist of neutral
particles, each having mass nearly that of proton.
Isotopes- the atoms of an element, which have These particle were called neutrons. 42𝐻𝑒 + 49𝐵𝑒 → 10𝑛
same atomic number but different in mass + 126𝐶 . A free neutron decay is spontaneously, with
number hydrogen have three isotopes Protium half-life about 1000 sec, into electron, proton,
( 11𝐻), Deuterium ( 21𝐻), Tritium( 31𝐻).and chlorine have antineutrino. → 11𝐻 + −1𝑒 +ʋ.
1 0
0𝑛
two isotopes 35
17𝐶𝑙 and 37
17𝐶𝑙

Nuclear forces- These are the strong attractive


Isobars- The atoms of different elements have forces between the protons and neutrons which
different atomic number but same in mass hold them in a tiny nucleus. These are the short
number is called isobars. 40
20𝐶𝑎 and 40
18𝐴𝑟 both have range forces which operate over a very short
mass number 40 and 37
17𝐶𝑙 and 37
16𝑆 both have mass distance of about 2-3 fm from a nucleons. The
number 37. nuclear forces does not depends on the charges of
nucleons.
Isotones- The nuclides having same number of
neutrons are called isotones. Like 37
17𝐶𝑙 and 39
19𝐾 are Mass defect- The difference between rest mass of
isotones contains 20 neutrons in the nucleus. a nucleus and the sum of the rest masses of its
constituent nucleons is called its mass defect. It is
Isomers- These are the nuclei with same atomic given by ∆𝑚=Z𝑚𝑝 + (A-Z)𝑚𝑛 −m( 𝐴𝑍𝑋 ).
number and same mass number but in different
energy states. Binding energy- It may be defined as the energy
required to break up a nucleus into its constituent
Atomic mass unit (amu)- It is defined as of
1 th
12 protons and neutrons and separate them to such a
mass of one 12
6𝐶 atom. Its value is given by 1amu= large distance that they may not interact with
1.660565×10-27kg. each other. The binding energy of a nucleus 𝐴𝑍𝑋 is
given by ∆𝐸b =[Z𝑚𝑝 + (A − Z)𝑚𝑛 − m( 𝐴𝑍𝑋 )]c2.
Size of nucleus- It is found that radius of nucleus is
proportional to cube root of its mass number. r Binding energy per nucleons- It is the average
energy required to extract one nucleons from the

Advent Tutorial Front of Nehru Inter College Banarsidas, Auraiya


KP Singh 7248370933 ks5122213@gmail.com Page 4
Chapter- 12 Atom And Nuclei
nucleus. It is obtained by dividing binding energy 𝐴
𝑍𝑋 → 𝑍−2𝑌 + 2𝐻𝑒 +Q.
𝐴−4 4

of nucleus by its mass number.


𝐵.𝐸.
=
[Z𝑚𝑝 + (A−Z)𝑚𝑛 −m(𝐴
; in mass number range
𝑍𝑋 )]c2 Beta decay- It is the process of emission of
𝐴 𝐴
30<A<170, the binding energy per nucleons electron or a positron from a radioactive nucleus.
nearly constant, about 8MeV per nucleons. 𝑍+1𝑌 +𝛽 +ʋ (𝛽 − 𝑑𝑒𝑐𝑎𝑦);
𝐴 𝐴 −
𝑍𝑋 →
𝑍−1𝑌 +𝛽 +ʋ (𝛽 + 𝑑𝑒𝑐𝑎𝑦);
𝐴 𝐴 +
𝑍𝑋 →
Packing fraction- Mass defect per nucleons is Where ʋ ,ʋ are the antineutrino or neutrino.
defined as the packing fraction, it is directly
related to the availability of nuclear energy and Gamma decay- It is the process of emission of a
stability of the nucleus. 𝛾-rays photons during the radioactive
disintegration of a nucleus. This occurs when a
Nuclear energy levels- Like an atom, the nucleus nucleus in an excited state makes a transition to a
exists in nucleonic configurations, which state of lower energy
correspond to nuclear stationary states. The
𝐴 𝐴
𝑍 𝑋 (𝑒𝑥𝑐𝑖𝑡𝑒𝑑 𝑠𝑡𝑎𝑡𝑒) → 𝑍𝑋 (𝑔𝑟𝑜𝑢𝑛𝑑 𝑠𝑡𝑎𝑡𝑒)+ 𝛾-rays.
stationary state of lower energy is called the
ground state. When a nucleus makes a transition Radioactive decay laws- The number of atoms of a
from one energy level to a level of lower energy, radioactive sample disintegrating per second is
the emitted photon belongs to the gamma directly proportional to the number of undecayed
radioactive nuclei present at that instant. =−𝜆N
𝑑𝑁
region (𝛾) of the electromagnetic spectrum. 𝑑𝑡
the law may also expressed as N=N0 𝑒 −𝜆𝑡 where N0 is
Radioactivity- It is the phenomenon of the number of nuclei at t=0 and 𝜆 is decay
spontaneously disintegration of nucleus of an atom constant.
with the emission of one or more radiations like
𝛼-particle, 𝛽- particle, 𝛾-rays. These substances Decay constant- It may be defined as the
which spontaneously emit these penetrating reciprocal of time interval in which the number of
radiations are called radioactive substances. active nuclei in a given radioactive sample reduced
to 36.8% (or 1/e times) of its initial value. Its
Soddy-Fajan’s displacement law- It states that units are s-1.
i. When a radioactive nucleus emits an 𝛼-particle,
then its atomic number decrease by 2 and Half-life- The half-life of radioactive sample is the
mass number decrease by 4. time interval in which one-half of the initial
number of nuclei disintegrates. T1/2= ;
0.693
ii. When a radioactive nucleus emits 𝛽-particle, its 𝜆
1 𝑛
atomic number increase by 1 but mass number N=N0( ) where n is the no of half-lives in time
2
remains same. (T) = .
𝑇
𝑇1⁄
2
iii. The emission of 𝛾-rays, does not change mass
number and atomic number of the radioactive Mean-life- It may be defined as the ratio of the
nucleus. combined age of all the atoms to the total number
𝑇1⁄
of atoms present in the given sample. 𝜏= = =
1 2

Alpha decay- It is the process of emission of an 𝛼-


𝜆 0.693
1.44 𝑇1⁄ . Its unit is second.
particle from the radioactive nucleus.
2

Advent Tutorial Front of Nehru Inter College Banarsidas, Auraiya


KP Singh 7248370933 ks5122213@gmail.com Page 5
Chapter- 12 Atom And Nuclei
Decay rate or activity of the sample- It is defined moderators are water, heavy water (D2O) and
as the number of radioactive disintegrations taking graphite.
place per second in a given sample. R=|
𝑑𝑁
|= 𝜆N
𝑑𝑡
= 𝜆𝑁0 𝑒 −𝜆𝑡 or R=R0𝑒 −𝜆𝑡 where R0 is the initial Nuclear reactor- It is a device, in which a nuclear
activity of the sample. chain reaction is initiated, maintained and
controlled. The reaction is controlled by using
Becquerel- It is the SI unit of activity and is neutrons absorbing materials like cadmium rods
defined as the decay rate of 1 disintegration per called control rods.
second.
One Becquerel=1 decay per second. One currie= Nuclear fusion- It is the process of fusion of two
3.7×1010 disintegration per sec or bq. smaller nuclei into a heavier nucleus with the
Rutherford=106bq. liberation of large amount of energy. For example,

1𝐻 + 1𝐻 → 42𝐻𝑒+ 24 MeV. These reactions require


2 2

Natural radioactivity- It is the phenomenon of extreme condition of temperature and pressure so


spontaneous emission of 𝛼-, 𝛽- or 𝛾- radiations that the reacting nuclei can overcome their
from nuclei of naturally occurring isotopes. electrostatic repulsion. For this reason these
reactions are called thermonuclear reactions.
Artificial or induced radioactivity- It is the
phenomenon of inducing radioactivity in a certain Chain reaction- These are the nuclear reaction in
stable nuclei by bombarding them by suitable high which the product itself induce the forward
energy particles. reaction.

Nuclear reactions- It is a reaction in which


involves the change of nucleus of one element into
Atoms
the nucleus of another element, by bombarding Very Short Answer/Objective Type Questions [1

the former with suitable high energy particles. Mark)


1. If 13.6 eV energy is required to ionise the

Nuclear fission- It is process in which a heavy hydrogen atom, then energy required to

nucleus (A>230) when excited gets split up into remove an electron from n = 2 is

two smaller nuclei of nearly comparable masses. (a) 10.2 eV (b) 0 eV

For example, 235


92𝑈 + 10𝑛 → 56𝐵𝑎 + 36𝐾𝑟+3 0𝑛+Q(energy).
141 92 1 (c) 3.4 eV (d) 6.8 eV Ans (c)
2. In Bohr's model of an atom which of the
following is an integral multiple of ?

Thermal neutrons- These are the slow moving 2𝜋

neutrons of energy 0.0253eV corresponding to (a) Kinetic energy (b) Radius of an atom
the velocities of (c) Potential energy (d) Angular momentum
2200ms-1. Ans. (d)
3. The transition from the state n = 5 to n = 1 in
Moderator- Any substance which is used in to slow a hydrogen atom results in UV radiation.
down fast moving neutron to thermal energies is Infrared radiation will be obtained in the
called a moderator. The commonly used transition

Advent Tutorial Front of Nehru Inter College Banarsidas, Auraiya


KP Singh 7248370933 ks5122213@gmail.com Page 6
Chapter- 12 Atom And Nuclei
(a) 2 → 1 (b) 3 → 2 12. The ionisation energy of hydrogen atom is 13.6
(c) 4 → 3 (d) 6→2 Ans. (c) eV. Following Bohr's theory the energy
4. In Bohr's model, the atomic radius of the first corresponding to a transition between 3rd and
orbit is ro Then, the radius of the third orbit is 4th orbits is
(a) r0/9 (b) r0 (a) 3.40 eV (b) 1.51 eV
(c) 9r0 (d) 3r0 Ans. (c) (c) 0.85 eV (d) 0.66 eV Ans. (d)
5. The K.E. of the electron in an orbit of radius r 13. The energy of hydrogen atom in the nth orbit is
in hydrogen atom is proportional to En, then the energy in the nth orbit of single
ionised helium atom is
𝑒2 𝑒2
(a) (b)
𝑟 2𝑟
(a) (b) 2𝐸𝑛
2𝑒 2 𝑒2 𝐸𝑛
(c) (d) Ans. (b) 2
𝑟 3𝑟
(c) 4𝐸𝑛 (d) Ans. (c)
𝐸𝑛
6. The hydrogen atom can give spectral lines in 4
the Lyman, Balmer and Paschen series. Which 14. The spectral lines in the Brackett series arise
of the following statement is correct? due to transition of electron in hydrogen atom
(a) Lyman series is in the infrared region. from higher orbits to the orbit with
(b) Balmer series is in the visible region. (a) n=1 (b) n=2
(c) Paschen series is in the visible region. (c) n=3 (d) n= 4 Ans (d)
(d) Balmer series is in the ultraviolet region. 15. On moving up in the energy states of a H-like
Ans. (b) atom, the energy difference between two
7. The ratio between Bohr radii in K, L, M shell is consecutive energy states
(a) 1:2:3 (b) 2:4:6 (a) decreases. (b) increases.
(c) 1:4:9 (d) 1:3:5 Ans. (c) (c) first decreases then increases.
8. Which of the relation is correct between time (d) first increases then decreases. Ans. (a)
period and number of orbits while an electron 16. The transition of electron from n = 4, 5, 6, ...
is resolving in an orbit? to n=3 corresponds to
(a) 𝑇 ∝
1
(b) 𝑇 ∝ 𝑛2 (a) Lyman series (b) Balmer series
𝑛2
(c) 𝑇 ∝ 𝑛3 (d) 𝑇 ∝
1
Ans. (c) (c) Paschen series (d) Brackett series
𝑛
9. Energy of an electron in the second orbit of Ans. (c)
hydrogen atom is E and the energy of electron 17. As per Bohr model, the minimum energy (in
in 3rd orbit of He will be eV) required to remove an electron from the
(a) 𝐸3 =
16𝐸
(b) 𝐸3 =
16𝐸
ground state of double ionized Li atom (Z = 3)
3 9
(c) 𝐸3 =
4𝐸
(d) 𝐸3 =
4𝐸
Ans. (b) is
9 3
10. The longest wavelength in Balmer series of (a) 1.51 eV (b) 13.6 eV
hydrogen spectrum will be (c) 40.8 eV (d) 122.4 eV Ans. (d)
(a) 6557 Å (b) 1216 Å 18. Which of the following spectral series in
(c) 4800 Å (d) 5600 Å Ans. (a) hydrogen atom gives spectral line of 4860 Å?
11. In terms of Rydberg constant R, the wave (a) Lyman (b) Balmer
number of the first Balmer line is (c) Paschen (d) Brackett Ans. (b)
(a) R (b) 3R 19. In terms of Rydberg constant R, the shortest
(c) 5R (d) 8R Ans. (c) Wavelength in Balmer series of hydrogen atom
spectrum will have wavelength

Advent Tutorial Front of Nehru Inter College Banarsidas, Auraiya


KP Singh 7248370933 ks5122213@gmail.com Page 7
Chapter- 12 Atom And Nuclei
(a) (b) (b) is as important as electrostatic force for
1 4
𝑅 𝑅
(c) (d) Ans. (b) binding the two atoms.
3 9
2𝑅 𝑅
20. Taking the Bohr radius as a0 = 53 pm, the (c) cancels the repulsive electrostatic force
radius of Li++ ion in its ground state, on the between the nuclei.
basis of Bohr's model, will be about [NCERT (d) is not important because oxygen nucleus has
Exemplar] equal number of neutrons and protons. Ans. (a)
(a) 53 pm (b) 27 pm 24. Two H atoms in the ground state collide in-
(c) 18 pm (d) 13 pm Ans. (c) elastically. The maximum amount by which
21. The simple Bohr model cannot be directly their combined kinetic energy is reduced is
applied to calculate the energy levels of an [NCERT Exemplar]
atom with many electrons. This is because (a) 10.20 eV (b) 20.40 eV
[NCERT Exemplar] (c) 13.6 eV (d) 27.2 ev Ans. (a)
(a) of the electrons not being subject to a 25. A set of atoms in an excited state decays,
central force. [NCERT Exemplar]
(b) of the electrons colliding with each other. (a) in general to any of the states with lower energy

(c) of screening effects. (b) into a lower state only when excited by an
external electric field.
(d) the force between the nucleus and an
(c) all together simultaneously into a lower state.
electron will no longer be given by Coulomb's
(d) to emit photons only when they collide. Ans. (a)
law. Ans. (a)
26. An ionised H-molecule consists of an electron
22. For the ground state, the electron in the H-
and two protons. The protons are separated by
atom has an angular momentum = ħ,
a small distance of the order of angstrom. In
according to the simple Bohr model. Angular
the ground state, [NCERT Exemplar]
momentum is a vector and hence there will be
(a) the electron would move in circular orbits.
infinitely many orbits with the vector pointing (b) the energy would be (2)4 times that of a H-atom.
in all possible directions. In actuality, this is not (c) the electrons, orbit would go around the protons.
true, [NCERT Exemplar] (d) the molecule will soon decay in a proton and a
(a) Because Bohr model gives incorrect values of H-atom. Ans. (c)

angular momentum. 27. The Balmer series for the H-atom can be
(b) Because only one of these would have a observed
minimum energy. (a) if we measure the frequencies of light
(c) Angular momentum must be in the emitted when an excited atom falls to the
direction of spin of electron. ground state.
(d) Because electrons go around only in (b) if we measure the frequencies of light
horizontal orbits. Ans. (a) emitted due to transitions between excited
23. O2 molecule consists of two oxygen atoms. In states and the first excited state.
the molecule, nuclear force between the nuclei (c) in any transition in a H-atom.
of the two atoms [NCERT Exemplar] (d) as a sequence of frequencies with the lower
(a) is not important because nuclear forces are frequencies getting closely packed. Ans. (b)
short-ranged.

Advent Tutorial Front of Nehru Inter College Banarsidas, Auraiya


KP Singh 7248370933 ks5122213@gmail.com Page 8
Chapter- 12 Atom And Nuclei
1 𝑚𝑒 4
28. Let 𝐸𝑛 = − be the energy of the nth level As q is half with a proton in comparison to α-
8𝜖02 𝑛2 ℎ2

of H-atom. If all the H-atoms are in the particle, for same d, energy E has to be made

ground state and radiation of frequency half.


(𝐸2 − 𝐸1 )/ℎ, falls on it, 37. What is the ratio of radii of the orbits

(a) It will not be absorbed at all corresponding to first excited state and ground

(b) Some of atoms will move to the first excited state in a hydrogen atom?
Ans. As 𝑟 ∝ 𝑛2 ;
𝑟1 4
state. 𝑟2
=
1

(c) All atoms will be excited to the n = 2 state. 38. Write the expression for Bohr's radius in
(d) All atoms will make a transition to the n = hydrogen atom.
3 state. Ans. (d) Ans. Bohr's radius in hydrogen atom,
29. Non-radiating electron orbits in an atom are 𝜖0 ℎ 2
𝑟= ;𝑛 = 1
𝜋𝑚𝑒 2
called _______________ orbits. Ans. stationary
39. Find the ratio of energies of photons produced
30. At distance of closest approach, kinetic energy
due to transition of an electron of hydrogen
of α-particle is ______________ . Ans. zero
atom from its
31. The centripetal force required for revolution of
(i). Second permitted energy level to the first
electron in an orbit is provided by ______________
level, and
between the electron and the nucleus.
(ii) the highest permitted energy level to the
Ans. electrostatic attraction
first permitted level.
32. ______________ is the perpendicular distance of the
Ans. - 10.2 51
velocity vector of the a-particle from the ∆𝐸1 10.2 51
= =
centre of the nucleus. Ans. Impact parameter ∆𝐸2 13.6 68
40. What is the distance of closest approach?
33. The number of waves, contained in unit length
Ans. The minimum distance up to which an
of the medium is called _____________.
energetic particle travelling directly towards a
Ans. wave number
nucleus can reach.
34. Angular momentum and energy of an electron
41. An electron in a hydrogen atom is revolving
in an atom is ______________ . Ans. quantised.
round a positively charged nucleus. Which two
35. Number of possible spectral lines emitted on
physical quantities explain the orbit of an
de-excitation of electron from energy level n to
electron?
ground state is equal to _____________.
Ans. Two physical quantities are: (i) angular
Ans.
𝒏(𝒏−𝟏)
𝟐
momentum, and
36. In the Rutherford scattering experiment, the
(ii) total energy of electron.
distance of closest approach for an α-particle is
42. What will happen if an electron instead of
d0. If an α-particle is replaced by a proton,
revolving becomes stationary in H-atom?
how much kinetic energy in comparison to α-
Ans. Then the electrostatic field of the nucleus
particle will it require to have the same
will attract the electron into the nucleus itself.
distance of closest approach d0?
43. Calculate the speed of electron revolving
Ans. At distance of closest approach, the KE.
around the nucleus of a hydrogen atom in
with the charged particle is converted into
order that it may not be pulled into the
electrostatic P.E.
𝑘(𝑍𝑒)𝑞 1
= 𝑚𝑝 𝑣 2 𝑜𝑟 𝐸
𝑑0 2
nucleus by electrostatic attraction.

Advent Tutorial Front of Nehru Inter College Banarsidas, Auraiya


KP Singh 7248370933 ks5122213@gmail.com Page 9
Chapter- 12 Atom And Nuclei
Ans. It is only possible when the 13.6
𝐸 = − 2 𝑒𝑉
𝑛
centripetal force is equal to electrostatic 13.6
𝑚𝑣 2 𝑒2
−3.4 = 2 ⇒ 𝑛2 = 4 ∴ 𝑛 = 2
force of attraction. i.e. when
1 𝑛
=
𝑟 4𝜋𝜖𝑜 𝑟 2
Angular momentum of an electron in nth orbit,
Putting n = 2, we get 𝐿 =
𝑛ℎ 2ℎ ℎ
𝐿= =
Short Answer Type Questions [2 Marks]
2𝜋 2𝜋 𝜋
5. What is the difference between Rutherford and
1. Define the distance of closest approach. An a-
Bohr's model?
particle of kinetic energy 'K’ is bombarded on a
Ans. According to the Rutherford model,
thin gold foil. The distance of the closest
electrons are revolving round the nucleus in
approach is 'r'. What will be the distance of
different orbits. These revolving electrons have
closest approach for an α-particle of double the
some acceleration, so this will radiate energy in
kinetic energy? [Delhi 2017]
the form of EM wave, hence the orbits of
Ans. The minimum distance up to which an
electrons will go on decreasing and finally they
energetic α-particle travelling directly towards
will fall into the nucleus.
a nucleus can reach. The relation between
According to the Bohr model, electrons can
closest approach and kinetic energy of α-
revolve only in definite orbits without radiating
particle is given by
2𝑍𝑒 2
any energy, where the angular momentum of
or 𝑑0 ∝
1
𝑑0 =
4𝜋𝜖0 𝐾 𝐾 an electron is an integral multiple of ℎ/2𝜋.
So, when kinetic energy is doubled, the distance
6. Write two important limitations of Rutherford
of closest approach will reduce to half.
nuclear model of the atom. [Delhi 2017]
2. A proton strikes another proton at rest.
Ans. Limitations of Rutherford's atomic model:
Assume impact-parameter to be zero, i.e.
(i) This model cannot explain the stability of an
head-on collision. How close will the incident
atom, as according to electromagnetic theory
proton go to other proton?
revolving electron should continuously radiate
Ans. ∵ Change in kinetic energy of proton =
energy and move in orbits of gradually
Change in potential energy of the two proton
decreasing radii and finally it should collapse
𝑒2 𝑒2
system
1 1
𝑚𝑣 2 = . ; 𝑟= into the nucleus.
2 4𝜋𝜖0 𝑟 2𝜋𝜖0 𝑚𝑣 2
3. Find the ionization energy and ionization (ii) As electron can revolve in orbits of all
potential of ground state electron in hydrogen possible radii, so it should emit a continuous
atom. spectrum. But, spectrum of hydrogen atom is
Ans. Ionization energy is the energy required to always a discrete line spectrum.
free an electron from the ground state to 7. Draw a labelled diagram of Geiger and
infinity 𝐸∞ − 𝐸1 = 0 − (− ) = 13.6 𝑒𝑉, and the
13.6
𝑛2 Marsden experiment on a-particle scattering.
ionisation potential will be 13.6 V. [∵ Energy = Explain how does it help to find the size of a
qV, 13.6] nucleus.
Ionisation potential V = 13.6 Volt Ans. In this experiment, the energised a-
4. What is the angular momentum of an electron particles were bombarded on gold foil, K.E. of
in Bohr's hydrogen atom whose energy is -3.4 α-particle is changed into P.E. of α-particle
eV? and a gold nucleus, due to nuclear repulsion.
Ans. The energy of electron in nth Bohr orbit of During this scattering process, mechanical
hydrogen atom is given by energy of the system remains constant.

Advent Tutorial Front of Nehru Inter College Banarsidas, Auraiya


KP Singh 7248370933 ks5122213@gmail.com Page 10
Chapter- 12 Atom And Nuclei
(K.E)inital - (PE)final Ans. The potential energy of an electron
revolving in an orbit of radius r in hydrogen
2𝑍𝑒 2
or 𝑟0 =
1 (2𝑒)(𝑍𝑒) 1
𝐾𝑎 = . .
4𝜋𝜖0 𝑟0 4𝜋𝜖0 𝐾𝛼
𝑒2
atom is given by 𝑃. 𝐸. = −
1
∴ By knowing ro, the size of the nucleus can be 4𝜋𝜖0 𝑟
The kinetic energy of the electron is given by
1
𝐾. 𝐸. = 𝑚𝑣 2
2
𝑚𝑣 2 1 𝑒2
=
𝑟 4𝜋𝜖0 𝑟 2
(∵ Centripetal force for revolution of electron is
provided by electrostatic force between nucleus
and electron.)
1 1 𝑒2
∴ 𝐾. 𝐸. = ( )
2 4𝜋𝜖0 𝑟
8. State Bohr's postulate of hydrogen atom which Total energy 𝐸𝑛 = 𝐾. 𝐸. +𝑃. 𝐸.
successfully explains the emission lines in the 1 𝑒2
∴ 𝐸𝑛 = −
4𝜋𝜖0 𝑟
spectrum of hydrogen atom. Use Rydberg
The negative sign signifies that the electron is
formula to determine the wavelength of Hα
bound to the nucleus of the atom. The binding
line. [Given: Rydberg constant R = 1.09x1011
energy is
m] [AI 2015]
1 𝑒2
Ans. According to the Bohr's third postulate, 𝐵. 𝐸. = +
4𝜋𝜖0 𝑟
an electron might make a transition from one
of its specified non- radiating orbits to another 10. Find the relation between the three
of lower energy. When it does so, a photon is wavelengths and, from the energy level
emitted having energy equal to the energy diagram shown below. C
difference between the initial and final states. As 𝑬𝑪𝑨 = 𝑬𝑪𝑩 + 𝑬𝑩𝑨 𝜆1
The frequency of the emitted photon is given by Ans. 𝐸𝐶𝐴 = 𝐸𝐶𝐵 + 𝐸𝐵𝐴 B
ℎʋ = 𝐸𝑖 − 𝐸𝑓 ℎ𝑐 ℎ𝑐 ℎ𝑐
= + 𝜆2 𝜆3
𝜆3 𝜆1 𝜆2
Where 𝐸𝑖 and 𝐸𝑓 are the energies of initial and 1 𝜆1 +𝜆2 𝜆1 𝜆2
= ⇒ 𝜆3 =
𝜆3 𝜆1 𝜆2 𝜆1 +𝜆2 A
final states (𝐸𝑖 > 𝐸𝑓 )
Wavelength of Hα, line is given by
11. Define ionization energy. How would the
1 1 1
= 1.09 × 107 [ 2 − 2 ] ionization energy change when electron in a
𝜆 2 3
1 1 1 5
= 1.09 × 107 [ − ] = 1.09 × 107 × hydrogen atom replaced by a particle of mass
𝜆 4 9 36
36 × 10−7 36 × 10−7 200 times that of the electron but having the
𝜆= =
5 × 1.03 5.15 same charge? [AI 2016]
𝜆 = 6990 × 10−10 = 6990 Å
9. Using Rutherford model of the atom, derive the Ans. The minimum energy required to emit the

expression for the total energy of the electron electron from the ground state of the atom is

in hydrogen atom. What is the significance of called ionization energy.


𝑚𝑒 4
total negative energy possessed by the electron? Ionization energy, 𝐸𝑛 =
8𝑛2 𝜖02 ℎ 2

[AI 2014] Therefore, the ionization energy of a particle


will become 200 times, the ionization energy
required by an electron.

Advent Tutorial Front of Nehru Inter College Banarsidas, Auraiya


KP Singh 7248370933 ks5122213@gmail.com Page 11
Chapter- 12 Atom And Nuclei
Long Answer Type [I] Questions [3 Marks] draw the energy level diagram showing how
1. In the study of Geiger-Marsden experiment on the line spectra corresponding to Balmer series
scattering of a-particles by a thin foil of gold, occur due to transition between energy levels.
draw the trajectory of α-particles in the [Delhi 2013]
coulomb field of target nucleus. Explain briefly 7. Using the postulates of Bohr's model of
how one gets the information on the size of the hydrogen atom, obtain an expression for the
nucleus from this study. From the relation R = frequency of radiation emitted when the atom
R0A1/3, where R0 is constant and A is the mass makes a transition from the higher energy
number of the nucleus, show that nuclear state with quantum number ni to the lower
matter density is independent of A. [Delhi energy state with quantum number nf (nf < ni).
2015] 8. (a) State Bohr's postulate to define stable orbits
2. In an experiment on a-particle scattering by a in hydrogen atom. How does de Broglie's
thin foil of gold, draw a plot showing the hypothesis explain the stability of these orbits?
number of particles scattered versus the [Chennai 2019]
scattering angle θ. (b) A hydrogen atom initially in the ground
Why is it that a very small fraction of the state absorbs a photon which excites it to the n
particles are scattered at θ > 90°? = 4 level. Estimate the frequency of the photon.
Write two important conclusions that can be [CBSE 2018]
drawn regarding the structure of the atom 9. (a) Using Bohr's second postulate of
from the study of this experiment. [Foreign quantization of orbital angular momentum
2013] show that the circumference of the electronic
3. Using Bohr's postulates, derive the expression in the nth orbital state in hydrogen atom is n
for the orbital period of the electron moving in times the de Broglie wavelength associated with
the orbit of hydrogen atom. [Dehradun 2019, it.
Foreign 2017] (b) The electron in hydrogen atom is initially in
4. Consider energy level diagram of a hydrogen the third excited state. What is the maximum
atom. How will the kinetic energy and potential number of spectral lines which can be emitted
energy of electron vary if the electron moves when it finally moves to the ground state?
from a lower level to a higher level? [Delhi 2012]
5. Using Bohr's postulates for hydrogen atom,
show that the total energy (E) of the electron Long Answer Type [II] Questions [5 Marks]
in the stationary states can be expressed as the 1. (a) Using Bohr's postulates, derive the
sum of kinetic energy (K) and potential energy expression for the total energy of the electron
(U), where 𝐾 = − 𝑈. Hence deduce the in the stationary states of the hydrogen atom.
1
2
expression for the total energy in the energy (b) Using Rydberg formula, calculate the
level of hydrogen atom. [Similar Dehradun wavelengths of the spectral lines of the first
2019, Foreign 2012] member of the Lyman series and of the Balmer
6. Using Bohr's postulates, obtain the expression series. [Foreign 2014]
for the total energy of the electron in the 2. Using Bohr's postulates, derive the expression
stationary states of the hydrogen atom. Hence for the frequency of radiation emitted when

Advent Tutorial Front of Nehru Inter College Banarsidas, Auraiya


KP Singh 7248370933 ks5122213@gmail.com Page 12
Chapter- 12 Atom And Nuclei
electron in hydrogen atom undergoes transition 11. If 𝜆1 and 𝜆2 are the wavelengths of the first
from higher energy state (quantum number ni) member of Lyman and Paschen series
to the lower state (nf). When electron in respectively, then determine the ratio 𝜆1/𝜆2.
hydrogen atom jumps from energy state ni = 4 12. A 12.5 eV electron beam is used to bombard
to nf = 3, 2, 1, identify the spectral series to gaseous hydrogen at room temperature. Upto
which the emission lines belong. which energy level the hydrogen atoms would
[AI 2013] be excited?
Calculate the wavelengths of the first member
Numerical Problems of Lyman and first member of Balmer series.
3. The radius of innermost electron orbit of a [Delhi 2014, AI 2017]
hydrogen atom is 5.3x107 m. What is the 13. The electron in a given Bohr's orbit has a total
radius of orbit in the second excited state? energy of -1.5 eV. Calculate its (i) kinetic
4. Find out the wavelength of the electron energy,
orbiting in the ground state of hydrogen atom. (ii) potential energy and (iii) wavelength of
[Delhi 2017] radiation emitted, when this electron makes a
5. Find the wavelength of the electron orbiting in transition to the ground state.
the first excited state in hydrogen atom. [Delhi [Given: Energy in the ground state =- 13.6 eV
2017] and Rydberg's constant = 1.09x107 m-1] [Delhi
6. A single electron orbits around a stationary 2011C]
nucleus of charge +Ze, where Z is a constant 14. The short wavelength limit for the Lyman
and e is the magnitude of electronic charge. It series of the hydrogen spectrum is 913.4 Å.
requires 47.2 eV to excite the electron from Calculate the short wavelength limit for Balmer
the second to the third Bohr orbit. Find the series of the hydrogen spectrum. [AI 2017]
value of Z. 15. The ground state energy of hydrogen atom is -
7. Determine the maximum wavelength that 13.6 eV.
hydrogen in its ground state can absorb. What (i) What is the potential energy of an electron
would be the next smaller wavelength that in the 3 excited state?
would work? (ii) If the electron jumps to the ground state
8. Find the ratio of minimum to maximum from the 3 excited state, calculate the
wavelength of radiation emitted by an electron wavelength of the photon emitted.
in the ground state of Bohr's hydrogen atom. 16. the ground state energy of hydrogen atom is -
9. The second member of the Lyman series in 13.6 eV. If the electron makes a transition
hydrogen spectrum has wavelength 5400 Å. from an energy level -0.85 eV to -3.4 eV,
Find the wavelength of the first member. calculate the wavelength of the spectral line
10. Determine the ratio of distance of closest emitted. To which series of hydrogen spectrum
approach of a proton and an alpha particle, does this wavelength belong? [AI 2017, 12]
incident on a thin gold foil, if they are 17. The energy levels of an element are given
accelerated through same potential difference below:
(V)

Advent Tutorial Front of Nehru Inter College Banarsidas, Auraiya


KP Singh 7248370933 ks5122213@gmail.com Page 13
Chapter- 12 Atom And Nuclei
- 0.85 eV
A 9. What is the impact parameter for scattering of 𝛼-
-1.5 eV
B C particle by 180°?
-3.4eV

𝜆2 D Short Answer type Questions (2 Marks)


10. Show that the radius of orbit in hydrogen atom
-13.6 eV
varies as 𝑛2 . Where n is the principle quantum
Identify, using necessary calculations, the number of atom.
transition, which corresponds to the emission of 11. Calculate the shortest wavelength in the Balmer
a spectral line of wavelength 482 nm. series of hydrogen atom. In which region (infrared,
18. The ground state energy of hydrogen atom is - visible, ultraviolet) of hydrogen spectrum does this
13.6 eV. wavelength lie?
(i) What is the kinetic energy of an electron in 12. When an electron in hydrogen atom jumps from
the 2nd excited state? the third excited state to ground state, how would
(ii) If the electron jumps to the ground state be de-Broglie wavelength associated with the
from the 2nd excited state, calculate the electron change? Justify your answer.
wavelength of the spectral line emitted. 13. Using Rutherford model of the atom, derive the
expression for total energy of electron in H-atom.
What is the significance of total negative energy
Very Short Answer type Questions (1 Marks) possessed by the electron?
1. When is 𝐻𝛼 -line of the Balmer series in the 14. A 12.5eV electron beam is used to bombard
emission spectrum of H-atom obtained? gaseous hydrogen at room temperature. Up to
2. Why is the classical (Rutherford) model for an which energy level the hydrogen atom would be
atom of electron orbiting around the nucleus not excited? Calculate the wavelengths of the first
able to explain the atomic structure? member of Lyman and first member of Balmer
3. In 𝛼-particle experiment, most of the 𝛼-particles series.
passed through the gold foil without any 15. Using Bohr’s postulates of the atomic model,
deflection. Why? derive the expression for radius of nth electron
4. Name the spectral series of H-atom lying in the orbit, obtain the expression for Bohr’s radius.
infrared region. 16. In the ground state of hydrogen atom, its Bohr’s
5. Give the expression for velocity of electron in radius is given as 5.3×10-11 m. the atom is
Bohr’s orbit. excited such that the radius becomes 21.2× 10-
6. The energy of an electron in an excited H-atom is 11m. Find (i) the value of the principal quantum
-3.4eV. Calculate the angular momentum of the number and (ii) the total energy of the atom in
electron according to bohr’s theory. Plank’s this excited state.
constant, h=6.6236×10-34 j-sec. 17. (i) In H-atom, an electron undergoes transition
7. What is the ratio of mass of an 𝛼-particle to that from second excited state to the first excited state
of an electron? and then to the ground state. Identify the spectral
8. Calculate the radius of the first orbit of H-atom. series to which these transitions belong.
Show that the velocity of electron in the first orbit (ii). Find out the ratio of the wavelengths of the
is 1/137 times the velocity of light. emitted radiations in the two cases.

Advent Tutorial Front of Nehru Inter College Banarsidas, Auraiya


KP Singh 7248370933 ks5122213@gmail.com Page 14
Chapter- 12 Atom And Nuclei
18. How does speed of electron vary with change in How will the distance of closest approach be
quantum number n in H-atom? affected when the kinetic energy of the 𝛼-particle
19. Give the observations and conclusion of the is doubled?
Rutherford’s 𝛼- particle scattering experiment. 26. Draw the energy level diagram for the hydrogen
20. Derive the expression for the wavelength of the H- atom.
atom for different spectral series. 27. What is the difference between emission spectra
21. Explain, why scattering of 𝛼-particles in and absorption spectra?
Rutherford’s experiment is not affected by the 28. Explain the origin of spectral lines of hydrogen
mass of the nucleous? using Bohr’s theory.
29. What does emperical formulae means? Hence
Short Answer type Questions (3 Marks) explain that how Balmer proposed this formula?
22. Using Bohr’s postulates, obtain the total expression 30. How did de-Broglie equation lead to the
for the energy of the electron in the stationary quantisation condition laid laid down by Bohr?
states of the H-atom. 31. Explain the significance of negative energy of an
Hence, draw the energy level diagram showing electron in an orbit. What is the energy possessed
how the line spectra corresponding to Balmer by an electron for n=∞?
series occur due to transition between energy 32. The total energy of an electron in the first excited
levels. state of the hydrogen atom is about -3.4 eV.
23. Using Bohr’s postulates for H-atom, show that the a. What is the kinetic energy of the electron in
total energy (E) of the electron in the stationary this state?
states can be expressed as the sum of kinetic b. What is the potential energy of the electron in
energy (K) and potential energy (U), where 2K=- this state?
U. c. Which of the answers above would change if the
Hence, deduce the expression for the total energy choice of the zero of potential energy is
in the nth energy level in H-atom. changed?
24. (i) Using Bohr’s second postulate of quantisation of 33. If Bohr's quantisation postulate (angular
orbiting angular momentum, show that the momentum = nh/2π) is a basic law of nature, it
circumference of the electron in the nth orbital should be equally valid for the case of planetary
state in H-atom is n-times the de-Broglie motion also. Why then do we never speak of
wavelength associated with it. quantisation of orbits of planets around the sun?
(ii). The electron in H-atom is initially in the third
excited state. What is the maximum number of
spectral lines which can be emitted when it finally Nuclei
moves to the ground state?
Very Short Answer/Objective Type Questions [1 Mark]
25. In a Geiger-Marsden experiment, calculate the
1. Suppose we consider a large number of
distance of closest approach to the nucleus of
containers each containing initially 10000
Z=80, when an 𝛼-particle of 8 MeV energy
atoms of a radioactive material with a half-life
impinges on it before it comes to momentarily rest
of 1 year. After 1 year, [NCERT Exemplar]
and reverses its direction.
(a) all the containers will have 5000 atoms of
the material.

Advent Tutorial Front of Nehru Inter College Banarsidas, Auraiya


KP Singh 7248370933 ks5122213@gmail.com Page 15
Chapter- 12 Atom And Nuclei
(b) all the containers will contain the same (d) Q1 = (Mx - My-2me)c2 and Q2 = (Mx-My+
number of atoms of the material but that Ans. (a)
number will only be approximately 5000. 5. Tritium is an isotope of hydrogen whose nucleus
(c) the containers will in general have different Triton contains 2 neutrons and 1 proton. Free
numbers of the atoms of the material but their neutrons decay into 𝑝 + 𝑒 + ʋ. If one of the
average will be close to 5000. neutrons in Triton decays, it would transform
(d) none of the containers can have more than into He3 nucleus. This does not happen. This is
5000 atoms. Ans. (c) because [NCERT Exemplar]
2. The gravitational force between an H-atom (a) Triton energy is less than that of a He3
and another particle of mass m will be given by nucleus the beta decay process
Newton's law: 𝐹 = , where r is in km and (b) the electron created in cannot remain in
𝐺𝑀𝑚
𝑟2
[NCERT Exemplar] the nucleus,
(a) 𝑀 = 𝑚𝑃𝑟𝑜𝑡𝑜𝑛 + 𝑚𝑒𝑙𝑒𝑐𝑡𝑟𝑜𝑛 (c) both the neutrons in triton have to decay
(b) 𝑀 = 𝑚𝑃𝑟𝑜𝑡𝑜𝑛 + 𝑚𝑒𝑙𝑒𝑐𝑡𝑟𝑜𝑛 − simultaneously resulting in a nucleus with 3
𝐵
(𝐵 = 13.6 𝑒𝑉).
𝑐2
(c) M is not related to the mass of the protons, which is not a He3 nucleus.
hydrogen atom. (d) because free neutrons decay due to external
(d) 𝑀 = 𝑚𝑃𝑟𝑜𝑡𝑜𝑛 + 𝑚𝑒𝑙𝑒𝑐𝑡𝑟𝑜𝑛 − . |𝑉|=magnitude of perturbations which is absent in a triton
|𝑉|
𝑐2
the potential energy of electron in the H- nucleus. Ans. (a)
atom). Ans. (b) 6. Heavy stable nuclei have more neutrons than
3. When a nucleus in an atom undergoes a protons. This is because of the fact that [NCERT
radioactive decay, the electronic energy levels Exemplar]
of the atom [NCERT Exemplar] (a) neutrons are heavier than protons.
(a) do not change for any type of radioactivity. (b) electrostatic force between protons are
(b) change for α and β radioactivity but not for repulsive.
γ-radioactivity. (c) neutrons decay into protons through beta
(c) change for α-radioactivity but not for decay.
others. (d) nuclear forces between neutrons are weaker
(d) change for β-radioactivity but not for than that between protons. Ans. (b)
others. Ans. (b) 7. Samples of two radioactive nuclides A and B
4. Mx and My denote the atomic masses of the are taken A and 𝜆B are disintegration constants
parent and the daughter nuclei respectively in of A and B respectively. In which of the
a radioactive decay. The Q-value for a β- decay following cases, the two samples can
is Q1 and that for a β+ decay is Q2 If me simultaneously have the same decay rate at
denotes the mass of an electron, then which of any time?
the following statements is correct? [NCERT (a) Initial rate of decay of A is twice the initial
Exemplar] rate of decay of B and 𝜆A = 𝜆B
(a) Q1 = (Mx - My)c2 and Q2 = (Mx-My-2me )c2 (b) Initial rate of decay of A is less than the
(b) Q1 = (Mx - My)c2 and Q2 = (Mx-My)c2 initial rate of decay of B and 𝜆A < 𝜆B
(c) Q1 = (Mx - My-2me)c2 and Q2 = (Mx-My+ (c) Initial rate of decay of B is twice the initial
2me)c2 rate of decay of A and 𝜆A > 𝜆B

Advent Tutorial Front of Nehru Inter College Banarsidas, Auraiya


KP Singh 7248370933 ks5122213@gmail.com Page 16
Chapter- 12 Atom And Nuclei
(d) Initial rate of decay of B is same as the rate (d) the daughter nucleus has two nucleus more
of decay of A at t = 2h and 𝜆B = 𝜆A Ans. (d) than parent nucleus. Ans. (c)
8. The variation of decay rate of two radioactive 12. When a radioactive nucleus emits a B-particle,
samples A and B with time is shown in figure. the mass number of the atom:
Which of the following statements are true? (a) increases by one. (b) remains the same.
(c) decreases by one. (d) decreases by four.
Ans. (b)

𝑑𝑁 13. In a β-decay
𝑑𝑡 (a) the parent and daughter nuclei have the
same number of protons.
B
(b) the daughter nucleus has one proton less
A
than parent nucleus.
t
(c) the daughter nucleus has one proton more
(a) Decay constant of A is greater than that of than the parent nucleus.
B, hence A always decays faster than B. (d) the daughter nucleus has one neutron more
(b) Decay constant of B is greater than that of than the parent nucleus. Ans. (c)
A but its decay rate is always smaller than that 14. When a β-particle is emitted from a nucleus
of A. then its neutron-proton ratio
(c) Decay constant of A is equal to that of B. (a) increases
(d) Decay constant of B is smaller than that of (b) decreases
A but still its decay rate becomes equal to that (c) remains unchanged.
of A at a later instant. Ans. (d) (d) may increase or decrease depending upon
9. Radioactivity is the phenomenon associated the nucleus. Ans. (b).
with 15. The relation between half-life T1/2 of a
(a) decay of nucleus. radioactive sample and its mean life τ is
(b) production of radio waves. (a) T1/2 = 0.693 τ (b) τ = 0.693T1/2
(c) transmission of radio waves. (c) τ = T1/2 (d) τ = 2.718T1/2
(d) reception of radio waves. Ans. (a) Ans. (a)
10. Which of the following are not emitted by 16. β-rays emitted by a radioactive material are
radioactive substances? (a) electromagnetic radiations.
(a) Electrons (b) Protons (b) electrons orbiting around the nucleus.
(c) Gamma rays (d) Helium nuclei Ans. (b) (c) neutral particles.
11. In an α-decay (d) charged particles emitted by nucleus
(a) the parent and daughter nuclei have some Ans. (d)
number of protons. 17. During fraction that disintegrates is mean life
(b) the daughter nucleus has one proton more of a radioactive element the
than parent nucleus. (a) e (b) 1/e
(c) the daughter nucleus has two protons less (c) (e-1)/e (d) e/(e-1) Ans. (c)
than parent nucleus. 18. γ-rays are originated from
(a) nucleus.

Advent Tutorial Front of Nehru Inter College Banarsidas, Auraiya


KP Singh 7248370933 ks5122213@gmail.com Page 17
Chapter- 12 Atom And Nuclei
(b) outermost shell of nucleus. (c) 2 log 𝑒 2, and (d) and Ans. (b)
1 1 1
𝜆 𝜆 𝜆
(c) innermost shell of nucleus. 28. A radioactive nucleus emits a beta particle
(d) outermost shell of atom. Ans. (a) parent and daughter nuclei are
19. Binding energy per nucleon of a stable nucleus (a) isotopes (b) isotones
is (c) isomers (d) isobars Ans. (d)
(a) 8 eV (b) 8 KeV 29. In the disintegration series the values of Z and
(c) 8 MeV (d) 8 BeV Ans. (c) A respectively will be
20. Sun's radiant energy is due to -α -β 𝐴
𝑍𝑌
238
92𝑈 X
(a) nuclear fission.
(b) nuclear fusion. (a) 92, 236 (b) 88, 230
(c) photoelectric effect. (c) 90, 234 (d) 91, 2344 Ans (d)
(d) spontaneous radioactive decay. Ans. (b) 30. A nucleus 𝐴
𝑍𝑋 emits an α-particle. The resultant
21. Average binding energy is maximum for nucleus emits a β-particle. The respective
(a) C12 (b) Fe56 atomic and mass numbers of the daughter
(c) U235 (d) Po210 Ans. (b) nucleus will be
22. The quantity which is not conserved in a (a) Z-3, A-4 (b) Z-1, A-4
nuclear reaction is (c) Z-2, A – 4 (d) Z, A-2 Ans (b)
(a) momentum. (b) charge. 31. In the nuclear reaction, What does X stand for?
(c) mass. (d) none of these. Ans. (c)
23. A nucleus undergoes γ-decay due to 11 -α
6𝑈
11
5𝐵 + 𝛽+ + 𝑋
(a) excess of protons. (b) excess of neutrons,
(c) large mass. (d) its excited state. (a) Electron (b) Proton
Ans. (d) (c) Neutron (d) Neutrino . Ans. (d)
24. The half-life of a radioactive nucleus is 3 hours, 32. Neutrino is a particle, which is charge less and
9 hours, its activity will be reduced to a factor has spin. Ans. ±
𝟏
𝟐
of 33. Isotones have the same number of ____________.
(a) 1/9 (b) 1/27 Ans. neutrons
(c) 1/6 (d) 1/8 Ans. (d) 34. Packing fraction of a nucleus is its _________ its
25. A radioactive element has half-life period per nucleon. Ans. mass defect
1600 years. After 6400 years what amount 35. How is the radius of a nucleus related to its
will remain? mass number? [Panchkula 2019, AI 2011 C]
(a) 1/2 (b) 1/16 Ans. The radius R of a nucleus off mass number
(c) 1/8 (d) 1/4 Ans. (b) A is related as 𝑅 = 𝑅0 𝐴1/3 , where 𝑅0 , is a
26. Ratio of the radii of the nuclei with mass constant.
numbers 8 and 27 would be 36. Two nuclei have mass numbers in the ratio 27:
(a) 27/8 (b) 8/27 125. What is the ratio of their nuclear radii?
(c) 2/3 (d) 3/2 Ans. (c) Ans. Since 𝑅 ∝ 𝐴1/3 , 𝑅1 ∶ 𝑅2 = 27
1⁄ 1⁄
3: 125 3 = 3: 5.
27. The decay constant of a radioactive substance is 37. Two nuclei have mass numbers in the ratio 2:
𝜆. Its half-life and mean life, respectively are 5. What is the ratio of their nuclear densities?
(a) and log 𝑒 2 (b) , and
1 log𝑒 2 1
𝜆 𝜆 𝜆

Advent Tutorial Front of Nehru Inter College Banarsidas, Auraiya


KP Singh 7248370933 ks5122213@gmail.com Page 18
Chapter- 12 Atom And Nuclei
Ans. Nuclear density is independent of mass 44. In both 𝛽 − and 𝛽+ decay processes, the mass
number. So, the ratio will be 1: 1. number of a nucleus remains same, whereas
38. What is the relation between the binding the atomic number Z increases by one in 𝛽 −
energy per nucleon and stability of a nucleus? decay and decreases by one in 𝛽+ decay.
Ans. The larger the binding energy per nucleon, Explain, giving reason. [Foreign 2014]
the more stable is the nucleus. Ans. In 𝛽 − decay, one neutron inside the
39. Write any two characteristic properties of nucleus decays into one proton and one
nuclear force. [Chennai-2019, AI 2012, 13] electron (𝛽 − ) . The proton remains inside the
Ans. The following are the two characteristic nucleus and the electron is ejected out.
properties: 𝐴
𝑍𝑋
𝐴
𝑍+1𝑌 + 𝛽− + ʋ
(i) Nuclear force is a short range force. In 𝛽+ decay, the conversion of proton into
(ii) Nuclear forces show the saturation effect. neutron and position (𝛽 + ) takes place.
40. How is the mean life of a radioactive sample 𝐴
𝑍𝑋
𝐴
𝑍−1𝑌 + 𝛽+ + ʋ
related to its half-life? [Foreign 2011]
Ans. 𝜏 = 1.44 × 𝑇1/2 45. Which nucleus has greater mean life, A or B?
41. Define the activity of a given radioactive
substance. Write its SI unit.
Ans. The rate of disintegration or count rate of 𝑑𝑁
𝑑𝑡
sample of radioactive material is called activity.
The SI unit of activity is becquerel (Bq). A

42. The radioactive isotope D decays according to Ans. 𝑅 = 𝑅0 𝑒 −𝜆𝑡 B


the sequence Slop of graph =
𝑑𝑅
= −𝜆𝑅 t
𝑑𝑡

𝛽− α-Particle Since, slope of A is more than slope of B.


𝐷 𝐷1 𝐷2
Therefore, 𝜆 is high and mean life 𝜏 = for A is
1
𝜆
If the mass number and atomic number of D, small.
are 176 and 71 respectively, what is (i) the 46. Why is a free neutron unstable but a free
mass number (ii) atomic number of D? proton is a stable particle?
Ans. Ans. A free neutron is unstable outside the
𝛽− α- 176
nucleus with an average life of 1000 s. It
180
71𝐷2
180
72𝐷 73𝐷 1

decays into a proton and emits a 𝛽 − particle,


(i) Mass number of D = 180 i.e. 1
0𝑛
1
1𝐻
0 +
+ −1𝑒 ʋ + 𝑄
(ii) Atomic number of D = 72
43. A nucleus, n X m emits one α-particle and one 47. A neutron strikes a nucleus of 10
5𝐵 and emits an
β-particle. What is the mass number and alpha particle. Write down the nuclear reaction
atomic number of the product nucleus? for it.
Ans. m 2He
4
m-4 -1β
0
m-4
Ans. 1
0𝑛 + 10
5𝐵
7
3𝐿𝑖 + 4
2𝐻𝑒 + 𝑄
nX n-2 A n-1 B

Hence, the mass number of product is m-4, 48. Write the necessary condition required for

atomic number of product is n-1. fusion reaction.

Advent Tutorial Front of Nehru Inter College Banarsidas, Auraiya


KP Singh 7248370933 ks5122213@gmail.com Page 19
Chapter- 12 Atom And Nuclei
Ans. (i) Nuclear fusion will occur when the Ans. In case, the total mass energy of the decay
kinetic energy of colliding nuclei is enough to products is less than the mass energy of the
overcome the strong electrostatic forces of original nuclide; (Q-value of the process is
repulsion between the protons. For this, high positive) the energy is liberated. Though, the
temperature is required. number of nucleus is same, it is their mass that
(ii) The density of nuclei should also be very gets converted into the energy.
high to increase the number of collisions. 2. Using the curve for the binding energy per
49. Out of 30 6
14𝑋; 3𝑌; and 40𝑍,
130
which is more likely to nucleon as a function of mass number A, state

undergo the nuclear fusion? clearly how the release in energy in the
processes of nuclear fission and nuclear fusion
Ans. A lighter unstable nucleus Y can undergo
can be explained.
the nuclear fusion.
Ans. According to the binding energy curve, a
50. What is the effect of temperature on
very heavy nucleus (A > 170), has lower
radioactivity?
binding energy per nucleon compared to nuclei
Ans. No effect. Radioactivity is independent of
of middle mass number (30 < A < 170), When
temperature.
a heavy nucleus breaks into two nuclei of mass
51. What is the difference between an electron and
number between 30 and 170 respectively,
a 𝛽-particle?
nucleons get more tightly bound. This implies
Ans. An electron resides outside the nucleus,
that energy would be released in the process
whereas 𝛽-particle is an electron like particle of
(nuclear fission). When two light nuclei (A ≤ 10)
nuclear origin.
join to form a heavier nucleus, the binding
52. What is the source of stellar energy?
energy per nucleon of fused heavier nucleus
Ans. Nuclear fusion reactions.
increases. Again it indicates that energy would
53. Four nuclei of an element fuse together to form
be released in the process (nuclear fusion).
a heavier nucleus. If the process is accompanied
3. Draw a plot of the binding energy per nucleon
by the release of energy, which of the two-the
as a function of mass number for a large
parent or the daughter nucleus would have a
number of nuclei, 2 < A < 240. How do you
higher binding energy/nucleons? [CBSE 2018]
explain the constancy of binding energy per
Ans. Daughter nucleus.
nucleon in the range 30 < A < 170 using the
54. You are given two nuclei 73𝑋; and 43𝑌 , which one
property that nuclear force is short-ranged?
of the two is likely to be more stable? Give
[AI 2011]
reason.
Ans.
Ans. 73𝑋 is more stable, as it contains more
neutrons than protons.

Short Answer Type Questions [2 Marks]


1. If both the number of protons and the number
of neutrons are conserved in a nuclear reaction
like in what way is mass converted into
energy? Explain.
20
12
6𝐶 + 12
6𝐶 10𝑁𝑒 + 4
2𝐻𝑒

Advent Tutorial Front of Nehru Inter College Banarsidas, Auraiya


KP Singh 7248370933 ks5122213@gmail.com Page 20
Chapter- 12 Atom And Nuclei
Nuclear force is short-ranged. For a sufficiently For example,
large nucleus, a nucleon is under the influence 234
90𝑇ℎ
234
91𝑃𝑎 + −1
0
𝑒 + Q
of only some of its neighbours, which come 90𝑇ℎ; 𝑝 = 1.6 and 91𝑃𝑎; 𝑝 = 91 =
234 𝑛 144 234 𝑛 143
= 1.57
90
within the range or the nuclear force. If a 6. (a) Write the β-decay of tritium in symbolic
nucleon can have maximum of P neighbours form.
within the range of nuclear force, binding (b) Why is it experimentally found difficult to
energy, would be proportional to P. Thus, on detect neutrinos in this process? [Foreign
increasing A by adding nucleons, binding 2015]
energy will remain constant. Ans. (a)
3
1𝐻
3
2𝐻𝑒 + 𝛽− + ʋ + Q
decay.
4. Draw a plot representing the law of radioactive (b) Due to their very weak interaction with
decay. Define the activity of a sample of a matter.
radioactive nucleus. Write its SI unit. [Similar 7. A radioactive nucleus A undergoes a series of
Chennai 2019] decays according to the following scheme:
Ans. The plot representing the law of 𝛾
𝛼 𝛽 𝛼 𝐴3
𝐴 𝐴1 𝐴2 𝐴4
radioactive decay is as shown below.
the mass number and atomic number of A are
180 and 72 respectively. What are these
numbers for A4?
Ans. The series can be shown as below:

180 𝛼 176 𝛽 176 𝛼 172 𝛾 172


72𝐴 70𝐴1 71𝐴2 69𝐴3 69𝐴4

So, the atomic number of A4 is 69 and the


mass number of A4 is 172.
8. If the nucleons bound in a nucleus are
separated from each other, the sum of their
Activity is the total decay rate R of a sample of
masses is greater than the mass of the nucleus.
one or more radionuclides. The SI unit of
Where does this mass difference come from?
activity is becquerel (Bq).
Explain briefly.
5. Explain with an example, whether the neutron
Ans. If the nucleons bound in a nucleus are
to proton ratio increases or decreases during (i)
separated apart from each other, we need to
alpha decay, and (ii) beta decay.
transfer a total energy equal to the B.E. of the
Ans. During α-decay, the ratio of neutron to
nucleus to these particles.
proton in a nucleus increases.
Consequently, the sum of their masses becomes
For example 238
92𝑈
234
90𝑇ℎ + 4
2𝐻𝑒
+ Q greater than mass of nucleus according to the
Einstein's mass-energy equivalence, i.e. E = mc2.
In 92𝑈; 𝑝 = 92 = 1.58 and in
238 𝑛 146

9. State two characteristics of nuclear force. Why


90𝑇ℎ; 𝑝 = 90 = 1.6
234 𝑛 144

does the binding energy per nucleon decrease


(i) During 𝛽-decay, the ratio of a neutron to a
proton decreases..

Advent Tutorial Front of Nehru Inter College Banarsidas, Auraiya


KP Singh 7248370933 ks5122213@gmail.com Page 21
Chapter- 12 Atom And Nuclei
with increase in mass number for heavy nuclei tritium will remain un-decayed after 25 yrs.
like 235U? [NCERT Example]
Ans. The following are the two characteristics Ans. t = 25 yrs; T1/2 = 12.5 yrs
of nuclear forces: ∴ No. of half-lives, 𝑛 =
𝑡 25
= = 2;
𝑇1/2 12.5
(i) Nuclear forces are the strongest forces in 𝑁 1 1 2
= ( ) =
nature. 𝑁0 2 4
1
(ii) Nuclear forces depend on spin or angular 𝑁 = 𝑁0
4
momentum of nuclei. ∴ One-fourth of the sample of the initial pure
B.E. per nucleon decreases with increase in tritium will remain un-decayed.
mass number as electrostatic repulsion 13. The half-life period of a radioactive element A
increases due to increase in number of protons is the same as the mean life of another
and nucleus becomes less stable. radioactive element B. Initially both of them
10. Show that the decay rate R of a sample of a have same number of atoms. Show that the
radionuclide is related to the number of radioactive element B decay faster than A.
radioactive nuclei N at the same instant by the Ans. 𝜆1 and 𝜆2, be the decay constants of
expression 𝑅 = 𝜆𝑁. elements A and B respectively, then, according
Ans. We know that 𝑁 = 𝑁0 𝑒 −𝜆𝑡 to the question Half-life of A = Mean life of B
On differentiating w.r.t. time, we get T1/2 (A) = τ (B)
𝑑𝑁
= −𝜆𝑁0 𝑒 −𝜆𝑡 0.693 1
𝑑𝑡 =
𝑑𝑁 𝜆1 𝜆2
𝑅= − = 𝜆𝑁0 𝑒 −𝜆𝑡 = 𝜆𝑁 𝜆1
𝑑𝑡
= 0.693
𝜆2
𝜆1 > 𝜆2
Where 𝑅 is the decay rate of the sample.
No. of atoms initially in each sample is same
N is the number of nuclei that have not yet 𝑅1 𝜆1 𝑁
=
undergone decay. 𝑅2 𝜆2 𝑁
11. Are the equations of nuclear reactions
'balanced’ in the sense a chemical equation (e.g. Long Answer Type [I] Questions [3 Marks]

2H2 + O2 → 2H2O) is? If not, in what sense are 1. (a) Explain the processes of nuclear fission and

they balanced on both sides? [NCERT Example] nuclear fusion by using the plot of binding

Ans. A chemical equation is balanced in the energy per nucleon (BE/A) versus the mass

sense that the number of atoms of each number A. [Dehradun-2019]

element is the same on both sides of the (b) A radioactive isotope has half-life of 10

equation. In a nuclear reaction, the number of years.

atoms of each element is not necessarily How long will it take for the activity to reduce

conserved in a nuclear reaction. to 3.125%? [CBSE 2018]

In nuclear reactions, the number of protons 2. Distinguish between nuclear fission and fusion.

and the number of neutrons are the same on Show how in both these processes energy is

the two sides of the equation. released. Calculate the energy release in MeV in

12. Tritium has a half-life of 12.5 yrs undergoing the deuterium-tritium fusion reaction:

beta decay. What fraction of a sample of pure 2 + 3 4 1


1𝐻 1𝐻 2𝐻𝑒 + 0𝑛

Advent Tutorial Front of Nehru Inter College Banarsidas, Auraiya


KP Singh 7248370933 ks5122213@gmail.com Page 22
Chapter- 12 Atom And Nuclei
Using the data: [Foreign 2017]
𝑚( 21𝐻 ) = 2.014102 𝑢, 𝑚( 31𝐻 ) = 3.016049 𝑢 8. (a) Derive the relation between the decay
𝑚( 42𝐻𝑒) = 4.002603 𝑢, 𝑚( 10𝑛) = 1.008665 𝑢 constant and half-life of a radioactive
1 𝑢 = 931.5 𝑀𝑒𝑉/𝑐 2
[Delhi 2015]
substance.
3. (a) In a nuclear reaction,
(b) A radioactive element reduces to 25% of its
+1 +1 initial mass in 1000 years. Find its half-life.
3 + 3 4
2𝐻𝑒 2𝐻𝑒 2𝐻𝑒 1𝐻 1𝐻 + 12.86
MeV [Foreign 2017]
though the number of nucleons is conserve on
9. (a) Write the process of β-decay. How can
both sides of the reaction, yet the energy is
radioactive nuclei emit β-particles even though
released. How? Explain.
they do not contain them? Why do all electrons
(b) Draw a plot of potential energy between a
emitted during β-decay not have the same
pair of nucleons as a function of their
energy?
separation. Mark the regions where potential
(b) A heavy nucleus splits into two lighter
energy is (i) positive and (ii) negative.
nuclei.
[Delhi 2013, AI Z014]
Which one of the two-parent nucleus or the
4. Deduce the expression, 𝑁 = 𝑁0 𝑒 −𝜆𝑡 , for the law
daughter nuclei has more binding energy per
of radioactive decay.
nucleon? [Foreign 2017]
(b) (i) Write symbolically the process expressing
10. (a) Write symbolically the B decay process of
the β-decay of 11Na .
23 Also write the basic
15𝑃.
32

nuclear process underlying this decay.


(b) Derive an expression for the average life of a
(ii) Is the nucleus formed in the decay of the
radionuclide. Give its relationship with the half-
nucleus 11Na ,
23 an isotope or isobar?
life. [Delhi 2019]
[Delhi 2014]
11. (a) What is meant by half-life of a radioactive
5. Derive the expression for the law of radioactive
element?
decay of a given sample having initially N0,
(b) The half-life of a radioactive substance is 30
nuclei decaying to the number N present at
s.
any subsequent time t. Plot a graph showing
Calculate (i) the decay constant, and
the variation of the number of nuclei versus the
(ii) time taken for the sample to decay by
timer lapsed. Mark a point on the plot in terms
3/4th of the initial value.
of T1/2, value when the number present 𝑁 =
12. The half-life of C is 5700 years. What does it
𝑁0 /16. [Dehradun 2019, Foreign 2013]
mean? Two radioactive nuclei X and Y initially
6. How is the size of a nucleus experimentally
contain an equal number of atoms. Their half-
determined? Write the relation between the
lives are 1 hour and 2 hours respectively.
radius and mass number of the nucleus. Show
Calculate the ratio of their rates of
that the density of nucleus is independent of its
disintegration after two hours.
mass number. [Delhi 2011C]
13. Explain, giving necessary reactions, how energy
7. (a) State the law of radioactive decay. Write
is released during (i) fission and (ii) fusion.
the SI unit of 'activity'.
[Chennai 2019, AI 2011C]
(b) There are 4√2 × 106 radioactive nuclei in a
14. (a) Write the basic nuclear process involved in
given radioactive sample. If the half-life of the
the emission of β in a symbolic form, by a
Sample is 20 s, how many nuclei will decay in

Advent Tutorial Front of Nehru Inter College Banarsidas, Auraiya


KP Singh 7248370933 ks5122213@gmail.com Page 23
Chapter- 12 Atom And Nuclei
radioactive nucleus. 4. In a given sample, two radioisotopes, A and B,
(b) In the reactions given below: are initially present in the ratio 1:4. The half-
lives of A and B are respectively 100 years and
(𝑖) 11 𝑧
6𝐶 𝑥𝐵 + 𝑥 +ʋ
(𝑖𝑖) 12 + 126𝐶 20𝑁𝑒 + 𝑏𝑐𝐻𝑒 50 years. Find the time after which the
6𝐶 𝑎
amounts of A and B become equal. [Foreign
Find the values of x, y and z and a, b and c. [AI 2012]
2016] 5. If 20% of a radioactive substance decay in 10
days. What percentage of original material is
Long Answer Type [II] Questions [5 Marks] left after 30 days?
15. (a) Draw the plot of binding energy per 6. A neutron is absorbed by a 63𝐿𝑖 nucleus with the
nucleon (BE/A) as a function of mass number subsequent emission of an alpha particle.
A. Write two important conclusions that can be (i) Write the corresponding nuclear reaction.
drawn regarding the nature of nuclear force. (ii) in Calculate the energy released in MeV in
(b) Use this graph to explain the release of this reaction.
energy in both the processes of nuclear fusion Given: mass ( 63𝐿𝑖 ) = 6.015126 𝑢;
and fission. mass (neutron) = 1.0086654 𝑢;
(c) Write the basic nuclear process of neutron mass (alpha particle) = 4.0026044 𝑢 and
undergoing β-decay. Why is the detection of mass (tritium) = 3.0100000 𝑢.
neutrinos found very difficult? [AI 2013] Take 1u = 931 MeV/c2
16. (a) Define the terms (i) half-life (T1/2) and (ii) 7. The decay constant, for a given radionuclide,
average life (τ). Find out their relationship with has a value of 1.386 day-1. After how much
the decay constant (𝜆). [Dehradun 2019] time will a given sample of this radionuclide get
(b) A radioactive nucleus has an decay constant reduced to only 6.25% of its present number?
𝜆 = 0.3465 (day). How long would it take the 8. Calculate the energy released if 238U nucleus
nucleus to decay to 75% of its initial amount? emits an α-particle. Given: atomic mass of
[Foreign 2014] 238U = 238.0508 𝑢,
atomic mass of 234Th = 234.04363 𝑢,
Numerical Problems atomic mass of alpha particle = 4.00260 𝑢
1. Calculate the excitation energy the nuclei and 1 𝑢 = 931 𝑀𝑒𝑉/𝑐 2
produced when 235U and 238U absorbs thermal 9. How many disintegrations per second will occur
neutrons. Given 𝑚𝑛 = 1.0087 𝑢; in one gram of 92U
238, if its half-life against an
𝑚(𝑈 235 ) = 235.0439 𝑢; 𝑚(𝑈 236 ) = 236.0456 𝑢 alpha decay is 1.42x1017s?
𝑚(𝑈 238 ) = 238.05084 𝑢 𝑚(𝑈 239 ) = 239.0543 𝑢
10. Calculate the energy in fusion reaction:
2. A nucleus with mass number A = 240 and 2 2 3 1
1𝐻 + 1𝐻 2𝐻𝑒 + 0𝑛
BE/A = 7.6 MeV breaks into two fragments
Where Binding energy of 21𝐻 = 2.23 𝑀𝑒𝑉 and of
each of A = 120 with BE/A = 8.5 MeV.
3
1𝐻 = 7.73 𝑀𝑒𝑉 [Delhi 2016]
Calculate the released energy. [Delhi 2016]
11. A heavy nucleus X of mass number 240 and
3. A radioactive isotope has a half-life of T years.
binding energy per nucleon 7.6 MeV is split
After how much time is its activity reduced to
into two fragments Y and Z of mass numbers
6.25% of its original activity?
110 and 130. The binding energy of nucleons

Advent Tutorial Front of Nehru Inter College Banarsidas, Auraiya


KP Singh 7248370933 ks5122213@gmail.com Page 24
Chapter- 12 Atom And Nuclei
in Y and Z is 8.5 MeV per nucleon. Calculate he 7. In a given sample, two radio isotopes, A and B are
energy Q released per fission in MeV. initially present in the ratio of 1:4. The half-lives
12. When four hydrogen nuclei combine to form a of A and b are 100yr and 50yr, respectively. Find
helium nucleus, estimate the amount of energy the time after which the amounts of A and B
in MeV released in this process of fusion. become equal.
(Neglect the masses of electrons and neutrons.) 8. How the size of nucleus experimentally
13. 3
2𝐻𝑒 and 3
1𝐻 nuclei have the same mass number. determined? Write the relation between the radius
Do they have the same binding energy? and mass number of the nucleus. Show that the
14. Draw a graph showing the variation of decay density of nucleus is independent of its mass
rate with number of active nuclei. number.
15. Why do stable nuclei never have more neutrons 9. Draw the graph showing the variation of binding
than energy per nucleons with mass number. Also, give
16. The following table shows some measurements the reasons fo0r the decrease of binding energy
of the decay rate of a radionuclide sample. Find per nucleons for high mass number nuclei.
the disintegration constant. [CBSE Sample 10. If N0 is the number of radioactive nuclei at some
Paper 2016] initial time t0.find out the relation to determine
Time 30 100 164 218 the number N present at a subsequent time. Draw
(Minutes )
ln R (Bq) 5.08 3.29 1.52 1.00 a plot of N as a function of time.
11. In a given sample, two radio isotopes, A and B are
initially present in the ratio of 1:8. The half-lives
Very Short Answer type Questions (1 Marks)
of A and B are 200yr and 100yr, respectively.
1. Why it is found experimentally difficult to detect
Find the time after which the amounts of A and B
neutrinos in nuclear 𝛽-decay?
become equal.
2. A nucleus undergoes 𝛼-decay. How does its
12. A radioactive nucleus, A undergoes a series of
i. Mass number and
decays according to the following scheme;
ii. Atomic number change? 𝛼 𝛽 𝛼
A→ A1 → A2 → A3 → A4 the mass number and
3. Two nuclei have mass numbers in the ratio of
atomic number of A4 are 172 and 69,
27:512. What is the ratio of their nuclear radii?
respectively. What are these numbers for A?
4. Name the materials used as moderators in nuclear
13. If the both the numbers of protons and neutrons
reactor and write the reasons for their use as
are conserved in a nuclear reaction like
moderator. 12
6𝐶 + 12
6𝐶 → 20
10𝑁 + 42𝐻𝑒
5. A nucleus 235
92𝑈 undergoes 𝛽-decay and transforms
in what way is the mass converted into the
to x-nucleus. What is
energy? Explain.
i. The mass number and
ii. Atomic number of the nucleus produced?
Short Answer type Questions (3 Marks)
6. Define the term ‘activity’ of a radio nuclide (by
14. (i). Write the characteristic properties of nuclear
neutrons) to sustain a chain reaction? What type
force.
of nuclei re (preferably) needed for slowing down
(ii). Draw a plot of potential energy of pair of
fast neutrons?
nucleons as a function of their separation. Write

Short Answer type Questions (2 Marks)

Advent Tutorial Front of Nehru Inter College Banarsidas, Auraiya


KP Singh 7248370933 ks5122213@gmail.com Page 25
Chapter- 12 Atom And Nuclei
two important conclusions that can be drown 5. (i). The number of nuclei of a given
from the graph. radioactive sample at time t =0 and t =T are
1. In the study of Geinger-Marsdon experiment N0 and N0/n respectively. Obtain an
on scattering of 𝛼-particle by a thin foil of expression for the half-life T1/2 of the nucleus
gold, draw the trajectory of 𝛼-particle in the in terms of n and T.
coulomb field of target nucleus. Explain (ii). write the basic nuclear process underlying
briefly how one gets information on the size 𝛽 − decay of a given radioactive nucleus.
of nucleus from the study. From the relation
R=R0A1/3, where R0 is constant and A is the 6. Draw a plot of potential energy between a
mass number of the nucleus, show that pair of nucleons as a function of their
nuclear matter density is independent of A. separation. Marks the region where potential
energy is (i) Positive (ii) Negative
2. Distinguish between nuclear fission and
nuclear fusion. Show how in both these 7. (i). What characteristic property of nuclear
processes energy is release in MeV in the force explains the constancy of binding
deuterium-tritium fusion reaction. energy per nucleons (BE/A) in the range of
2
1𝐻 + 3
1𝐻→ 4
2𝐻𝑒 + 10𝑛 usingthe data mass number A lying 30<A<170?
𝑚( 21𝐻 )
= 2.014102 𝑢, 𝑚( 31𝐻 ) = (ii). Show that the density of nucleons over a
3.016049 𝑢, 𝑚( 42𝐻𝑒 ) = 4.002603 𝑢, 𝑚( 10𝑛) =
wide range of nuclei is constant and
1.008665 𝑢, 1𝑢 = 931.5 MeV/c2.
independent of mass number A.

3. Draw a plot of potential energy of a pair of


Long Answer type Questions (5 Marks)
nucleons, as a function of their separations.
8. (i) Draw the plot of binding energy per
Marks the regions where the nuclear force is
nucleons (BE/A) as a function of mass
(i) attractive (ii) repulsive. Write any two
number A. write two important conclusions
characteristic features of nuclear forces.
that can be drown regarding the nature of
nuclear force.
4. (i) Deduce the expression N=N0𝑒 −𝜆𝑡 for the
(ii) Use this graph of explain the release of
law radioactive decay.
energy in both the processes of nuclear fission
(ii). (a). write the symbolically the process
and nuclear fusion.
expressing the 𝛽 + decay of 11𝑁𝑎 .
22
Also, write
(iii) Write the basic nuclear process of
the basic nuclear process underlying this
neutron undergoing 𝛽-decay. Why is the
decay.
detection of neutrinos found very difficult?
(b). Is the nucleus formed in the decay of the
nucleus 22
11𝑁𝑎, an isobar?

Advent Tutorial Front of Nehru Inter College Banarsidas, Auraiya


KP Singh 7248370933 ks5122213@gmail.com Page 26
13 Semi-conductor
devices and circuit
Chapter
Electronic devices- Any device whose action is considerably lower number density of charge
based on the controlled flow of electrons through carriers than metals. Semi-conductors may be
it is called an electronic device. The branch of elemental (Si, Ge) and compound (GaAs, CdS, etc).
physics that deals with the study of these
electronic devices is called electronics. These Energy bands in solids- in an isolated atom, the
electronics devices are of two types (i) Vacuum electrons occupy well defined discrete energy
tubes (ii) Solid State electronic devices. levels. But due to interatomic interactions in a
crystal, the electron of outer shells are forced to
Vacuum tubes- These include vacuum diode, have energies different from those in isolated
triode, etc. in a vacuum tube, the electrons atoms. Each energy level splits into a number of
obtained from a heating cathode are controlled by energy levels forming a continuous band. An
varying voltages between its different electrodes. enormously large number of energy levels closely
These devices are bulky, consume high power, spaced in a very small range constitute an energy
operate generally at high voltages, have limited life band. The allowed energy bands are separated by
and low reliability. region in which energy levels cannot exist. These
forbidden regions are called band gaps or energy
Solid-state electronic devices- in such devices, the gaps. The highest energy band occupied by the
charge carriers flow through solid-state valence electrons is called valence band and the
semiconductors. These devices include junction next empty allowed band is called the conduction
diode, transistors and integrated circuit(IC). These band.
are small in size, consume low power, operate at
low voltages, have long life and high reliability. Fermi level- the highest energy level filled with
electrons at absolute zero is called Fermi level and
Classification of solid material on the basis of their the energy corresponding to the Fermi level is
resistivity- metal have low resistivity (10-2 to 10- called Fermi energy.
8Ωm), insulator have high resistivity (>108Ωm),
while semiconductor have intermediate value Distinction between metals, insulators and
between (105 to 100) of resistivity. semiconductors on the basis of band theory-
Metals- in metals, either the conduction band is
Semiconductor- they have much higher resistivity partially filled or the valence and conduction
than metals. Their temperature coefficient of bands partly overlap. Here Eg=0; this makes the
resistivity (α) is both negative and high. They have available a large number of free electrons for
Chapter- 13 Semiconductor devices and circuit
electric conduction. So metals have high Trivalent dopants- such as In, B and Al. these are
conductivity or low resistivity. also called acceptors.
Extrinsic semiconductors- a semiconductor doped
Insulators- here the conduction band is empty and with suitable impurity atoms so as to increase the
the valence band is filled. The forbidden energy conductivity is called an extrinsic semiconductor.
gap is large (Eg>3eV). Electrons cannot be excited Extrinsic semiconductors are of two types
from the valence band to the conduction band (i) n-type semiconductors
even by applying a strong electric field. Therefore, (ii) p-type semiconductors.
no electrical conduction is possible.
n-type semiconductors- the pentavalent impurity
Semiconductors- the empty conduction band is atom are called donors because they donate
separated from the filled valence band by a small electrons to the host crystal and the
energy gap (Eg<3eV). Some electrons of valence semiconductor doped with donors is called n-type
band easily thermally excited to the conduction semiconductor. In n-type semiconductors,
band and can conduct electricity. So electrons are the majority charge carrier and holes
semiconductors acquire small conductivity even at are the minority charge carriers thus ne≈ 𝑁𝐷 >>nh.
room temperature.
p-type semiconductors- the trivalent impurity
Intrinsic semiconductors- the pure semiconductors atoms are called acceptors because they create
in which the electrical conductivity is totally holes which can accept electrons from the nearby
governed by the electrons excited from the valence bonds. A semiconductor doped with acceptor type
band to conduction band and in which no impurities is called p-type semiconductors. In p-
impurity atoms are added to increase the their type semiconductor, the holes are majority carrier
conductivity are called intrinsic semiconductor and and electrons are the minority carriers.
their conductivity is called intrinsic conductivity. Thus nh≈ 𝑁𝐴 >>ne in any semiconductor nenh=ni2
Electrical conduction in pure semiconductors moreover, the material on the whole is electrically
occurs by means of electron-hole pairs. In an neutral.
intrinsic semiconductors. ne=nh=ni where ne
electron density in conduction band nh hole Holes- The vacancy or absence of electron in the
density in valence band and ni the intrinsic carrier bond of a covalently bonded crystal is called a
concentration. holes. A hole serves as a positive charge carrier.

Doping- the process of deliberate addition of a Mobility- The drift velocity acquired by a charge
desirable impurity to a pure semiconductor so as carrier in a unit electric field is called its electrical
to increase its conductivity is called doping. The mobility and is denoted by µ. µ= SI unit of
𝑣𝑑
𝐸
impurity atoms added are called dopants. Dopants mobility is m2V-1s-1. The mobility of electron in
are of two types: the conduction band is greater than that of the
holes (or the electron) in the valance band.
Pentavalent dopants- such as As. Sb and P . these
are also called donors. p-n junction- it is a single crystal of Ge or Si
doped in such a manner that one half portion of it

Advent Tutorial Front of Nehru Inter College Banarsidas, Auraiya


KP Singh 7248370933 ks5122213@gmail.com Page 2
Chapter- 13 Semiconductor devices and circuit
acts as p-type semiconductor and other half
functions as n-type semiconductor. As soon as a

I mA (Forward Bias Current)


p-n junction is formed, the holes from the p-
region diffuse into the n-region and electrons
from n-region diffuse into p-region. This results in
the development of potential barrier VB across the
junction which opposes the further diffusion of
V (Forward Bias Voltage)

electrons and holes through the junction. The small


region in the vicinity of the junction which is Dynamic resistance- the dynamic or ac resistance
depleted of free charge carriers and has immobile of a diode is the ratio of small change in applied
ions is called the depletion region. It is represented voltage ∆𝑉 to the corresponding change in
current∆𝐼. It is given by rd = it decreases as
∆𝑉
by the symbol. ∆𝐼
increasing the applied voltage.
Rectification- the process of converting the a.c.
into d.c. is called rectification and the device is
called rectifier.
It is of two type Half wave rectifier and full wave
rectifier.
Anode Cathode

Junction diode as an rectifier- A junction diode

Forward and reverse bias of a p-n junction- if the conducts when it is forward bias and does not

positive terminal of a battery connected to the the conduct when reverse biased. This unidirectional

p-side and the negative terminal to the n-side, characteristic of the diode enables it to be used as

then the p-n junction is said to be forward biased. a rectifier. A half wave rectifier uses only single

Both electrons and holes toward the junction. A diode and rectify only the half wave. While a full

current, called forward current, flow across the wave rectifier uses two diode and rectify complete

junction. Thus a p-n junction offers a low wave.

resistance when it is forward biased.


If the positive terminal of a battery is connected Half wave rectifier- in a half wave rectifier, it

to the n-side and negative terminal to the p-side, consist of a single diode connected with the load

the p-n junction is said to be reverse biased. The resistance along with a step down transformer as

majority carrier moves away from the junction. shown in the figure. It work on the principle of

The potential barrier offers high resistance during when diode connected in the forward bias it

the reverse bias. However, due to minority charge conduct the current while, when it reverse bias it

carriers a small current, called reverse or leakage would not do for even half cycles it conduct but

current flows in the opposite direction. Thuds the for odd half cycles it stops the current. And out

junction has almost a unidirectional flow of put for given input is shown in the figure. Half

current. The forward bias current is large (mA) wave rectifier only rectify the half wave of input,

while reverse bias current is small (µA) in a p-n that why it called half wave rectifier.

junction diode

Advent Tutorial Front of Nehru Inter College Banarsidas, Auraiya


KP Singh 7248370933 ks5122213@gmail.com Page 3
Chapter- 13 Semiconductor devices and circuit
resistance (RL), current through thezener diode
may change without change in the voltage across
it.

Full wave rectifier- A full wave rectifier is the


Photodiode- it is a junction diode made from a
combination of two half wave rectifiers. For odd
photosensitive semiconducting material in such a
half cycles one diode is forward bias and allow the
way that light can fall on its junction. It is operate
current while second diode become reverse bias
in reverse bias condition. The photon excitation
and stops the current. And for remaining even
results in a change of reverse saturation current
half cycle’s first diode become reverse bias and
which help us to measure the light intensity. It is
second one is in the state of forward bias and
used to detect optical signal, photo switches,
allows the current to pass. So both diode for given
converting optical signal to electrical signal.
input produces the output as shown in the figure.

It rectifies the complete wave that’s why it is


called full wave rectifier.
Light emitting diode(LED)- it is a forward bias p-

Zener diode- A junction diode specially designed n junction which spontaneously converts the

to work only in the reverse breakdown region biasing electrical energy into the optical energy,

continuously is called Zener diode. Voltage drop like infra-red and visible light. It is made from a

across it is independent of current flowing through translucent semiconductor like GaAs or InP. LEDs

it which enables it to work as an voltage regulator. are used in calculator, hand clock, and digital

Zener diode as a voltage regulator- For widely display.

zener currents, the voltage across the zener diode


remains constant. on the accounts of this fact we
can use the zener diode as a voltage regulator. For
input Vi > Vz, zener diode is in breakdown
condition. Thus, for wide range of values of load

Advent Tutorial Front of Nehru Inter College Banarsidas, Auraiya


KP Singh 7248370933 ks5122213@gmail.com Page 4
Chapter- 13 Semiconductor devices and circuit
relatively thicker sections of n-type crystals or a
narrow section of n-type crystal between two
thicker p-type crystals. The first type is called n-
p-n transistor and second type is called p-n-p
transistor. Each types of transistors have three
components Emitter (E), Collector (C), and Base
(B).
The base is very thin and lightly doped the emitter
is always forward biased and the collector is
always reverse biased. The emitter supplies a large
no of majority charge carriers for the flow of
current through the transistor. The base controls
the flow of majority charge carriers from the
emitter to collector. The collector collects the
majority charge carriers for the transistor
operation.
n-p-n
E C

Solar Cell- it is a junction diode used to convert B


the solar energy into electrical energy. It is based
on the photovoltaic effect (Generation of voltage
by bombardment of light photons). The materials Action of transistor- When the emitter-base
used for solar cells are Si and GaAs. junction is forward biased majority charge carriers
are pushed from emitter to base. Few electrons
and holes recombine in the base region. Most of
the majority carriers cross over to the reversed
biased collector. Collector collect the current Ic is
always slightly less than the emitter current. IE =
IC+IB
Three configurations of a transistor- A transistor
can be used in one of the following three
configuration:
VOC
Common-base (CB) circuit
Common-emitter (CE) circuit
Common-Collector (CC) circuit
Common-emitter characteristics of a transistor-
ISC
these are the graphs between the appropriate
Junction transistors- it is a three terminal solid voltage and currents for a transistor when it
state device obtained by growing either a narrow emitter is taken as common terminal and
section of p-type crystal between the two grounded (zero potential), base is the input

Advent Tutorial Front of Nehru Inter College Banarsidas, Auraiya


KP Singh 7248370933 ks5122213@gmail.com Page 5
Chapter- 13 Semiconductor devices and circuit
terminal and collector is the output terminal. If the emitter base junction is not forward biased
Three types of characteristic curves are studied: or if the bias is insufficient to start forward bias
Input characteristic- it is the graph showing the current across the junction while collector base
variation of base current IB with base emitter junction is reverse biased, then the transistor is
voltage VBE at constant collector emitter voltage said to be in cut-off state.
VCE. The input resistance is ri of the transistor in If the emitter-base junction is forward biased and
CE configuration is defined as the ratio of the the collector current flows in linear proportion
small change in the base-emitter voltage to the with the base current. The transistor is said to be
corresponding change in base current , when the in the active state.
collector- emitter voltage is kept fixed ri= If the emitter-base junction is heavily forward
biased and the collector current produces such
∆𝑉𝐵𝐸
[ ]
∆𝐼𝐵 𝑉 = 𝑐𝑜𝑛𝑠𝑡𝑎𝑛𝑡
𝐶𝐸

Output characteristics- it is the graph that show large drop across the load resistor that the base-

the variation of collector current IC with the collector junction also gets forward biased, then

collector- emitter voltage VCE at the constant the transistor is said to be in saturation state.

base-current IB. the output resistance ro of a Transistor as a switch- A transistor can be used as

transistor in CE configuration is defined as the a switch if it is biased in its cut off and saturation

ratio of small change in the collector emitter states only. The active state is forbidden when it is

voltage to the corresponding change in the used as a switch.

collector current when the base current is kept Transistor as an amplifier- An amplifier is a

constant. ro = [
∆𝑉𝐶𝐸
] circuit which is used for increasing the voltage,
∆𝐼𝐶 𝐼 = 𝑐𝑜𝑛𝑠𝑡𝑎𝑛𝑡
𝐵
current or power of alternating form. A transistor
Transfer characteristic- it is a graph showing the
operates as an amplifier when its emitter-base
variation of collector current IC with the base
junction is forward biased while collector-base
current IB at constant collector-emitter voltage𝑉𝐶𝐸 .
junction is reverse-biased. Transistor biased in this
Current gain of transistor- usually two current
way is said to be in active state.
gain are defined:
Common base current amplification factor or ac
Common emitter transistor amplifier-
gain 𝛼- it is the ratio of the small change in
d.c. current gain- it is defined as 𝛽𝑑𝑐 =
𝐼𝐶

collector current to the small change in emitter 𝐼𝐵


a.c. current gain- it is defined as the 𝛽𝑎𝑐 =
∆𝐼𝐶
current when the when the collector-base voltage ∆𝐼𝐵

is kept constant. 𝛼 = [
∆𝐼𝐶
]
Voltage gain- it is defined as 𝐴𝑉 = = 𝛽𝑎𝑐 . =
∆𝐼𝐸 𝑉 = 𝑐𝑜𝑛𝑠𝑡𝑎𝑛𝑡 ∆𝑉𝐶𝐸 𝑅𝑜
𝐶𝐵

Common emitter current amplification factor or


∆𝑉𝐵𝐸 𝑅𝑖
𝐴𝑖 . 𝐴𝑟
ac current gain 𝛽- it is the ratio of the small Power gain – it is defined as 𝐴𝑃 =
𝑂𝑢𝑡𝑝𝑢𝑡 𝑝𝑜𝑤𝑒𝑟
=
𝐼𝑛𝑝𝑢𝑡 𝑝𝑜𝑤𝑒𝑟
change in the collector current to the small change .
∆𝑉𝐶𝐸 ∆𝐼𝐶
= 𝐴𝑉 . 𝛽𝑎𝑐 = 𝛽𝑎𝑐
2 . 𝑅𝑜
∆𝑉𝐵𝐸 ∆𝐼𝐵 𝑅𝑖
in the base current when the collector emitter
voltage is kept constant.
∆𝐼𝐶
𝛽=[ ]
∆𝐼𝐵 𝑉 = 𝑐𝑜𝑛𝑠𝑡𝑎𝑛𝑡
𝐶𝐸
Trans-conductance- it is defined as the ratio of
Relation between 𝜶 𝒂𝒏𝒅 𝜷- the current 𝛼 𝑎𝑛𝑑 𝛽 are small change in collector current (∆𝐼𝐶 ) to the small
related as 𝛼= and 𝛽=
𝛽 𝛼
change in input base-emitter voltage. gm= its
∆𝐼𝐶
1+𝛽 1−𝛼 ∆𝑉𝐵𝐸
unit is Ω-1 or siemen (S). it depends on the
Three states of transistor- A transistor in a circuit geometry, doping level and biasing of the
can be in one of the three conditions: transistor.

Advent Tutorial Front of Nehru Inter College Banarsidas, Auraiya


KP Singh 7248370933 ks5122213@gmail.com Page 6
Chapter- 13 Semiconductor devices and circuit
Analog circuit- these circuits process a signal output (1) if input A and B are both high (1)
(current or voltage) in the form of continuous, otherwise it gives low output (0) it is describe by
time varying voltage or current. Such signals are the Boolean expression Y=A.B:
called continuous or analog signals.
NOT gate- it is a digital circuit having only one
Digital circuits- these circuits, instead of using input and one output. It gives a high output (1) if
continuous signals, make use of discrete two-level the input A is low (0) and vice versa. It is
or binary signals such signal are called digital described by the Boolean expression, Y=𝐴̅
signals. Here only two values (0 and 1) of the
input and out voltage are permissible. NAND gate- it is obtained by connecting output of
AND gate to the input of NOT gate. It’s output is
Logic gates- A logic gate is a digital circuit that high if both the inputs A and B are not high. It is
has one or more inputs but only one output. It describe by the Boolean expression, Y=𝐴.
̅̅̅̅̅
𝐵
follows a logical relationship between input and
output voltages. NOR gate- it is obtained by connecting output of
OR gate to the input of NOT gate. It’s output is
Truth table- this table shows all possible input high if neither inputs A nor input B is high. It is
combinations and the corresponding outputs for a described by the Boolean expression, Y=𝐴. 𝐵 .
̅̅̅̅̅
logic gate.
XOR or exclusive OR gate- the XOR gives a high
Boolean expression- it is a shorthand method to input if either input A or B is high but not when
describe the function of a logic gate in the form of both A and B are high or low. It can be obtained
an equation or an expression. It also relates the all by using a combination of two NOT gates, two
possible combinations of the inputs of a logic gate AND gates and one OR gate. It is described by
to the corresponding outputs. Boolean expression: Y= 𝐴.
̅ 𝐵 + 𝐴. 𝐵̅ the XOR gate is
called difference gate because its output is when
Positive and negative logic- if in a system, the both the inputs are different.
higher voltage level represents 1 and lower voltage
level represent 0, the system is called a positive Integrated circuits (ICs)- A miniature electronic
logic. If the higher voltage level represents 0 and circuit, consisting of many passive components like
the lower voltage represent 1, then the system is R and C and active components like diode
called a negative logic. transistor, fabricated within a single
semiconductor chip is called an integrated circuit.
OR gate- it is the digital circuit having two or ICs are obtained by a complex procedure involving
mare inputs but only on output. It gives a high diffusion, oxidation, photolithography,
output if either input A or B or both are high (1) matallisation, etc these circuits have revolutionised
otherwise it gives low output (0). It is described by the electronic industry.
Boolean expression: Y =A+B
Basic Gates
And gate- it is a digital circuit having two or
more input but only one output. It gives a high

Advent Tutorial Front of Nehru Inter College Banarsidas, Auraiya


KP Singh 7248370933 ks5122213@gmail.com Page 7
Chapter- 13 Semiconductor devices and circuit

Not Gate
A Y =𝐴̅
0 1
1 0

AND Gates OR Gate

A B Y = A.B A B Y= A+ B

0 0 0 0 0 0

0 1 0 0 1 1

1 0 0 1 0 1

1 1 1 1 1 1

NAND GAte
NOR Gate
A B Y= ̅̅̅̅̅
𝐴. 𝐵
A B Y=𝐴
̅̅̅̅̅̅̅̅
+𝐵
0 0 1
0 0 1
0 1 1
0 1 0
1 0 1
1 0 0
1 1 0
1 1 0

Advent Tutorial Front of Nehru Inter College Banarsidas, Auraiya


KP Singh 7248370933 ks5122213@gmail.com Page 8
Chapter- 13 Semiconductor devices and circuit
Very Short Answer type Questions (1 Marks) 10. Draw the circuit diagram of a full wave rectifier
1. Show the variation of resistivity of Si with the using p-n junction diode. Explain its working and
temperature in a graph. show the output and input waveforms.
B 11. (i) State briefly the processes involved in the
formation of p-n junction explaining clearly how
C the depletion region is formed.
A
(ii) Using the necessary circuit diagrams, show how
The graph shown in the figure, represents a plot the V-I characteristic of a p-n junction are
of current versus voltage for a given obtained in
semiconductors. Identify the region if any, over (a) Forward biasing (b) reverse biasing?
which the semiconductor has a negative resistance. How these characteristics are made use of in
Short Answer type Questions (2 Marks) rectification?
2. Explain with the help of a circuit diagram, the 12. (i) How is a depletion region formed in p-n
working of a p-n junction diode as a half-wave junction?
rectifier. Draw a circuit diagram (ii) With the help of a labelled circuit diagram,
3. Assuming that the two diode D1 and D2 used in explain how a junction diode is used as a full wave
the electric circuit as shown in the figure are ideal, rectifier. Draw its input, output wavefronts.
find out the value of the current through 1Ω (iii) How do you obtain steady DC output from the
resistor. pulsating voltage?
4. Write two characteristics features distinguish 13. Why is Zener diode considered as a special purpose
Between n-type and p-type semicunductors. semiconductor diode? Draw the I-V characteristics
5. Give two advantages of LED’s over the of Zener diode and explain briefly how reverse
conventional incandescent lamps. current suddenly increase at the breakdown
6. The current in the forward bias (~mA) is known voltage?
to be more than the current in the reverse bias Describe briefly with the help of a circuit diagram,
(~µA). what is the reason. To operate the how a Zener diode works to obtained a constant
photodiode in reverse bias? DC voltage from the unregulated DC output of a
7. With what considerations in view, a photodiode is rectifier.
fabricated? State it’s working with the help of a 14. (i) Describe briefly with the help of a diagram, the
suitable diagram. Even though the current in the role of the two important processes involved in the
forward bias is known to be more than in the formation of a p-n junction.
reverse bias. What is the reason? (ii) Name the device which is used as a voltage
8. Write the distinguishing features between regulator. Draw the necessary circuit diagram and
conductors, semiconductors and insulators on the explain its working.
basis of energy band diagrams.
9. Mention the important considerations required
D1 2Ω
while fabricating a p-n junction diode to be used a
light emitting diode (LED). What should be the D2 2Ω
order of the band gap of an LED, if it is required
6V
to emit light in the visible range? 1Ω

Advent Tutorial Front of Nehru Inter College Banarsidas, Auraiya


KP Singh 7248370933 ks5122213@gmail.com Page 9
Chapter- 13 Semiconductor devices and circuit
Very Short Answer/Objective Type Questions [1 (d) D1 and D2 are both reverse biased and
Mark] hence no current flows from A to B and vice
1. The conductivity of a semiconductor increases versa. Ans (b)
with increase in temperature, because 4. _________________ is a vacancy created when an
(a) number density of free current carriers electron leaves a covalent bond.
increases. 5. The output of the given circuit in figure is
(b) relaxation time increases. given below. [NCERT Exemplar]
(c) both number density of carriers and
relaxation time increase.
(d) number density of carriers increases, 𝑉𝑚 Sin𝜔𝑡 ~

relaxation time decreases but effect of


decrease in relaxation time is much less than
increase in number density. (a) would be zero at all times.
[NCERT Exemplar] Ans. (d) (b) would be like a half-wave rectifier with
2. In the given figure V0 is the potential barrier positive cycles in output.
across a p-n junction, when no battery is (c) would be like a half-wave rectifier with
connected across the junction. negative cycles in output.
[NCERT Exemplar] (d) would be like that of a full-wave rectifier.
(a) 1 and 3 both correspond to forward bias Ans. (c)
of junction. 6. What happens during regulation action of a
(b) 3 corresponds to forward bias of junction Zener diode? [NCERT Exemplar]
and (a) The current in and voltage across the
1 corresponds to reverse bias of junction. Zener remains fixed.
(c) 1 corresponds to forward bias and 3 (b) The current through the series resistance
corresponds to reverse bias of junction. (Rs) does not change.
(d) 3 and1 both correspond the reverse bias (c) The Zener resistance is constant.
of junction. Ans. (b) (d) The resistance offered by the Zener
3. In figure given, assuming the diodes to be changes. Ans. (d)
ideal [NCERT Exemplar] 7. Silicon is a semiconductor. If a small amount
D1 of As is added to it, then its electrical

-10 V A conductivity__________ Ans. increases


D2 8. When the electrical conductivity of a
B semiconductors due to the breaking of its
covalent bonds, then the Semiconductor is
(a) D1 is forward biased and D2 is reverse said to be____________ . Ans. intrinsic
biased and hence current flows from A to B. 9. Electrical conductivity of a semiconductor.
(b) D2 is forward biased and D1 is reverse and (a) decreases with the rise in its temperature.
hence ne current flows from B to A and vice (b) increases with the rise in its temperature.
versa. (c) does not change with the rise in its
(c) D1 and D2 are both forward biased and temperature.
current flows from A to B.

Advent Tutorial Front of Nehru Inter College Banarsidas, Auraiya


KP Singh 7248370933 ks5122213@gmail.com Page 10
Chapter- 13 Semiconductor devices and circuit
(d) first increases and then decreases with the
rise in its temperature. Ans. (b)

Input
10. The forbidden energy band gap in conductors,
semiconductors and insulators are EG1, EG2,
and EG3, respectively. The relation among

Output
them is A B C D
(a) EG1 = EG2 = EG3 (b) EG1 < EG2 < EG3
(c) EG1 > EG2 > EG3 (c) EG1 < EG2 > EG3
Ans. (b) (a) A, C (b) B, D
11. In an insulator, the forbidden energy gap (c) B, C (d) A, D Ans. (b)
between the valence band and the conduction 15. Draw energy band diagram for an
band is of the order of _____________ . Ans. 5 eV intrinsic semiconductor at T = 0K.
12. An-type semiconductor is Ans.
EC
(a) negatively charged.
(b) positively charged. Electron energy
(c) neutral. level Eg

(d) none of these Ans. (c)


13. In the half-wave rectifier circuit shown.
EV
Which one of the following waveforms is true
for VCD the output across C and D?
16. Draw energy band diagram for an
intrinsic semiconductor at T > OK.
Ans.
EC

Electron energy
level Eg

EV

(a) (b)
17. Why are the elemental dopants mainly taken
from 13th and 15th group, for doping Silicon
(c)
(d) or Germanium?
Ans. The dopant has to be such that it does
not distort the original pure semiconductor
Ans. (b) lattice. So that the sizes of the dopant and
14. A full-wave rectifier circuit along with the the semiconductor atoms should be nearly the
input and output voltages is shown in the same.
figure, The contribution to output voltage 18. What is a hole? What is its physical
from diode 2 is significance?

Advent Tutorial Front of Nehru Inter College Banarsidas, Auraiya


KP Singh 7248370933 ks5122213@gmail.com Page 11
Chapter- 13 Semiconductor devices and circuit
Ans. Hole is the vacancy of electron in valence
band. The vacancy with the hole behaves as V
an apparent free particle with effective
I
positive charge.
19. Name one impurity each, which when added Ans. Solar cell.
to pure Si, produces (i) n-type, and (ii) p- 26. State the reason, why GaAs is most
type semiconductor. commonly used in making of a solar cell.
Ans. (i) for n-type, arsenic. Ans. It has higher absorption coefficient.
(ii) for p-type, Indium. 27. Name the type of biasing of a p-n junction
20. Why is the conductivity of n-type semi- diode so that the junction offers very high
conductor greater than that of the p-type resistance.
semi-conductor even when both of these have Ans. Reverse biasing.
same level of doping? 28. What is internal field emission?
Ans. In n-type semiconductor charge carriers Ans. The emission of electron from the host
are electrons and mobility of electrons is more atoms present in the p-n junction due to the
than that of holes. high electric field is known as internal field
21. Name two factors on which electrical emission or field ionisation.
conductivity of a pure semiconductor at a 29. Why is a typical solar cell drawn in fourth
given temperature depends. quadrant?
Ans. (i) Band gap (ii) Biasing Ans I- V characteristics of solar cell is drawn
22. What is an ideal diode? in the fourth quadrant because a solar cell
Ans. It is a p-n junction diode which offer does not draw current but supplies the same
zero resistance in forward biasing and infinite to the load.
resistance in reverse biasing, i.e. current flows 30. Why are materials like CdS or CdSe (Eg ~ 2.4
through it in one direction only. eV) not used for the fabrication of a solar cell?
23. What happens to the width of depletion layer Ans. Materials like CdS or CdSe (Eg ~ 2.4 eV)
of a p-n junction when it is (i) forward are not used because they will use only the
biased, (ii) reverse biased? [AI 2011] high energy component of the solar energy
Ans. (i) The width of depletion layer for photo-conversion and a significant part of
decreases. energy will be of no use.
(ii) The width of depletion layer increases.
24. What do you understand by a dynamic Short Answer Type Questions [2 Marks]
resistance of p-n junction diode. 31. C, Si and Ge have same lattice structure. Why
Ans. Dynamic resistance is the ratio of a small is C insulator, while Si and Ge are intrinsic
change in voltage ΔV to a small change in semiconductors? [NCERT Example]
current Δl, i.e. Ans. The 4 bonding electrons of C, Si or Ge
Δ𝑉
Δ𝐼
25. Name the junction diode whose I-V lie, respectively, in the second, third and
characteristics are drawn below [Delhi 2017] fourth orbit. Hence, energy required to take
out an electron from these atoms (i.e
ionisation energy Eg) will be least for Ge,
followed by Si and highest for C. Hence, the

Advent Tutorial Front of Nehru Inter College Banarsidas, Auraiya


KP Singh 7248370933 ks5122213@gmail.com Page 12
Chapter- 13 Semiconductor devices and circuit
number of free electrons for conduction in Ge Conduction Conduction
and Si are significant but negligibly small for Band EC Band EC

C. ED ≈0.01eV
Electron energy
32. What is doping? Why is it needed? Electron energy
level Eg
level Eg
Ans. (a) The deliberate addition of a desirable EA≈0.01- 0.05 e V
impurity is called doping. Valance Valance
(b) At room temperature the conductivity of band EV Band EV
intrinsic semiconductor is very low. Hence, n-type p-type
there is a necessity of improving their Semiconductor Semiconductor
conductivity. This can be done by doping T>0K T>0K
33. Distinguish between 'intrinsic' and 'extrinsic' 36. Distinguish between an intrinsic
semiconductors. [Delhi 2015] semiconductor and p-type semiconductor.
Ans. Intrinsic semiconductors Give reason, why a p-type semiconductor
(i) There are no impurity atoms crystal is electrically neutral, although nh >>
(ii) The number density of electrons is equal to ne [CBSE Foreign 2017]
the number density of holes, i.e. ne = nh Ans.
Extrinsic semiconductors Intrinsic semiconductor p-type semiconductor
(i) They are doped with impurity atoms (i) This form of semi- (i) It is doped
(trivalent/pentavalent). conductor does not semiconductor with
(ii) ne ≠ nH contain any impurity. trivalent impurity.
34. Carbon and silicon both have four valence (ii) Here ne = nh (ii) Here ne << nh
electrons each. How then are they In the formation of p-type semiconductor,
distinguished? [Delhi2011C] the number
Ans. Although both carbon and silicon have of negatively charged electrons is equal to
same lattice structure, the four bonding positively charged holes. So the p-type
electrons of carbon and Silicon are semiconductor thus formed is electrically
respectively in the second and third orbits. neutral
Thus, ionisation energy is less for silicon than 37. Draw the circuit diagrams showing how a p-
carbon. Hence, the number of free electrons in n junction diode is
silicon is more than of carbon. Thus, they can (i) forward biased and
be distinguished on the bases of their (ii) reverse biased. How is the width of
conductivity. depletion layer affected in the two cases?
35. Draw energy band diagrams of an n-type [Chennai 2019, AI 2011C]
and a p-type Semiconductor at temperature Ans. The following figures show the required
T > 0 K. Mark the donor and acceptor energy circuit diagrams
levels with their energies. [Dehradun 2019,
Foreign 2014]
Ans.

Advent Tutorial Front of Nehru Inter College Banarsidas, Auraiya


KP Singh 7248370933 ks5122213@gmail.com Page 13
Chapter- 13 Semiconductor devices and circuit
When an external load is connected, current
flows as:
(i) photocurrent is proportional to the
intensity of light.
(ii) change in reverse biased current is more
distinct in comparison to the change in
When the p-n junction is forward biased as in
forward biased current for same change in
Figure (i), the width of the depletion layer
the intensity of light Hence, photodiode is
decreases. When an external voltage is
used to detect optical signals
applied, the barrier potential is reduced
39. In the following diagram S is a semiconductor.
thereby, decreasing the width of depletion
Would you increase or decrease the value of R
layer. When the p-n junction is reverse biased
to keep the reading off the ammeter A
as in Figure (ii), the barrier potential
constant when S is heated? Give reason for
increases, thereby, increasing the thickness of
your answer. [AI 2017]
the depletion layer.
38. Explain with the help of a circuit diagram the
V S
working of a photodiode. Write briefly how it
is used to detect the optical signals. (Delhi
2013, Foreign Al 2017] R
Ans. When a photodiode is illuminated with A
light having photons of energy (hʋ) greater
Ans. Electric current, 𝐼 =
𝑉

than energy gap (Eg) of semiconductor, the 𝑅+𝑅𝑆


As the resistivity of the semiconductor
electron-hole pairs are generated in the
decreases on heating, so RS, will decrease and
depletion region. An electric field across the
to maintain constant current R should be
junction separates the holes and electrons.
increased.
Thus, the electrons reach n-side and holes
40. Name the semiconductor device that can be
reach p-side. Hence, the accumulation of
used to regulate an unregulated de power
electrons on n-side and holes on p-side
supply. With the help of I-V characteristics of
develops an emf.
this device, explain its working principle.
[Dehradun 2019, Panchkula 2019, Delhi
2011]

Advent Tutorial Front of Nehru Inter College Banarsidas, Auraiya


KP Singh 7248370933 ks5122213@gmail.com Page 14
Chapter- 13 Semiconductor devices and circuit
Ans. The semiconductor device is a zener grown on one-side by diffusion process. The
diode. It is observed from the graph that for other side of p-Si is coated with a metal
applied reverse biased voltage (breakdown (back contact). On the top of n-Si layer,
voltage, VZ) for a large variation of current, metal finger electrode (or metallic grid) is
zener voltage remains constant deposited. This acts as a front contact. The
41. Explain, with the help of a circuit diagram, metallic grid occupies only a very small
the working of a p-n junction diode as a fraction of the cell area (< 15%) so that light
half-wave rectifier. [Al 2014] can be incident on the cell from the top.
Ans. The secondary of the transformer
supplies ac voltage across A and B. When A is
at higher potential with respect to the
potential at B, the diode is forward biased
and it conducts. In the reverse condition,
diode does not conduct. This way output
voltage appears across the load resistance or
RL only for half cycle of ac voltage
The output voltage though still varying in
magnitude is flowing in only one direction 44. What are the basic processes involved in the
and is said to be rectified. generation of emf by a solar cell?
Ans. The three basic processes are: generation,
separation and collection-(i) generation of e-h
pairs due to light (with hʋ >E) close to the
junction; (ii) separation of electrons and holes
due to electric field of the depletion region.
Electrons are swept to n-side and holes to p-
side; (iii) the electrons reaching the n-side are
collected by the front contact and holes
42. Draw a circuit diagram showing the biasing of
reaching p-side are collected by the back
an LED. State the factor which controls (i)
contact. Thus, p-side becomes positive and
wavelength of light, and (ii) intensity of light
Side becomes negative giving rise to photo
emitted by the diode.
voltage.
Ans The following circuit shows the biasing of
45. What are the important criteria for the
an LED.
selection of a material for solar cell
(i) The wavelength of emitted light depends
fabrication?
upon the nature of the material of
Ans. The important criteria for the selection
diode/band gap.
of a material for solar cell fabrication are: (i)
(ii) Biasing of LED.
band gap (~1.0 to 1.8 eV), (ii) high optical
43. Explain how a solar cell is fabricated. [Foreign
absorption (~104 cm-1), (iii) electrical
2017]
conductivity, (iv) availability of the raw
Ans A p-Si wafer of about 300 µm is taken
material, and (v) cost.
over which a thin layer (-0.3 µm) of n-Si is

Advent Tutorial Front of Nehru Inter College Banarsidas, Auraiya


KP Singh 7248370933 ks5122213@gmail.com Page 15
Chapter- 13 Semiconductor devices and circuit
46. (a) In the following diagram, which bulb out 4. Draw V-I characteristics of a p-n junction
of B1 and B2 will glow and why? [AI 2017] diode. Answer the following questions, giving
D1 D2 reasons:
(i) Why is the current under the reverse bias
almost independent of the applied potential
B1
9V B2 up to a critical voltage?
(ii) Why does the reverse current show a
(b) Draw a diagram of an illuminated p-n sudden increase at the critical voltage?
junction solar cell. Name any semiconductor device which
operates under the reverse bias in the
breakdown region. [AI 2013]
5. (a) Describe the working of light emitting
diodes (LEDs).
(i) Which semiconductors are preferred to
make LEDs and why?
(c) Explain briefly the three processes due to
(ii) Give two advantages of using LEDs over
which generation of emf takes place in a solar
conventional incandescent low power lamps.
cell.
[Chennai 2019, Al 2011C]
6. (a) Why is a photodiode operated in the
Long Answer Type [I] Questions [3 Marks]
reverse bias mode? [Delhi 2019]
1. (a) Explain with the help of a diagram, how
(b) For what purpose is a photodiode used?
depletion region and potential barrier are
(c) Draw its I-V characteristics for different
formed in a junction diode. [Delhi 2019, 17]
intensities of illumination. [Foreign 2014; Al
(b) If a small voltage is applied to a p-n
2011C]
junction diode how will the barrier potential
7. The graph of potential barrier versus width of
be affected when it is (i) forward biased, and
depletion region for an unbiased diode is
(ii) reverse biased? [AI 2015]
shown in A. In comparison to A. graphs B and
2. How is a Zener diode fabricated? What causes
C are obtained after biasing the diode in
the setting up of high electric field even for
different ways. Identify the type of biasing in
small reverse bias voltage across the diode?
B and C and justify your answer.
Describe, with the help of a circuit diagram,
the working of Zener diode as a voltage
A B
regulator. [Dehradun 2019, AI 2015]
V(x)

V(x)

3. With what considerations in view, a


photodiode is fabricated? State its working
with the help of a suitable diagram. Even x x
though the current in the forward bias is
known to be more than in the reverse bias, C
V(x)

yet the photodiode works in reverse bias.


What is the reason? [Delhi 2015]

Advent Tutorial Front of Nehru Inter College Banarsidas, Auraiya


KP Singh 7248370933 ks5122213@gmail.com Page 16
Chapter- 13 Semiconductor devices and circuit
Explain why: 12. The circuit shown in figure uses a zener diode
LEDs are made of compound semiconductor of 12 V as a voltage regulator to run a 12 V,
and not by elemental semiconductors. [CBSE 6 W tape recorders on 110 V d.c. line. What
Sample Paper 2016] should be the value of the resistor R, if the
8. Draw a block diagram of a full-wave rectifier d.c. supply varies from 102 V-115 V?
with capacitor filter. Draw input and output [HOTS]
R
(filtered) voltage of rectifier.

Tape recorder
Long Answer Type [II] Questions [5 Marks] DC 102-115 V
9. (a) State briefly the processes involved in the
formation of p-n junction explaining clearly
how the depletion region is formed.
[Delhi 2017] 13. You are given three semiconductors: A, B and
(b) Using the necessary circuit diagrams, show C with respective band gaps of 3 eV, 2 eV
how the V-I characteristics of a p-n junction and 1 eV for use in photo-detector to detect .
are obtained in = 1200 nm. Select the suitable
(i) Forward biasing semiconductor. Give reasons.
(ii) Reverse biasing [Chennai 2019] 14. Draw the output waveform across the resistor
How are these characteristics made use of in (See figure) [Similar Dehradun 2019]
rectification? [Delhi 2014]
15. Can the potential barrier across a p-n
10. Explain briefly with the help of necessary junction be measured by simply connecting a
diagrams, the forward and the reverse biasing voltmeter across the junction?
of a p-n junction diode. Also draw their
characteristic curves in the two cases. [Delhi 16. (i) Name the type of a diode whose
2017] characteristics are shown in Figure (a) and
Figure (b).
Numerical Problems (ii) What does the point P in Figure (a)
11. The circuit shown in the figure has (two represent?
oppositely connected ideal diodes connected in (iii) What does the points P and Q in Figure
parallel. Find the current flowing through (b) represent?
each diode in the circuit. [Foreign 2013]
2Ω D2 17. A photo detector is made of a semiconductor
In 0.53 Ga 0.47 as with Eg = 0.73 eV. What is

D1 the maximum wavelength, which it can


3Ω
detect?

18. Three photo diodes D1, D2, and D3, are made
4Ω of semiconductors having band gaps of 2.5eV,
12 V
2eV and 3eV, respectively. Which ones will be
able to detect light of wavelength 6000 Å?

Advent Tutorial Front of Nehru Inter College Banarsidas, Auraiya


KP Singh 7248370933 ks5122213@gmail.com Page 17
Chapter- 13 Semiconductor devices and circuit
1. An n- type semi-conductor is (a). Increases

(a). Negatively charged (b). Decreases

(b). Positively charged (c). Remains same

(c). Neutral (d) First increases then decreases

(d) None of these

7. In a common emitter transistor, the current

2. A p-type semi-conductor has acceptor levels gain is 80. What is the change in collector

57 MeV above the valance band. The current, when change in base current is 250

maximum wavelength of light required to µA?

create a hole is (a). (80×250) µA (b). (80-250) µA

(a). 11.61×10-33 Å (b). 57×10-3 Å (c). (80+250) µA (d) (80/250) µA

(c). 217.1×103 Å (d) 57 Å

8. In common base amplifier, what is the

3. In a common transistor circuit, the current change in base current if that in emitter

gain is 0.98. On changing emitter current by current is 2 mA, 𝛼= 0.98

5.00 mA the change in collector current is (a). 0.04 mA (b). 7.96 mA

(a). 0.196 mA (b). 2.45 mA (c). 2 mA (d) 98 mA

(c). 4.9mA (d) 5.1mA


9. For Ge-crystal, the forbidden energy gap in
joules is
4. In common-base amplifier, the phase
(a). 1.12×10-19 (b). 1.76×10-19
difference between input signal voltage and (c). 1.6×10-19 (d) Zero
output voltage is

(a). 0 (b). 𝜋/4 10. The electrical conductivity of semiconductor

(c). 𝜋/2 (d) 𝜋 increases when electromagnetic radiation of


wavelength less than 2480 mm is incident
on it. The band gap in eV is
5. The forbidden energy band gap between
(a). 0.9 (b). 0.7
conductor, semiconductor and insulator are
(c). 0.5 (d) 1.1
EG1, EG2 and EG3 respectively. Thus,

(a). EG1 = EG2 = EG3 11. Which of the following Not true?
(b). EG1 > EG2 > EG3 (a). Resistance of intrinsic semiconductors

(c). EG1 < EG2 < EG3 decreases with increase in temperature
(b). Doping sure Si with trivalent impurities
(d) EG1 < EG2 > EG3
gives p-type semiconductor
(c). Majority carriers in n-type
6. In forward bias the width of potential
semiconductor are holes
barrier in p-n junction diode

Advent Tutorial Front of Nehru Inter College Banarsidas, Auraiya


KP Singh 7248370933 ks5122213@gmail.com Page 18
Chapter- 13 Semiconductor devices and circuit
(d). A p-n junction diode can act as a 18. The resistance of a pure semiconductor.
semiconductor diode When small quantity of arsenic is added to
it, its conductivity
12. Copper has fcc lattice with interatomic (a). Increases (b). Decreases
spacing equal to 2.54 Å. The value of lattice (c). Remains the same (d) Become zero
constant for this lattice is
(a). 3.59 (b). 2.54 19. The impurity atoms with which pure silicon
(c). 1.27 (d) 5.08 should be doped to make a p-types
semiconductor are those of
13. The peak voltage in the output of a half wave (a). Phosphorus (b). Boron
diode rectifier fed with sinusoidal signal (c). Antimony (d) Aluminium
without filter is 10 V. the d.c. component of
output voltage is 20. The diode current in forward and reverse
(a). 10√2 V (b). 10𝜋 Volt bias is measured in
(c). 10 V (d) 20/𝜋 V (a). Milliamperes only
(b). Microamperes only
(c). Milliampere and Microampere respectively
14. In common-base mode of transistor, the
(d) Microampere and Milliampere respectively.
collector current is 5.488 mA and for
21. A transistor is a device in which the
emitter current of 5.60 mA, the value of
(a). Emitter is always at forward bias
base current amplification factor
(b). Emitters is always at reverse bias
(a) 48 (b). 49 (c). 50 (d) 51
(c). Collector is always at forward bias
(d) collector is always at reverse bias
15. In an n-p-n transistor, the collector current
is 24 mA. If 60% of electrons reach collector,
22. The current gain for p-n-p transistor 𝛼 is
its base current in mA is
0.9. the value of current gain 𝛽 would be
(a). 36 (b). 26 (c). 16 (d). 6
(a). 0.9 (b). 1
(c). 9 (d) 90
16. If the ratio of concentration of electrons to
that of holes is 7/5 and ratio of current
23. The most commonly employed circuit of a
7/4, then what is the ratio of their drift
transistor amplifier is
velocities?
(a). Common-base amplifier because its
(a). 4/7 (b). 5/8 (c). 4/5 (d) 5/4
current gain is maximum
(b). Common-base amplifier because its
17. Germanium is a semiconductor. When small
current gain is minimum
quantity of arsenic is added to it, its
(c). Common-emitter amplifier because its
conductivity
current gain is maximum
(a). Zero (b). Very low but not zero
(d). Common-emitter amplifier because its
(c). Very high (d) Infinity
current gain is minimum

Advent Tutorial Front of Nehru Inter College Banarsidas, Auraiya


KP Singh 7248370933 ks5122213@gmail.com Page 19

You might also like